Você está na página 1de 129

Bank Probationary Officer

Quantitative Aptitude

NUMBER SYSTEM

Numbers are collection of certain symbols or figures called digits.


The common number system in use is
Decimal number system. In this system
we use ten symbols each representing a
digit. These are 0,1,2, 3,4,5,6,7,8 and 9. A
combination of these figures representing a
number is called a numeral. We also have
Binary number system. It uses only 0 and
1. There are other number systems too.
We would confine our discussion to Decimal number system in this topic.
Every digit has a face value which equals
the value of the digit itself, irrespective of its
place in the numeral. Each digit in a number or numeral has a place value besides its
face value. For a given number / numeral we
begin from the extreme right as unit's place,
ten's place, hundred's place, thousand's
place and so on.

Units
10

Tens
101

Thousands
103
Hundreds
102

Ten thousands
104

10 5

Lacs

Example : The number 795698 is represented as shown below :

We read it as :
Seven lac ninty five thousand six hundred
and ninty eight.
In the above number,
The place value of 8 is (8 x 10) = 8
The place value of 9 is (9 x 101) = 90
The place value of 6 is (6 x 102) = 600 and
so on.

The face value of a digit in a number is the


value of digit itself wherever it may be .
Thus, the face value of 7 in the above numeral
is 7, the face value of 9 is 9 and so on.

Types and Nature of Numbers.


Natural numbers : These are also called
counting numbers as these numbers are the
ones which we use for counting purpose. It is
represented by N.
eg : N = {1, 2, 3,............................}
Whole numbers : It includes all natural
numbers with zero. We can denote it by W
.
Eg : W = {0,1,2, ....................}
Integers : It includes all whole numbers
along with negative numbers. It is represented by I
eg : I = { -............-2,-1,0,1,2,......}
Thus we see that whole numbers are nothing
but positive integers and zero. Similarly, natural numbers consists of positive integers.
Even number : A number which is completely divisible by 2 is called an even number. In other words such numbers has 2 as
a factor when they are written as a product
of different numbers.
Eg : 2, 4, 6, 8....................
A number is said to be a factor or submultiple of another when it divides the other exactly. For example 5 and 3 are factors of 15.
Odd number : These numbers are not
completely divisible by 2. In other words,
a number which is not even is an odd number.
Eg: 1, 3, 5, 7,.................
It may be noted that zero is an exception
to this even-odd classification.

Real number : Any measurement carried out in the physical world leads to some
meaningful figure or value or number. This
number is called real number. It consists
of two groups :
(i)

(ii)

Rational number : A rational number can


always be represented by a fraction of

number but it is not a prime number as it is


divisible by 3.

Composite number : A composite number is one which has other factors besides
itself and unity. Thus it is a non-prime number.
Eg : 4, 6, 9, 14, 15, ......................

p
the form q , , where p and q are integers

Note : (1) 1 is neither prime nor composite.

and q is not equal to zero. All integers


and fractions are rational numbers.

(2) A composite number may be even or


odd.

Irrational number : An irrational num-

The number of ways in which a number N


can be expressed as the product of two
factors which are relatively prime to each
other. is 2M-1, where M is the number of different prime factors of N.

ber can't be expressed in the form of


p
,
q

where q 0

Eg : 540 = 22 x 33 x 5
Eg :

3,

2,

It gives only an approximate answer in


the form of a fractional or decimal number.
The digits after the decimal point are nonending. The same holds true for
which is again irrational. Alternatively we
can say that an infinite non-recurring decimal number is an irrational number.

Prime factors of 540 are 2,3 and 5


M
Number of ways = 23-1 = 4
ie 20 x 27, 4 x 135, 108 x 5, 540 x 1.

Consecutive Integers : These are series of numbers differing by 1 in ascending


order.

Prime number : A prime number is a


number which has no factors besides itself and unity, ie, it is divisible only by itself and 1 and by no other numbers.

Eg : 5, 6, 7, 8, ...................

Eg : 2, 3, 5, 7, 11, 13, 17, 19, 23,------

gers. It is expressed in the form

The largest prime number known so far


is 22281 1 which is a number of about
700 digits.
Note :
(1) 2 is the only even number which is
prime.
(2) All prime numbers other than 2 are odd
numbers but all odd numbers are not
prime numbers, for example, 9 is an odd

Fractions : A fraction is a number which


represents a ratio or divisions of two intea
,
b

where 'a' and 'b' are integers. 'a' is called


the numerator and 'b' is called the denominator. A fraction cannot have zero (b 0
as a denominator since division by zero is
not defined.
Zero divided by any integer is always zero.
A fraction with 1 as the denominator is the
same as its numerator.
Two fractions are equivalent if they represent the same ratio or number. So if we

multiply or divide the numerator and denominator of a fraction by the same nonzero integer, the result obtained will be
equivalent to the original fraction.

4.

Divisibility by 5 : A given number is divisible by 5, if its unit digit is either 0 or 5.

5.

Divisibility by 6 : A given number is divisible by 6, if it is divisible by both 2 and


3.

6.

Divisibility by 8 : A given number is divisible by 8, if the number formed by its


last three digits is divisible by 8 or the last
three digits are 000.

7.

Divisibility by 9 : A given number is


divisible by 9, if the sum of its digits is
divisible by 9.

8.

Divisibility by 10 : A given number is


divisible by 10, if its unit digit is 0.

9.

Divisibility by 11 : A given number is


divisible by 11, if the difference of the sum
of its digits at odd places and the sum of
its digits at even places, is either 0 or a
number divisible by 11.

10.

Divisibility by 12 : A given number is


divisible by 12, if it is divisible by both 3 and
4.

11.

Divisibility by 14 : A given number is divisible by 14, if it is divisible by both 2 and


7.

12.

Divisibility by 15 : A given number is


divisible by 15, if it is divisible by both 3 and
5.

13.

Divisibility by 16 : A given number is


divisible by 16, if the number formed by
the last 4 digits of the given number is
divisible by 16 or the last four digits are
0000.

14.

Divisibility by 18 : A given number is divisible by 18, if it is divisible by both 2 and 9.

15.

Divisibility by 20 : A given number is divisible


by 20, if it is divisible by both 4 and 5.

16.

Divisibility by 7, 13, 17 and 19 :

Decimals : A collection of digits (0, 1,


2, .........9) after a period ( called the decimal point) is called a decimal fraction.
Eg : 0.629, 0.013, 10.652, etc.
Every decimal number represents a fraction. These fractions have denominators
with power of 10.
eg : 0 .5

5
4
5
, 0 .4 5

10
10 100

A number containing a decimal point is


called a decimal number.

Mixed number : A mixed number consists of a whole number and a fraction.


Eg :

3
5

is a mixed fraction.

This is equivalent to 4 +
can be written as 4

3
and hence
5

3
3
4
5
5

Here 4 is the whole number and

3
5

is

the fraction.

TEST OF DIVISIBILITY
1.

Divisibility by 2 : A given number is divisible by 2, if its unit digit is any one of 0,


2, 4, 6 and 8.

2.

Divisibility by 3 : A given number is divisible by 3, if the sum of its digits is divisible by 3.

3.

Divisibility by 4 : A given number is divisible by 4, if the number formed by its


last two digits is divisible by 4 or the last
two digits are 00.

There is no direct method to test the


divisibility by 7, 13, 17 and 19. In all the

above cases, the operator for checking is


the number itself or its prime factors. But
here the operator is not the number itself or
its factors.
In this case each number has a particular
operator. For example, the operator for 7 is
2, 13 is 4, 17 is 5 and 19 is 2.
To find the operator for 7; consider the
multiples of 7, such that the multiple is very
near to the multiple of 10 (ie 21 =3 x 7 is
near to 20 = 2 x 10). Here 2 ( the number
multiplied by 10 to get 20) is taken as the
operator. Similarly the multiple of 13 (ie 3
x 13 = 39) near to the multiple of 10
(ie 40= 4 x 10)is 39. Here 4 (the number
multiplied by 10 to get 40) is taken as the
operator. Through the same way we can
determine the operator for 17 as 5 and that
for 19 as 2.
The multiple of 7 and 17 (ie 3 x 7 = 21 and
3 x 17 =51) are one more than the multiple of
10 (ie 20 = 2x 10 and 50 =5 x10), therefore
the operator for 7 and 17 are called one more
operator. Also the multiple of 13 and 19 (ie 3
x 13 = 39 and 1 x 19 = 19) are one less than
the multiple of 10 (ie 40 = 4 x 10, and 20 = 2
x 10), therefore the operator for 13 and 19
are called negative operators.
Divisibility by 7 :
To test the divisibility by 7, multiply the
last digit of the given number by the operator
of 7 (ie 2 ) and subtract the product from
the number obtained by removing the last
digit of the given number. Again take the
last digit of the difference and multiply by
the operator, the product is subtracted from
the number obtained by removing the last
digit of the just previous difference. The
above process is repeated. If the difference
obtained is zero or a multiple of 7, then we
say that the number is divisible by 7.
Eg : To test the number 3563 is divisible
by 7.

The operator for 7 is 2 and the last digit of


the given number is 3. Then its product is
3 x 2 = 6.
The number obtained by removing the last
digit is 356.
Then 356 - 6 = 350
Here the difference is 350 and its last digit
is 0. The product with operator is 2 x 0
= 0.
The number obtained by removing the last
digit of the difference is 35
Then 35 - 0 = 35.
Here 35 is a multiple of 7. Therefore the
number 3563 is divisible by 7.
Divisibility by 13
Multiply the last digit of the given number
by the operator for 13 (ie 4) and add the
product to the number obtained by removing the last digit of the given number. Again
take the last digit of the sum (previously
obtained) and multiply by the operator. Add
the product to the number obtained by removing the last digit of the sum (just previously obtained). The above process is
repeated. If at last the sum obtained is
the multiple of 13, then we say that the
given number can be divisible by 13.
Eg : To test the number '6539' is divisible
by 13.
The operator for 13 is 4, and the last digit
of the given number is 9; then its product
is 4 x 9 = 36.
The number obtained by removing the last
digit of the given number is 653.
Then, sum = 653 + 36 = 689
Here the sum is 689, its last digit is 9 and
its product with the operator is 9 x 4 = 36.

The number obtained by removing the last


digit of the sum is 68.

Which is the smallest of such numbers.?


Ans : By hit and trial we find that 555555
is divisible by 13.

Then sum, 68 + 36 = 104


Again 4 x 4 = 16

555555
42735
13

Then sum = 10 + 16 = 26
Here 26 is a multiple of 13. Ttherefore the
number 6539 is divisible by 13.
Divisibility by 17 :

Same as 7

Divisibility by 19 :

Same as 13.

the required number = 42735


6.

Ans :

SOLVED EXAMPLES.
1.

When the number is divided by 779, let


the quotient be k.
number = 779k + 47
= 19 x 41k + 19 x 2 + 9
= 19 (41k + 2) + 9

What is the difference in the place value


and the face value of 8 in 78975?
Ans :
The place value of 8 is 8000

So, when the number is divided by 19, the


quotient is (41k + 2) and the remainder is 9

The face value of 8 is 8


the required difference = 8000 - 8 =
7992
2.

A number when divided by 779 gives 47


as remainder. On dividing the same number by 19, what would be the remainder ?

7.

The numbers p, p + 2, p + 4 are all primes.


Find the value of p.

A positive number when decreased by 4


is equal to 21 times the reciprocal of the
number. Find the number.
Ans :

Ans : When p = 3, the given numbers are


3, 5, 7; which are prime.
3.

4.

ie x

21

x
- 4x -21 = 0

(x - 7) (x + 3) = 0 x = 7

Find the least number of five digits that is


exactly divisible by 456.

A certain number consists of two digits


whose sum is 9. If the order of the digits is
reversed, the new number is 9 less than
the original number. The original number
is :

Ans :

Ans :

The least five digit number = 10000.


when 10000 is divided by 456, the remainder is 424.

Let ten's digit be x and unit's digit be y.

the required number

(10x + y) - (10y + x ) = 9 x y 1

= 10000 + (456 -424) = 10032


5.

Then x - 4 =

If 42 k 8 is a multiple of 9, find the value of k


Ans : Sum of digits = (4 + 2 + k +8)
= (14 + k).
Now, least value of k for which (14 + k)
is divisible by 9 is k = 4.

Let the number be x

When a certain number is multiplied by


13, the product consists entirely of fives.

8.

Then x + y = 9

Solving x + y = 9 & x- y =1,


we get x = 5 & y = 4
the required number = 54

9.

A boy was asked to multiply a certain number by 25. He multiplied it by 52 and got
his answer more than the correct one by
324. The number to be multiplied was :

4.

4
5
th of a number. By mistake, he found
5
4

th of it and his answer was 180 more than


the correct answer. Find the given number.
1) 81
2) 890
3) 400
4) 500

Ans : Let the required number be x


Then 52x - 25x = 324
x

10.

324
12
27

5.

A number whose fifth part increased by 5


is equal to its fourth part diminished by 5.
Find the number.
Ans : Let the required number be x

ie

x x
10 x 200
4 5

6.
PRACTICE TEST
1.

2.

2) 80
4) 400

7.

3
2

1) 150

2)

3) 160

4) 300

Sum of two numbers is

1
1
rd of th of
3
5

195 and their product is

1
1
th of
th of
6
4

960. Find
them.
1) 1
3) 3

3
th of the amount of
5

Vishnu is equal to

5
th of the amount of
7

At an election, a candidate who gets

3
th
4

8.

One-third of the total marks are required


to pass an examination. A candidate who
gets 178 marks, fails by 22 marks. The
total marks in the examination are
1) 550
2) 600
3) 535
4) 660

9.

A sum is divided between Raju and Biju

1
rd of the difference between
3

2) 9
4) 27

4
2
9
th of a number exceeds its rd of
th
5
3
10

of the total votes, is elected by a majority


of 2000 votes. The total number of votes
polled and the number of votes secured
by the candidate who was elected, are respectively,
1) 4000, 3000
2) 8000, 6000
3) 4500, 2500
4) 5000, 3000

4
2
of a number exceeds its
rd by 20.
5
3

Find the number

3.

Ananthan such that

by 120. Find the number.


1) 600
2) 140
3) 800
4) 660

3
1
4
th of rd of thof a number is 80,
4
3
5

find that number


1) 300
3) 14

A sum is divided between Vishnu and

Ananthan. If Vishnu gets Rs. 750, how


much does Ananthan get?
1) Rs. 450
2) Rs. 600
3) Rs. 630
4) Rs. 730

x
x
5 5
5
4

Then

In an examination, a student was asked to find

such that

5
th of the amount of Raju is
6

equal to

encyclopaedias. Find the total number of


books in the library.
1) 1,200
2) 1,500
3) 2,000
3) 750

4
th the amount of Biju. If Raju
5

gets Rs. 240, find the sum.


1) Rs. 500
2) Rs. 250
3) Rs. 490
4) Rs. 400
10.

The number of students in a school are


1
th of it. Find the total
4

1125 more than


strength
1) 175
3) 2000
11.

12.

17.

18.

In an examination, a student was asked


to find

5
of a number. By mistake, he
7

found

7
of it. His answer was 96 more
5

7
7
th of a number exceeds its
th by
8
16

5
th of 120% of a number is 235 more
8

84. Find

than 245. Find that number.


1) 523
2) 684
3) 576
4) 640

1) 144
3) 138
19.

1
A basket contains 44 more than
th of
5

1
In a library, of the books are story books,
2
2
rd of the remaining books are reference
3

books and the remaining 250 books are

3
th of the number..
4

2) 192
4) 204

The sum of three consecutive even integers is 132. Find the difference between
3 times the least and

total number of apples. How many apples


are there in the basket ?
1) 66
2) 55
3) 65
4) 33
15.

1
3
of the adults are male, th of
2
5

than the correct answer. The given number is


1) 196
2) 128
3) 156
4) 140

3
1
th of th of Rs. 450.
5
9

2) Rs. 705
4) Rs. 605

5
th of the population are
9

the adult females are illiterate. If 800 adult


females are illiterate, then the population
of the village is
1) 4,000
2) 4,800
3) 9,000
4) 5,600

The amount that Satheesh had was Rs.

Find the amount.


1) Rs. 700
3) Rs. 625

14.

In a village,
adults.

2) 1500
4) 1225

A water tank having 1300 litres of capacity was filled by adding 75 litres of water
and as many buckets of water as each
bucket had a capacity. What was the
capacity of each bucket in litres ?
1) 32
2) 35
3) 48
4) 45

675 more than

13.

16.

1) 44
3) 144
20.

1
of the greatest.
2

2) 89
4) 103

The sum of four consecutive numbers is


90. Find the difference between five times
the third number and 4 times the fourth
number.
1) 21
2) 29
3) 24
4) 19

21.

24 is divided into two parts such that 6


times the first part exceeds 4 times the
second part by 4. Find the first part.
1) 14
2) 16
3) 10
4) 12

22.

The sum of 11 results is 55. The sum of


the first 6 results is 30 and that of the last
6 is 48. Find the sixth result.
1) 20
2) 24
3) 13
4) 23

23.

The total age of 9 boys is 128. The total


age of the first 4 boys is 62 and that of the
next 4 boys is 48. Find the age of the 9th boy.
1) 16
2) 18
3) 12
4) 17

24.

340 soldiers are arranged in a parade such


that the number of soldiers in each column is the same as the number of columns. Find how many soldiers are excluded from the group to make such an
arrangement possible.
1) 16
2) 21
3) 14
4) 24

25.

The smallest number which must be subtracted from 1300 to make it a perfect
square is
1) 2
2) 3
3) 4
4) 6

26.

Three books and a pen cost Rs. 70 and 2


pens and a book cost Rs. 40. How much
will a book and a pen cost ?
1) Rs. 10
2) Rs. 20
3) Rs. 30
4) Rs. 40

27.

the class. The teacher contributed Rs.


13 to make the total collection of Rs.49.
How many students are there in the class?
1) 70
2) 50
3) 60
4) 65
28.

The total marks secured by A, B and C are


540. A's marks were three times that of B's
and B's marks were twice that of 3) A's
marks are how much more than those of C ?
1) 154
2) 300
3) 265
4) 286

29.

When 6 is added to a number and the sum


is multiplied by 8, the result is same as
when 26 is multiplied by 10 and 12 is
added to the product. The number is
1) 28
2) 23
3) 26
4) 25

30.

Two pens and a pencil cost Rs. 18 and


two pencils and a pen cost Rs. 12. Find
the cost of a pen
1) 8
2) 2
3) 16
4) 6

31.

If the square of a number of two digits is


subtracted from the square of the number
formed by interchanging the digits, the
largest number by which the result is always divisible is
1) 9
2) 10
3) 11
4) 99

32.

The smallest number, which when subtracted from the sum of squares of 11 and
13 gives a perfect square, is
1) 1
2) 4
3) 5
4) 9

Each student in a class contributed as


many paise as the number of students in

ANSWERS TO PRACTICE TEST


1.
8.
15.
22.
29.

(4)
(2)
(2)
(4)
(1)

2.
9.
16.
23.
30.

(1)
(3)
(2)
(2)
(1)

3.
10.
17.
24.
31.

(1)
(2)
(4)
(1)
(4)

4.
11.
18.
25.
32.

(3)
(2)
(1)
(3)
(1)

5.
12.
19.
26.

(3)
(2)
(4)
(3)

6.
13.
20.
27.

(1)
(4)
(4)
(3)

7.
14.
21.
28.

(1)
(2)
(3)
(2)

BANK PROBATIONARY OFFICER


QUANTITATIVE APTITUDE
BASIC ARITHMETIC OPERATIONS
ADDITION

(a) 748 should be understood as seven


forty eight, not as seven hundred and forty
eight.

(i). Positive no. + Positive no. = Positive no.


Eg : + 2 + + 5 = + 7
(ii)

(b) 1098 should be understood as ten


ninety eight, not as one thousand and
ninety eight.

Negative no. + Negative no = Negative no.


Eg : -4 + - 6 = - 10

(iii)

Negative no. + Positive no. = Differece of


nos. and sign of greater no.
Eg : + 9 + - 11 = - 2, - 10 + + 4 = - 6
QUICKER & SHORT CUT METHODS
FOR ADDITION

1.

2.

In the Bank PO examination, there will


be a lot of situations in which you will have
to add various numbers. But the most
important thing is to add numbers in lightning speed. You should develop the habit
of seeing the numbers and adding them
instantly. You will not have the time to
write down the numbers with a pen on a
piece of paper and calculate in the usual
manner.
The moment you see
9 + 5, the number 14 should flash in your
mind.
As soon as you see
7 + 4 + 9, the number 20 should come.
Remember, you should not even read the
numbers as seven plus four plus nine.
Reading is time consuming. You just see
and calculate. Your eye can recognise
these numbers as 7 + 4 + 9 and instantly
the mind can come out with the answer 20.

3.

When you see a number, understand and


represent it with shorter possible words.

(c) 89876 should be eighty nine eight


seven six.
4.

Double column addition will enable you


to add numbers quicker and faster.
(a) Take the example of 78 + 65. The
moment you see the numbers visualise
in your mind that they are 78 + 60 + 5.
This way you can straight away get the
answer 143.
(b) 84 + 43 + 16 should be visualised as
(84 + 40 + 10 + 9) = 143.
(c) 6328 + 4233 + 2495. Here try the
double column addition
63

28

42

33

24

95

130

56

Ist double column is 28 + 33 + 95 =


28 + 30 + 90 + 8 = 156.
Write 56, and 1 is carried.
2nd double column is 63 + 42 + 24 + 1
= 63 + 40 + 20 + 7 = 130
Once you master double column addition,
you can easily visualise numbers in the
addable form and add numbers horizon-

BANK PROBATIONARY OFFICER


QUANTITATIVE APTITUDE
BASIC ARITHMETIC OPERATIONS
ADDITION

(a) 748 should be understood as seven


forty eight, not as seven hundred and forty
eight.

(i). Positive no. + Positive no. = Positive no.


Eg : + 2 + + 5 = + 7
(ii)

(b) 1098 should be understood as ten


ninety eight, not as one thousand and
ninety eight.

Negative no. + Negative no = Negative no.


Eg : -4 + - 6 = - 10

(iii)

Negative no. + Positive no. = Differece of


nos. and sign of greater no.
Eg : + 9 + - 11 = - 2, - 10 + + 4 = - 6
QUICKER & SHORT CUT METHODS
FOR ADDITION

1.

2.

In the Bank PO examination, there will


be a lot of situations in which you will have
to add various numbers. But the most
important thing is to add numbers in lightning speed. You should develop the habit
of seeing the numbers and adding them
instantly. You will not have the time to
write down the numbers with a pen on a
piece of paper and calculate in the usual
manner.
The moment you see
9 + 5, the number 14 should flash in your
mind.
As soon as you see
7 + 4 + 9, the number 20 should come.
Remember, you should not even read the
numbers as seven plus four plus nine.
Reading is time consuming. You just see
and calculate. Your eye can recognise
these numbers as 7 + 4 + 9 and instantly
the mind can come out with the answer 20.

3.

When you see a number, understand and


represent it with shorter possible words.

(c) 89876 should be eighty nine eight


seven six.
4.

Double column addition will enable you


to add numbers quicker and faster.
(a) Take the example of 78 + 65. The
moment you see the numbers visualise
in your mind that they are 78 + 60 + 5.
This way you can straight away get the
answer 143.
(b) 84 + 43 + 16 should be visualised as
(84 + 40 + 10 + 9) = 143.
(c) 6328 + 4233 + 2495. Here try the
double column addition
63

28

42

33

24

95

130

56

Ist double column is 28 + 33 + 95 =


28 + 30 + 90 + 8 = 156.
Write 56, and 1 is carried.
2nd double column is 63 + 42 + 24 + 1
= 63 + 40 + 20 + 7 = 130
Once you master double column addition,
you can easily visualise numbers in the
addable form and add numbers horizon-

tally, as it will provide you lightning speed


in addition.
5.

find the number that should be added to


this 7 to get 6 of 9687. But it is not possible to get such a positive number. So
treat 6 as 16 and this 1 is carried out for
the next step. Here 9 should be added to
7 to get 16. Write 9 as the hundredth
place digit of the answer.

For addition of numbers containing decimals, the same procedure of double column addition can be used.
Eg. 369.003 + 9.63 + 0.02 + .0003 + 948
= 1326.6533

6.

In the next step, 4 + 2 + 1 = 7, the 1 is got


from the previous step. Here 2 should be
added to this 7 to get 9 of 9687.

In the case of problems involving both operations addition and subtraction, subtract
the sum of all the negative terms from the
sum of all the positive terms.

9687 - 4363 - 2401 = 2923.


Similarly,

Eg. Find the value of 571 - 412 + 173 - 65


- 78 + 300

6884 - 2361 - 1592 = 2931


4328 - 325 - 659 = 3344
8203 - 3987 - 1697 = 2519.

Sum of positive terms = 571 + 173 + 300


= 1044
Sum of negative terms = 412 + 65 + 78 =
555

MULTIPLICATION
(i)

Positive no. x Positive no. = Positive no.


Eg : 8 x 3 = 24

(ii)

Negative no. x Negative no. = Positive no.

Required value = 1044 - 555 = 489


QUICKER & SHORT CUTS :
SUBTRACTION
Subtraction can be done through addition easily.
Eg : 9687 - 4363 - 2401 = ?
To find the answer, add all the unit's place
digits of the negative integers. (ie) 3 + 1 =
4. Now find the number that should be
added to 4 to get 7 of 9687. It is 3 and
write 3 as the unit's place digit of the answer.
Now add all the ten's place digit of the
negative numbers. (ie) 0 + 6 = 6. The
number that should be added to 6 to get 8
of 9687 will be the ten's place digit of the
answer. It is 2.
Now add all the hundred's place digits of
the negative numbers. (ie) 3 + 4 = 7. Now

Eg : -12 x -10 = 120


(iii)

Positive no. x Negative no. = Negative no.


Eg : 3 x -15 = -45

(iv)

Negative no. x Positive no. = Negative no.


Eg : -15 x 3 = -45
DIVISION

(i)

Positive No. Positive No = Positive


No.
Eg : 12 3 = 4

(ii)

Negative No. Negative No.= Positive No.


Eg: -12 -3 = 4

(iii)

Positive No Negative No = Negative No.


Eg : +12 (-3) = -4

(iv)

Negative No. Positive No. = Negative No.


Eg : -12 3 = -4

PRACTICE TEST
1.

3543 + 6413 + 5438 = ?


(1) 14294
(3) 15864

2.

15.

(2) 10072
(4) 9962
16.

(2) 861.4793
(4) 851.4893

(2) 3513
(4) 1029

17.

(2) 2656
(4) 2156

(2) 4123
(4) 5324

(2) 20100
(4) 20000

8
5

(2)

(3)

7
2

(4)

20.

3
5
2
5

6142 + ? = 5139 + 8136


(2) 7133
(4) 7033

91 x 33 - 33 = ?
(2) 3003
(4) 2970

64640 160 + 120 = ?


(1) 52.40
(3) 524

81038 - ? = 61038
(1) 19099
(3) 19999

(1)

(1) 0
(3) 91
19.

(2) 20202
(4) 2020220

6 x 21x 24
?
36 x 7 x 15

(1) 7313
(3) 6863
18.

(2) 10698.12
(4) 16989.05

282828280 14 = ?
(1) 202020
(3) 20202020

1352 + 4352 + ? = 9827


(1) 4213
(3) 3215

10.

(2) 10876.69
(4) 10866.69

(2) 14.875
(4) 14.975

2589.47 + 3009.59 + 5099.09 = ?


(1) 11609.85
(3) 19808.15

1286 + 655 - 423 + 638 = ?


(1) 1146
(3) 2056

9.

14.

(2) 53451
(4) 54361

15.04 - 0.065 = ?
(1) 15.795
(3) 14.957

4628 - 954 - 1253 = ?


(1) 2421
(3) 4232

8.

(2) 20223
(4) 13403

832.9 + 6.73 + 11.8393 = ?


(1) 851.4693
(3) 851.4793

7.

13.

8888 + 888 + 88 + 8 = ?
(1) 9872
(3) 8962

6.

(2) 176134
(4) 168282

(2) 1699
(4) 1409

549 x 99 = ?
(1) 55451
(3) 54351

8000.3 + 990.59 + 1885.8 = ?


(1) 10877.69
(3) 10886.69

5.

12.

98854 -64321 - 12512 = ?


(1) 22021
(3) 20032

4.

(2) 15394
(4) 15495

7329 - 2564 = 3256 + ?


(1) 1509
(3) 1599

92431 + 64273 + 10428 = ?


(1) 177232
(3) 167132

3.

11.

(2) 89.77
(4) 64.40

879 x 37 x 8 = ?
(1) 32523
(3) 260184

(2) 292707
(4) 257224

21.

21932 + 67 + 98232 + 100 = ?


(1) 121331
(3) 120231

22.

23.

25.

30.

(1) 90
(3) 2.25

37.

38.

(1)

1
4

(2)

1
3

(3)

1
2

(4)

1
5

0.9329 - 0.7321 + 4.329 + 0.002 = ?

39.

(2) 45.3180
(4) 4.2514

3 x 0.3 x 0.03 x 0.003 = ?


(2) 0.000081
(4) 0.00081

64 0.008 = ?
(2) 800
(4) 0.8

0.999999 0.011 = ?
(1) 90.908
(3) 9.0908

(2) 2534
(4) 0

(2) 4.004
(4) 0.004

(1) 80
(3) 8000

(2) 7422
(4) 7412

(2) 9
(4) 22.5

362

(1) 0.81
(3) 0.081

(2) 31.5
(4) 50

32.4 7.2
?
27 26.5

9 x 72

(1) 4.5318
(3) 0.45318
36.

182 x 14 - 14 = ?
(1) 2698
(3) 2674

35.

(2) 24289
(4) 22289

13025 + 1019 - ? = 7622


(1) 6412
(3) 6422

29.

(2) 488345
(4) 589425

756 18+ 6 = ?
(1) 48
(3) 46

28.

34.

(2) 235439
(4) 327484

0.0076
?
19
.

(1) 0.0004
(3) 0.04

(2) 16
(4) 48

43488 - 34567 + 14368 = ?


(1) 23289
(3) 13289

27.

33.

(2) 575
(4) 454

5584 x 51 = ?
(1) 284784
(3) 278484

4935 x 101 = ?
(1) 498435
(3) 489345

26.

(2) 74890
(4) 72880

5099.09 + 3009.59 + 2589.47 = ?


(1) 19608.15
(2) 10698.15
(3) 11609.85
(4) 16089.05

6945 + 977 + ? + 59 = 8435


(1) 1044
(3) 765

32.

12.48 3.9
?
39 38.8

(1) 8
(3) 24
24.

(2) 120331
(4) 100331

2740 x 27 = ?
(1) 73980
(3) 73990

31.

(2) 909.08
(4) 90.909

173 x 240 = 48 x ?
(1) 495
(3) 685

(2) 545
(4) 865

40.

219 + ? = 7608 - 5719


(1) 2108
(3) 1680

41.

(2) 1670
(4) 1570

140 x 20 8 x 75
?
11 x 50

46.
(2) 2
(4) 4
47.

1000
101

48.
(2)

44.

101
(4)
100

625 x 54 = ?
(2) 34750
(4) 33850

0.023 x 0.5 x 30 = ?
(2) 0.0345
(4) 3.45

89.467 - 45.971 + 9.991 = ?


(1) 42.505
(3) 43.404

1.113 - 0.8321 = ?
(1) 0.2809
(3) 0.2819

(2) 24240
(4) 4800

(1) 0.00345
(3) 0.345

100
101

49.
100
(3)
1001

608 x 8 - 48 = ?

(1) 33750
(3) 43750

(2) 1263470
(4) 122670

32 3232 x 100 = ?
(1)

(2) 20
(4) 5

(1) 2424
(3) 4816

1265 x 998 = ?
(1) 1262470
(3) 226470

43.

480 x 8 160
?
160 x 5

(1) 15
(3) 1.5

(1) 152
(3) 75
42.

45.

50.
(2) 0.3809
(4) 0.2009

(2) 32.515
(4) 53.487

13284 81 = ?
(1) 165
(3) 184

(2) 164
(4) 124

ANSWERS TO PRACTICE TEST

1.

(2)

2.

(3)

3.

(1)

4.

(2)

5.

(1)

6.

(1)

7.

(1)

8.

(4)

9.

(2)

10.

(4)

11.

(1)

12.

(3)

13.

(4)

14.

(2)

15.

(3)

16.

(4)

17.

(2)

18.

(4)

19.

(3)

20.

(3)

21.

(2)

22.

(1)

23.

(2)

24.

(2)

25.

(1)

26.

(1)

27.

(1)

28.

(3)

29.

(2)

30.

(2)

31.

(4)

32.

(1)

33.

(4)

34.

(3)

35.

(1)

36.

(2)

37.

(3)

38.

(4)

39.

(4)

40.

(2)

41.

(4)

42.

(1)

43.

(2)

44.

(1)

45.

(4)

46.

(3)

47.

(1)

48.

(3)

49. (4)

50.

(2)

Bank Probationary
Quantitative Aptitude

SIMPLIFICATION

VBODMAS

'VBODMAS' (Vinculum - Bracket - Of - Division


- Multiplication - Addition - Subtraction) rule
should be applied for solvingproblems involving
one or more mathematical operationslike multiplication, division, addition, subtraction etc.
Such problemsare solved in the order of vinculum, bracket, of, division, multiplication, addition and subtraction. Remember 'Of' in
VBODMAS means multiplication.

12 5 1
1
1
x 3 3
5 4 6
6
6

3 ( 8 5 ) ( 4 2 ) 2
?

1 3

4.

3 4

3 3 2

1
3

1 3

3 3 2 x

3 4

Solved Examples
1.

45 - 4 x 6 - 5 + 14 7 = ?
45 - 4 x 6 - 5 + 14 7 = 45- 24- 5 + 2 = 18

2.

21 3 (10 - 3) - 20 + 1 = ?
= 21 3 x 7 - 20 + 1
= 7 x 7 - 20 + 1
= 49 - 20 + 1 = 30

3.

3 of

13

3 3
3
7

5.

4 4 1
?
5 5 6

3 x17
13

3 x1 3
13

3 x1 7
17

(4 4 4 ) 4
?
666 6

( 4 4 4) 4
12 4
3

666 6
6 6 1 13

4 4 1 12 4 1


3 of
5 5 6
5 5 6

SIMPLIFICATION USING IDENTITIES


1.

a x (b + c) = a x b + a x c
2

11.

2.

(a + b) = a + 2ab + b

3.

(a - b)2 = a2 - 2ab + b2

4.

(a + b)2 = (a - b)2 + 4ab

5.

(a - b)2 = (a + b)2 - 4ab

12.

13.

6.

(a - b) (a + b) = a2 - b2

7.

(a + b)2 = a3 + 3ab (a + b) + b3

8.
9.
10.

(a - b) = a - 3ab (a - b) - b
3

a ab b2
a3 b3
2

a ab b2
a2 ab b2
3

a b

a2 ab b2
3

a b

ab

ab

1
ab

1
ab

a + b = (a + b) (a - ab + b )
3

14.

a3 b3
2

15.

(a + b)2 + (a - b)2 = 2 (a2 + b2)

16.

(a + b)2 - (a - b)2 = 4ab

a - b = (a - b) (a + ab + b )

Bank Probationary
Quantitative Aptitude

FRACTIONS

CLASSIFICATION OF FRACTIONS
1 . Proper Fraction : A Proper fraction is one
whose numerator is less than its denominator.
1 4
,
3 9
2 . Improper Fraction : An improper fraction
is one whose numerator is equal to or greater
than its denominator
eg.

6 4
,
5 4
3 . Mixed Fraction: A mixed fraction is a
quantity consisting of two parts, one a whole
number and other a proper fraction.

dividing the numerator and the denominator by


the same number.

a ac

b bc
Reduction of a fraction to its lowest terms
To change a fraction to its lowest terms,
divide its numerator and denominator by the
H.C.F. of the numbers.
ie.

eg. Reduce

eg.

1
3
,9
8
4
A mixed fraction can always be expressed as
an improper fraction.
eg. 4

2
2 (5 x 3) 2 17
5

3
3
3
3
Similarly an improper fraction can always be
expressed as a mixed fraction. For that divide
the numerator by the denominator and write
the quotient as the whole number part of the
mixed fraction, the remainder as the numerator and the divisor as the denominator.
eg. 5

19
4 26
5
3 ;
3
5
5 7
7
Basic Property of Fractions
1 . The value of a fraction is not altered by
multiplying the numerator and denominator by
the same number.
eg.

2.

a axc
ac

ie.
b bxc
bc
The value of a fraction is not altered by

12
to its lowest terms.
36

12 12 12
1

36 36 12
3

(Since H.C.F. of 12 and 36 is 12)


Reducing fractions to their common
denominators
To reduce fractions to their common
denominators, change the denominators into
their L.C.M.
3 4
,
4 5
L.C.M. of 4 and 5 = 20

eg.

3
into 20,
4
multiply it by 5. To convert the denominator of

To convert the denominator of

4
into 20, multiply it by 4.
5
3 x5 4 x 4
15 16
;
(ie)
;
4 x5 5 x 4
20 20
Comparing Fractions
(ie)

a
b
and
, be two fractions with same
c
c
denominator c.
Let

Then

a b

c c

if a>b

eg.

4 3

5 5

a b

c c

if a<b

eg.

1 2

3 3

if a=b

1 1
eg.

2 2

a b

c c

6.

13

2 1

1
19
2
20
20

1
3
5
11
12 11 10
?
3
4
6
12

L.C.M. of 3, 4, 6 and 12 is 12

Addition and Subtraction of Fractions

(13 10 12 11)

Method : Convert the fractions with the same


denominator by taking L.C.M. and then add or
subtract.

Examples
3 2 3 2 5

7 7
7
7

2.

1 3 4
1
4 4 4

3.

2 4
?
7 9

L.C.M. of 7, 9 = 63

1 . To multiply a fraction by a whole number, multiply the numerator by the whole number.

eg. 2 x

2 3 4
?
3 4 5

L.C.M. of 3, 4 and 5 is 60

5.

eg.

1 . To divide a fraction by a whole number,


multiply the denominator of the fraction by the
whole number.
eg.

= 3

15 16
1
3
20
20

2
2
2
7

3
3 x7 21

2 . To divide a fraction by a fraction, find the


reciprocal of the divisor and then multiply.

3
4
3 ?
4
5

3
4
3 4
6 3 (6 3)
4
5
4 5

4 3
4x3 1
x

5 12 5 x12 5

Division of Fractions

2 3 4 40 45 48

3 4 5 60 60 60
40 45 48 133
13

2
60
60
60

3 (2 x 3) 6

5
5
5

2 . To multiply a fraction by another fraction


multiply corresponding numerators and denominators and then simplify.

2 4 18 28 46

7 9 63 63 63

4 11 9 10 4 1

12
12 3

Multiplication of fractions

1.

4.

1 11 3 5


3 12 4 6

eg.

2 4 2 5 10 5

3 5 3 4 12 6

Note: Cancellation can be performed only to


multiplication and division of fractions; it can
not be perfomed in addition or subtraction of
fractions.
Point to remember:
1.

To multiply a whole number and a mixed

fraction together, perform separate multiplication and then add the results.
eg. 1 8 x 5

2
2
(1 8 x 5 ) 1 8 x
3
3

90 12 120
2 . To divide a mixed fraction by a whole
number divide the whole number part of the
mixed fraction by the divisor (let the quotient
be a). Reduce the remainder to a single fraction and divide this single fraction by the divisor. (Let the quotient be b). Now the required
result is a+b.
2
eg. 21 4
3

3
1
of his salary on house rent and
of the
10
8
salary on clothes. He still has Rs. 1,400 left
with him. Find his total salary.

2 3 1
of his toTotally he spends
5 10 8
tal salary.

20
1

Now

2 5

3 3

5
5 1 5
4 x
3
3 4 12

2
5
5
21 4 5
5
3
12
12

7
x total salary = 1400
40

40
Rs. 8000
7
In an examination, a studnet was asked

total salary = 1400 x


3.

to find

3
of a certain number. By mistake,
14

3
of it. His answer was 150 more
4
than the correct answer. Find the given number.
Let the given number be x, then

he found

3x 3x

150
4 12

There are 40 students in a class. One day

15
3 3
x 150
x 150
4 14
28

7
th of total students were present.
10
Find the number of absentees on that day.

only

x =

Number of absentees

150 x 28
280
15

= Fraction of absentees x Total number


4.
=

2.

33
1

40 x total salary = 1400

More Solved Examples


1.

ary.

(ie)
2
4) 21 (5
3

He saves 1 5 10 8 part of his sal

1 x 40 12 students
10

By how much is

4
5
of 70 less than
of
5
7

112?

5
4
x 112 x 70 5 x16 4 x14 24
7
5

2
A man spends
of his salary on food,
5
5.

5
part of what amount will be equal to
12

3
part of Rs. 100.
4

Let the amount be Rs.y


5
3
of y 3 of 100
12
4

5
15
y
x 100
12
4

15 x100 12
y
x
4
5

y 900
Decimal Fractions
Fractions that have powers of 10 in the
denominators are called decimal fractions.
(ie) Fractions whose denominators are 10, 102,
103, 104 ......... are called decimal fractions.
eg. 0.5, 0.063, 8.98 etc.

Subtraction
In subtraction also, the given numbers are
to be written in such a way that the number of
decimal places become equal for all numbers.
eg. 5 - 0.473
Maximum number of decimal place
= 3 (in 0.473)
ie. 5-0.473=5.000 - 0.473 = 4.527
Multiplication
1 . Multiplication of a Decimal Fraction by a
power of 10:
Shift the decimal point to the right by as
many places of decimal as the power of 10.
eg. 4.5291 x 100 = 452.91
2 . Multiplication of two or more decimal
fractions :
0.002 x 0.08 x 0.5 = ?
Step 1: Multiply the given numbers as if they
are without any decimal point.
ie. 2x8x5 = 80

Here

5
63
898
; 0.063
; 8.98
0.5 =
10
1000
100
Annexing zeros to the extreme right of
decimal fraction does not change its value. 0.47
= 0.470 = 0.4700 etc.
Addition
For adding a decimal number with another
decimal number or with another whole number write the given number in such a way that
the number of decimal places are equal for all
the numbers.
eg. 2+0.63 + 0.712
Here maximum number of decimal place= 3

Convert all the numbers to 3 decimal places.

2+ 0.63 + 0.712 =
2.000 + 0.630 + 0.712 = 3.342

Step 2 : Add the total number of decimal


places in the given numbers
ie 3+2+1 = 6
Step 3 : Write the result of step 1 and convert it to a number whose number of decimal
places is same as the number obtained in step
2 by shifting the decimal point to the left.

0.002x0.08x0.5=0.000080 = 0.00008
Division
1 . While dividing a decimal fraction by powers of 10, the result is obtained by shifting the
decimal point to the left by as many places of
decimal as is the power of 10.
eg. 3.45 10 = 0.345
961.1 100 = 9.611
2 . While dividing a decimal fraction by a natural number, divide the given fraction without
the decimal point by the given natural number.
In the answer thus got, place the decimal point

to the left as many places of decimal as are


there in the dividend.
eg.

125
.
?
25

First step is

125
5
25

When a divisor as well as dividend is a


decimal, we multiply both the dividend and the
divisor by suitable multiple of 10 to make the
divisor a whole number and then proceed division.
SOLVED EXAMPLES
1.

2 37 x 2 37 + 36 3 x3 6 3 + 2 x 23 7 x 36 3

125
.
0.05
25

3 . While dividing a decimal fraction by a decimal fraction, shift the decimal point to the right
of the dividend and the divisor both by equal
number of digits such that the divisor is converted into a whole number.
eg.

Given expression is of the form


a 2 + b 2 + 2ab = (a+b) 2
= (237 363)2 (600 )2 360000
2.

a 2 - b2 =(a+b) (a-b)
= (221 + 220) (221 - 220)

28.6 28600

200
0.143
143

Put 1 in the denominator under the decimal point and annex with it as many zeros as is
the number of digits after the decimal point.
Remove the decimal point and reduce the fraction to its lowest terms.
Thus

eg.

= 441 x 1 = 21
3.

a3 b3

To divide a decimal by any multiple of ten


move the decimal point as many places to the
left as is the number of zeros in the divisor.

a b 0.45 0.21 0.24

a2 ab b2
4.

4.7 x 6.5 5.3 x 6.5


?
13
. x 7.9 13
. x 6.9
ax bx (a b) x
Given expression is cy dy (c d) y

8.86
886

9.25
925

To multiply a decimal by any multiple of


ten, move the decimal point as many places to
the right as is the number of zeros in the multiplier.

0.45 x 0.45 x0.45 0.21 x 0.21 x 0.21


0.45 x 0.45 0.45 x 0.21 0.21 x 0.21

Given expression is of the form

0.125 125
1

1000
.
1000
8

If numerator and denominator of a fraction contain the same number of decimal


places, then we may remove the decimal sign.

2212 220 2 ?

Given expression is

3.15
315
.

0.9
3.5
35

Expressing a decimal into a vulgar fraction

Evaluate

(4.7 5.3) 6.5 10 x 6.5

50
(7.9 6.9)13
.
1x13
.

5.

0.75 x 0.75 0.74 x 0.74


?
149
.

Given expression is

a2 b2
a b
ab

= 0.75 - 0.74 = 0.01


6.4 x 6.4 2 x 6.4 x 3.6 3.6 x3.6

6.

(6.4) (3.6)

4
1

a2 b2

=
=

(a b)
a b (6.4 3.6)

(a b)(a b) a b (6.4 3.6)


= 4

=
7.

10 100 25
4

3
2.8 28
7
7

4
9

1
2
4

a2 2 ab b2

The given expression is

= 4

9 =
31

31 32 31 1

8
8
8

1 0 . Find the value of

0.7 x 0.7 x 0.7 - 0.3 x 0.3 x 0.3


- 3x0.7x0.3x0.4 = ?

1
1

1 1 1
2 2

The given expression is

1
1
3

1
1
1 1

3
4

a3 - b3 - 3ab (a-b)
1
1
1
1
1
1

1 1 1 1 1 1

2
2
3
3
4
4

= (a-b) 3 =(0.7-0.3) 3 =(0.4) 3


= 0.064
8.

Simplify
7

9.

1
1
1

1 1 1

4
9
16

3 8 15 5
x x

4 9 16 8

1
1 1
1 1
1
1
2
1

1


2
4
4
2
2
3
6

1
1 1
1 1
1
1
2
1

1


2
4
4
2
2
3
6

1 1 1 1

7
2
1 x 1
2 4 4 2

1 1 3
7 2
2 4 4

1 9 4
1
1
7 x 7 3 4
2 4 3
2
2
5

Find the value of 4

1 1 . Find the value of


2

2 2

1
2

1
2

1
1
2
4

2 2
1
2 2

2 2 2 2

2 2

2 2

2 2
2

2 4

2 2

x 3
1 2 . If y 4 then find the value of

6.

6 yx

7 yx

7.

x
1
6 y x 6
y


x
7 y x 7 1
y

2 of

3 3 1
?
4 4 4

1)

4
9

9.

10.

1.
2.

1)
4) 1 9

17

2) 1 3

3) 1 9

12.

4) 1 5

31

2) 1 5

3) 2 4

4) 4 2

1
3

2)

4
5

3)

1
2

4)

1
6

14.

(7 7 7 ) 7
?
3333
1)

3
11

2)

3
13

5
7

3) 5 4

4) 6 7 0

900

2) 3 0

3) 5 0 0

4) 1 2 2 5

75

2) 1 0. 3

3) 1 4. 5 4) 7 . 5

188
.
0.3

2)

1 88
30

3)

47
75

4)

16
25

4)

3
7

2) 1

3) 1 0

4) 1 6

(0.356)2 x 2 x 0.356 x 0.106 (0.106 )2


?
(0.632 )2 2 x 0.632 x 0.368 (0.368 )2

1)

0. 62 5

2) 0. 06 25

3)

0. 03 45

4) 0. 34 5

1)

1. 13 2

2) 0. 24 2

3)

1. 42 2

4) 1

4.75x 4.75x 4.75 125


. x125
. x125
.
?
4.75x 4.75 125
. x125
. 4.75x125
.
1)

3)

2) 2 7

. x0637
.
2x0637
. x0395
.
0395
. x0395
.
1 3 . 0637
?
0242
.

1 1 1
1
x
2 3 4 12
1)

5.

3) 1 7

1
of 35+4 (9-3) = ?
5
1)

4.

2) 1 5

6+[2+{4x(8-3) - (2x6)-1}+2]= ?
1)

3.

20

540

11. 0. 7x0.7x0. 7+0.3x0.3x0. 3+3x0.7x0.3=?

20 - [9-{7+(2x3)} +5] = ?
1)

1
4

24.4 x 24.4 2 x 24.4 x 5.6 5.6 x 5.6


?
24.4 x 24.4 5.6 x 5.6

1)

PRACTICE TEST

4) 2

8.9 x 8.9 x 8.9 1.4 x 1.4 x 1.4


?
8.9 x 8.9 8.9 x 1.4 1.4 x 1.4
1)

1
6
6 1
4

1
7
7
7 7
4

3) 2

32.5x32.5-2x32.5x2.5+2.5x2.5=
1)

3
1
6
4

7 1 3
4

3
2

2 3.5 x 2 3.5 3.5 x 3.5


?
9.6 x 9.6 2 x 9.6 x 8.6 8.6 x 8.6

1)
8.

2)

15.

5.25

2) 3 . 5

3) 0

4) 6

7 7 5 x7 7 5 2 2 5 x 2 2 5 7 7 5 x 2 2 5
?
7 7 5 x 7 7 5 x 7 7 5 + 2 2 5 x 2 2 5 x 2 2 5

1)
3)

1000
0. 00 1

2) 0 . 0 1
4) 0.0001

(0 .3 3 7 0.12 6 )2 (0.3 3 7 0 .1 26 )2
?
16.
0 .3 3 7 x 0.1 2 6

1)
3)

0. 21 1
4

25.

2) 0. 46 3
4) 2 . 1 1

(6 9 5 3 4 5 )2 (6 9 5 3 4 5 )2
?
17.
(6 9 5 )2 (3 4 5 )2

1)2

2) 3 4 5

3) 6 9 5

19.

(4.621 2.954 ) (4.621 2.954 )


?
4.621x 4.621 2.954 x 2.954
1) 4
2) 2
3) 0
4) 1

22.

27.

0.52 x 0.52 0.4 x 0.4 2 x 0.52 x 0.4


?
0.52 0.4
1.2
0.48

2) 0 . 9 2
4) 0 . 1 2

4.5

23.

1)

3) 5 . 1

28.

2)

27
32

3)

27
64

4)

107
112

3)

1
5

1 1
1
of
5 5
5 ?
1
1 1
of
5
5 5

2) 5

2)

5
3

3) 1

4)

3
5

4) 2 5

7 9 2 4 x 1 1 6 ?
5 x 9 (1 3 1 2 )

4) 2 . 2 0

1
2

1
2

27
16

1)

3)

5
4

2) 0 . 4 5

4) 21

1)

1) 1

(4.8 )3 0 .0 2 7
?
(4.8 )2 1.4 4 0 .0 9

1)

1
2

7
12

1 1 3
1 7
3
2 6 . 2 2 4 2 8 4 ?

6 4 8 5 2 x 6 4 8 5 2 2 4 8 5 2 x2 4 8 5 2
?
64852 24852

1)
3)

2) 2

3) 15

1) 2 0 0 0 0
2) 80000
3) 3 0 0 0 0
4) 40000
20.
126.5x126.5-2x126.5x6.5+6.5x6.5=?
1) 12000
2) 1 4 4 0 0
3) 17689
4) 1 4 4 0
21.

1
6

1)

4) 4

18.

1 4
5
of
4 5
6
?
1 1 3
1
4
21
3 5 1 0
5
3

2 9 . If

7
20

2)

55
4

4)

1
20

14 2 b a
a 7

, then
is equal
23 2 b a
b 8

to
3 1
1 1
2 4 . 5 4 2 2 0 .5 6 7 ?

1)

19
84

2) 2

61
23
47
3) 2
4) 2
84
84
84

1)

5
14

2)

5
9

3)

5
23

4)

5
92

1
3 0 . 2
3

3 5 7
997
2 2 2 ..... 2
?
5 7 9
999

1)

5
999

7
2)
1000

3)

1000
7

1001
4)
3

3 1 . If

-1

2) 3

3) 4

4) 5

1 1 1 1
1 1
1 , then the
x 2 3 4
2 3
value of x is

3 2 . If

1
4

1)

13
22

2)

33.

1)

1
2
2

19
2)
8
1

1)

5
9

7
3)
8

8
4)
9

1
9
2) 1

2) 2 0 11 1 0

3)

2 1 11 1 0

4) 1 09 9 99 9

1)

70

2) 7 5

3) 7 2

4) 8 2

3 9 . The sum of squares of two numbers is


80 and the square of their difference is
36. The product of the two numbers is
1)

22

2) 4 4

3) 5 8

4) 1 1 6

4 0 . The product of two numbers is 120. The


sum of their squares is 289. The sum of
the two numbers is
1)

20

2) 2 3

3) 1 6 9

1)

31 , 2 0

2) 30 , 1 9

3)

29 , 1 8

4) 28 , 1 7

4) 1 5 0

4 2 . The sum of two numbers is 10 and their


product is 20. The sum of their reciprocals is

1
4

1)

1 9 99 9 9

4 1 . The difference of two numbers is 11 and


one-fifth of their sum is 9. The numbers
are:

8
19

34. 1

4) 4

3) 2

1)

3 8 . The sum of two numbers is 22 and their


difference is 14. Find the product of the
numbers.

a 4
6a 4 b
is
, then the value of
b 3
6a 5b

1)

3 7 . The sum of the smallest six digit number


and the greatest five digit number is

1
2

2)

3) 1

4) 1

1
4

4 3 . If the sum of two numbers exceeds their


difference by 10, then the smaller of the
two numbers is

10
19

3)

10
19

4)

19
10

1)

2) 5

3) 8

4) 1 3

4 4 . The simplification of
3 5 . If (a-2) is 6 more than (c+4) and (a+2)
is 3 less than (c-4), then (a-3) is
1) 0 . 5
2) 1 . 0
3) 1 . 5 4) 2 . 0
3 6 . The expression
(7.98 x 7.98+7.98x x + 0.02 x 0.02) will
be a perfect square for x equal to
1)

4.0

2) 0 . 4

3)

0.04

4) 0. 00 4

1
3 9 of 9 yields
1
25
5
5
1
5
5
3

1) 0

2)

3)

81
74
4) 2
125
125

4 7 . The simplification of

5625
4 5 . Given that 83.9 x 93.4 0.7178 then

1 1

3
1 2 2 3
1 5
2 of
x3
4 3 1 1
3 6 yields
4

2 3

5.625
8.39 x 9.34 is equal to
1)
3)

71 7. 8
0. 071 78

2) 71 . 7 8
4) 0. 71 78

1)

7
18

2)

49
54

3)

2
3

4)

1
6

4 6 . The simplification of

1
1
1 1

1
1

4
1 yields
7

1)

1
3

2) 1

3)

1
7

4) 1

4 8 . The simplification of

1 1 1 1 1 yields
3

1
4

1)

1
3

2) 1

4
7

3)

1
8

4) 1

2
3

ANSWERS TO PRACTICE TEST

1.

(4)

2.

(1)

9.

(4)

10. (3)

11. (2)

12. (2)

13. (2)

14. (4)

15. (3)

16. (3)

17. (1)

18. (2)

19. (4)

20. (2)

21.(4)

22. (1)

23. (4)

24. (3)

25. (3)

26. (2)

27. (4)

28. (2)

29.(3)

30. (4)

31. (3)

32.(2)

33. (1)

34. (2)

35. (3)

36

(3)

37. (1)

38. (3)

39. (1)

40.(2)

41. (4)

42

43

44. (1)

45. (3)

46. (4)

47. (3)

48.(2)

(2)

3.

(1)

(2)

4.

(3)

5.

(4)

6.

(4)

7.

(1)

8.

(1)

BANK PROBATIONARY OFFICER


QUANTITATIVE APTITUDE
SQ UA RE & S QU AR E RO OT S
Square : If a number is multiplied by itself then
the product is the square of the number. Thus
the square of 5 is 5x5 = 25

2.

219961 = 469
Find the square root of 59.1361

7.69

1 1 1
1
eg. x
2
2 2 4

59 . 13 61
49

2 2 4
2
x
3
3 3 9

146

1013
876

Square root: The square root of a number is


one of two equal factors which is multiplied
together gives that number.

1529

1 37 6 1
0

eg. 121 = 11x11 = 11

59.1361 = 7.69

10000 = 100 x 100 = 100

Properties of a perfect square

Finding Square root by means of factorisation


When a given number is a perfect square,
we resolve it into prime factors and take the
product of prime factors, choosing one out of
each pair.
eg. Find the square root of 1156

No perfect square ends with 2,3,7,8

No perfect square ends with an odd


number of zeros.

The perfect square consisting of (n-1)


or n digits will have n 2 digits in their
root

Factors of 1156 is 2x2x17x17


1 1 5 6 = 2 x 2 x 1 7 x 1 7 = 2 2x 1 7 2
1156 = 2 2 x 17 2 = 2x17 = 34
General method to find the square root
In the given number mark off the digits
in pairs, from right and then find the square
root as shown in the example below.
eg. 1.
4
86
929

Find the square root of 219961


469
21, 99, 61
16
599
516
8361
8361
0

1 37 6 1

The square of a number other than unity


is either a multiple of 4 or exceeds a
multiple of 4 by 1.

CUBE ROOT
The cube root of a number is one of three
equal factors which if multiplied gives that
number. Cube root of a number can be found
out by using the following steps.
1 . Write down all the prime factors of the
given numbers.
2 . Write the prime factors in the index notation, ie, in a n form.
3 . Divide the index by 3, then the result will
be the cube root of the given number.

eg. 1. Find the cube root of 512


512=(2x2x2) x (2x2x2) x (2x2x2)

3.

9 13
3
512 = (2 ) 2 8
2. Find the cube root of 0.000027

3.

27
0.000027

1000000

13


33

13

3 13

100

3
0.03
100
Learn by heart the following square roots

4.

If

x
4
, the value of x is
49 7

1)

If x =

Square

1
2
3
4
5
6
7
8
9
10
11
12
13
14
15

Cube

1
4
9
16
25
36
49
64
81
100
121
144
169
196
225

1
8
27
64
125
216
343
512
729
1000
1331
1728
2197
2744
3375

No
16
17
18
19
20
21
22
23
24
25
26
27
28
29
30

1)

2= 1. 41421

6 =2. 44949

3= 1. 73205

7 = 2. 64575

5=2. 2360

10 = 3.1622

1 . If x = 3018 + 36+169, the value


of x is

2.

44

If x =
1)

2) 5 5

3) 6 9

4) 4 3

169
. x 0.9
, the value of x is
13
. x0.13

2) 1 3

3) 3 9

1)

6.

9.

2) 1 0

5
8

3) 8

4) 1 5

3 24
x 0. 4 ?
9
2)

8
5

3)

9
5

4)

10
5

169
169
. , the value of x is
x

If
1)

100

2) 1 0 0 0

3)

10000

4) 1, 00, 000

144
324
104

x
?
6
6
169

7.

8.

12

1)

26

2) 1 4

3)

10 1. 82

4) 3 6

? + 44 = 25% of 400

1)

If x =
1)

2) 3 6

3) 4 9

4) 1 6

81
, the value of x is
0.09

2) 3 0

3) 3 0 0

4) 0 . 3

1 0 . The largest of four digit numbers which


is a perfect squre is

PRACTICE TEST

1)

4) 4

196
18
65

the value
7
324
169

484
x
11

5.

Square
256
289
324
361
400
441
484
529
576
625
676
729
784
841
900

3) 1 6

of x is

SQUARES AND CUBES


No.

2) 4 9

4) 0 . 3 9

1)

9801

2) 9 9 0 4

3)

9804

4) 9 8 0 9

1 1 . A certain number of people collected Rs.


125. If each person contributed as many five
paise as they are in number, the number of
person were
1)

25

2) 5 0

3) 1 0 0

4) 1 2 5

1 2 . A gardener plants an orchard with 5776


trees. In each row there were as many trees
as the number of rows. Find the number of
rows.
1) 7 6
2) 9 6
3) 6 6
4) 1 8 6
1 3 . Each student in a class contributed as
many rupees as the number of studnets in the
class for a picnic. The school contributed Rs.
150 per teacher who led the trip. If the total
amount collected was Rs. 1350 and the number of teachers who led the trip was 3, how
many students were there in that class?
1)

36

2) 3 5

3) 3 4

9
55

1)

2)

3) 4

7 5
1)
3)

1
45
4)
11
11

1 8 . If

0.04 x 0.4 xa = 0.004 x 0.4 x

1)
3)

b,

a
is
b

then

6 2.4 5 , the

3) 1 . 7 3 4) 1 . 7 5

1 5 6 2 5 1 2 5 then the value of

1 5 6 2 5 1 5 6 .2 5 15
. 6 2 5 is

1)
3)

1. 38 75
13 8. 75

2 3 . If

2) 13 . 8 75
4) 15 6. 25

4 0 9 6 = 64 to then the value of

2 2

2) 7. 10 14
4) 7 . 1 2
1
equals
2 2

1)

2) 2 2

3)

2 2

4) 2 2

1)

12 3

2) 9 3

3)

3 3

4)

2 6 . If

5 2.24 then the value of

2 5 0.48

16x10

2) 16x10

16x10

-5

4) 16x10 - 6

1)
3)
1

0. 16 8
1 6. 8

4) 3
2 3

2) 1 . 6 8
4) 16 8. 0

2 7 . Suppose you know that 15 is approximately 3.88 which of the following is the best

2) 0

is

-3

1 9 . The value of 2 1 3 2 2 3 is

3) 1

4) 5 1

3 5
-4

(1) -1

3) 3 1

2 5 . 3 15 . 5 4 27 is equal to
2) 6 35
4) 1

2 2 . If

2
5

is
3
6
1 . 3 7 2) 1 . 5 7

is equal to

6 35

1)

5 2.2 4 and

1) 7 . 0 9
3) 7. 11 04
2 4 . The expression

7 5
17.

value of

2) 2 1

1 6 8 1 is

is

81
?
121

11
45

2 1 . if

41

400

40.96 0.4096 0.004096 0.00004096

x
13

then x is equal to.


144 12
0
2) 1 2
3) 1 3
4) 2 5

1 5 . If 1

169
4
1 6 . 225 9

1)

4) 3 0

1 4 . Some persons contributed Rs. 1089. Each


person gave as many rupees as they are in
number. Find their number.
1) 3 3
2) 6 6
3) 4 5
4) 2 3

1)

2 0 . The value of

approximation to

5
?
3

1)

0.43

2) 1 . 8 9

3)

1.29

4) 1 . 6 3

13.69 0.6025 x 37.25 then x

2 8 . If

is equal to
1)

10

2) 1 0 0

441
x
3

225
x
5

1)

21

2) 2 8

4 x

29.

3) 1 0 0 0 4) 1 1 2 5
64
256

3) 4 8

one place of decimal is


1)

1 8. 6

2) 4 . 6 3

3)

4.6

4) 4. 63 6

3 3 . If

4) 8 4

43
upto
2

of decimal is 9.3 then the value of

1 8 2 2 5 1 3 5 , then the value of

18225 182.25 18225


.
0.018225 is
30.

24 81

27
3

20

1)

1
2) 8
4

3)

2
8
3

3
4) 8
4

equals

1)

1. 499 85

2) 14 . 99 85

3)

14 9. 9 85

4) 14 99. 85

3 4 . If

6 2 .4 5

then

the

value

of

3 2 3

3 1 . If

6 2.45 then the value of

3 2
3 2

correct upto three places of decimals is


1)

1. 00 0

2) 0. 99 0

3)

0. 10 0

4) 0. 10 1

3 2 . If

equals

3 2 3
1)

0.41

2) 0 . 4 2

3)

4.10

4) 4 . 2 0

3 5 . The square root of

0 .3 2 4 x 0 .0 8 1 x 4.62 4
15
. 6 2 5 x 0.0 28 9 x 72 .9 x 6 4 is

8 6 when approximated to one place

1)

2 4. 0

2) 2 . 4

3)

0.24

4) 0. 02 4

ANSWERS TO PRACTICE TEST

1.

(2)

2.

(1)

9.

(2)

1 0 . (1)

1 7 . (1)

3.

(3)

4.

(3)

5 . (2)

6.

(3)

7.

(1)

8.

(3)

1 1 . (2)

1 2 . (1)

13. (4)

1 4 . (1)

1 5 . (4)

16.(2)

1 8 . (3)

1 9 . (3)

2 0 . (2)

21.(3)

2 2 . (3)

2 3 . (3)

2 4 . (1)

2 5 . (3)

2 6 . (2)

2 7 . (3)

2 8 . (4)

29.(4)

3 0 . (3)

3 1 . (3)

32.(3)

3 3 . (3)

3 4 . (2)

3 5 . (4)

Bank Probationery Officer

LAWS OF EXPONENTS

Quantitative Aptitude

A long product axaxa ....m factors can


be expressed, in short by notation a m, where
`a' is called the base and `m' the power (or
index or exponent) Thus axaxa.... 10 factors
= a 10 .

1.

2.

Negative Index : If a 0 then a

81 3 x 161 4 x 2 2 ?
4 14

1
m
a

3 2

2 x 2
2

3.

2 2

3 x 3
2

3 4

3
23

a, a

4.

a
3

If `a' and `b' are non-zero integers, `m' and


`n' are rational numbers, then

a xa a

ii).

am
am n
an

26 ?

2 7

34x 3
312

1
312
312

Evaluate
3 3 2 (2 7 )2

= 1

m n

i).

x 2 2

34 x 38

am

26 x 26
26 64
26

1
3 )
a

Fractional Index : a1 n

2 1 x 2 1 x 2 2 2 112 2 1

ie. a = 1 where a 0 eg. 5 = 1.

am . bm

3 13

Meaning of a : Any non-zero number raised


to zero power is equal to 1.

abm

2 x 2

(eg. (a2 )3 a6

eg. a

iv).

m
vi). a

Property 3 : (am )n amn

12

amn

SOLVED EXAMPLES :

(eg. a 3xa 4 = a 3+4 = a 7 )


am
Property 2 : a n=a m-n
5
(eg. a = a 5-3 = a 2 )
a3

(eg. a3

v). a 1

Definition am: If a is a natural number, then am


stands for the product of `m' factors each
equal to `a'.
Property 1: a m x a n = a m+n

m n

iii).

1
32

2 3

9
3
26

25
13

1
5
32 2 2
9
3

1 2

3 2

5

1
2

1
5
7
7
9 4 14 1 15
9
3
9
9

If 5 a 3 1 2 5 , find the value of 5 a 3

5.

8.
5a 3125 5a 55

a 5
5
6.

a3

1
3

1)

5 25

9.

If m and n are whole numbers such that

m 121 , find the value of (m 1)n1

3.

4.

(6 4 )

1
8

2 n 64 , the value of n is

1)

2)

3
16

3)

2) 4

4) 2

, x is equal to

1
2

3)

3) 6

21x92

2) 2 2 x 9 3

3)

2 3 x9 4

4) Zero

4x

(1 2 5 )0 .25 is equal to
5

2) 2 5

3) 5

1)

10

2) 1 5

3) -2

x 3 4 1

1)

If 2

2 x 1

1)

721

x 2 2 5

2) 3

1
8 x 3

4) -1

4)

data inadequate

1 2 . (4 )0 .5 x (0 .5 )4 is equal to
1

2) 4

3)

1
8

The value of

3) -2

x9
35n

1
32

is
1)

2) 8

3) 9

4) 1 0
2
3

gives

4) 7

, the value of x is

1 2 n

4)

1 3 . If 2 x 1 2 x 1 1280 , then the value of x

1
1 4 . The simplification of

1 25

is

3) 5

2) -1

4) 5 5

3) 2 5

The unit's digit in the product of


72

4) 2

4) 1 2

If 1.125x10 =0.001125, the value of k


is
2) -3

7
2

1)

1)

-4

x 3

1 1 . If a3 b ab3 c 180 and a, b, and c are


positive integers, the value of c is

equals

If

2 4 6 7

7.

(3 2 )

4) 2

1
16

153

6.

3) 1

1)

1)
5.

2) 2 2

2
3

between the obtained and the actual value is

1)

25 25 is divided by 24, the remainder is

4)

A boy was asked to write the value of

10. 5

PRACTICE TEST

2.

2)

1
9

2 x 9 2 . He wrote it as 2592. The difference

(m 1)n 1 (1 1 1)2 1 1 0 3 1 0 0 0 0

23

b

a

3)

mn 121 112 m 11, n 2

1)

9
13

1.

x 1

If
b
1)

5 3

2)

4) 2

1)

1
25

2)

3)

25

4) -25

1
25

1 5 . The units digit in 2 4 6 9 1 5 3 will be

2 n 7

is

1)

2) 3

3) 7

4) 9

2 4 3 0 .1 2 x 2 4 3 0 .0 8

16.

1)

2) 9

equals

2 1 . The value of

3) 1 2

4) 2 7

1 7 . The value of 4 7 1 6 4 x 1 6 is euqal to:


1)

1
16

x
1 8 . If 4

1)

2)

1
4

3) 4

2) 9

132
15

2)

15
132

3)

15
2

4)

15
8

4) 1

4) -9

1 9 . The value of 7 2 9 x 7 2 9
3

2) 9

1
12

3) 1 2

x 2 y 2
2 0 . The fraction 1
1
x y

2 2 . If

1)

26

3) -3
1
4

1)

1)

1
, the value of x is
64

is

x x4

10

1)

is equal to

2) 2

2) 5

2 n 4 2 (2 n )

x 1 y 1

2) x 1 y 1

3)

xy
xy

xy
3)
xy

xp , p is equal to
3) 1

4) 0

2 3 . If 5 5 x 5 3 5 3 2 5 a2 the value of
`a' is

4) 2 7

1)

(1)1 3 2
equals
5 3 1
1

24.

2 2 n3

3) 6

4) 8

2 3 is equal to
n 1

2) 2

1)

2 n1

3)

9
2n
8

1
8

4) 1

ANSWERS TO PRACTICE TEST


1.

(3)

2.

(1)

3.

(4)

4.

(2)

5.

(3)

9.

(4)

10. (3)

11. (4)

12. (3)

13. (3)

17. (4)

18. (3)

19. (2)

20. (3)

21.(4)

6.

(4)

7.

(3)

8. (4)

14. (3)

15. (4)

16.(1)

22. (4)

23. (1)

24.(4)

Bank probationary Officer


Quantitative Aptitutde

L .C .M & H. C. F

Factors and Multiples: If a number `m' divides


another number `n' exactly, then we say that
`m' is a factor of `n' and that `n' is a multiple
of `m'.
eg. 3 is a factor of 12 and therefore 12
is a multiple of 3.
Least Common Multiple (L.C.M.)
L.C.M. is the least non-zero number in common multiples of two or more numbers.
Multiple of 6 = 6, 12, 18, 24, 30, ........
Multiple of 8 = 8, 16, 24, 32, 40, ........
Common Multiple of 6 and 8 = 24, 48 ...............
Least Common Multiple = 24
Factorisation Method:
Find the L.C.M. of 12, 27 and 40
Factors of 12
= 2x2x3 = 2 2 x3
2 12 3 27 2 40
Factors of 27
2 6 3 9
2 20
= 3x3x3 = 3 3
3 3 3 3
2 10
Factors of 40
3
1
1
5 5
= 2x2x2x5 = 2 x5
1
L. C. M . 2 3 x 3 3 x 5 1 0 8 0
SHORT CUT METHOD
(Division Method)
Find the L.C.M. of 12, 27, 40
2 12,
2 6,
3 3,
1,

27, 4 0
27, 2 0
27, 1 0
9,1 0

L. C. M. 2x2x3x9x10 = 1080

HIGHEST COMMON FACTOR (H.C.F)


The highest common factor of two or
more numbers is the greatest number which
divides each of them exactly.
eg. Find the H.C.F. of 24 and 56
Factors of 24 = 1, 2, 3, 4, 6, 8, 12, 24
Factors of 56 = 1, 2, 4, 7, 8, 14, 28, 56
Common factors of 24 and 56 are 1, 2, 4, 8
H.C.F. of 24 and 56 = 8
Factorisation Method: H.C.F. can be found by
resolving the given numbers into prime factors and then taking the product of least powers of all common factors, that occur in these
numbers.
Eg. Find H.C.F. of 48, 108, 140
2 4 8 2 108
Factors of 48
2 24 2 54
= 2x2x2x2x3
2 12 3 27
= 2 4 x3
2 6 3
9
Factors of 108
3
3
= 2x2x3x3x3 = 2 2 x3 3
Factors of 140
= 2x2x5x7 = 2 2 x 5 x 7
H.C.F. = 2 2 = 4
Division Method
Find the H.C.F. of 48, 108, 140
2
4 8 , 1 08 , 1 4 0
2
24, 54,
70
12, 27,
35
H.C.F.=2x2= 4

2 140
2 70
5 35
7

QUICKER & SHORT CUT METHOD


Find the H.C.F. of 777 and 1147
777) 1 1 4 7 (1
777
370) 777(2
740
37) 370 (10
370
0
H.C.F. of 777 and 1147 is 37
*
The product of two given numbers is equal
to the product of their H.C.F. and L.C.M.
L.C.M. of two numbers
Product of numbers
= H. C. F. of numbers

L.C.M. of given fractions


L. C. M. of numerators
= H. C. F. of deno min ators

H.C.F of given fractions


H. C. F. of numerators
= L. C. M. of deno min at ors

The L.C.M of a given set of numbers


would be either the highest or higher than the
highest of the given numbers.

The H.C.F. of a given set of numbers


would be either the lowest or lower than the
lowest.
Solved Examples
1 . Find the L.C.M. of 125,64,8 and 3.
Ans :
Given numbers are 5 3, 26, 23 and 3
L.C.M. 5 3x2 6x3 = 24,000
2 . Find the L.C.M. of

1 5 5 10
, , ,
?
3 6 9 27

Ans: L.C.M. of fractions


L. C. M. of numerat ors
= H. C. F. of deno min ators

L.C.M. of 1, 5 and 10 is 10
H.C.F of 3, 6, 9 and 27 is 3
L.C.M. of given fractions =

3 . Find the H.C.F. of


Ans:
=

10
3

1 3 5 7 9
, , , ,
2 4 6 8 10

H.C.F. of fractions
H. C. F. of numerat ors
L. C. M. of deno min ators

H.C.F. of 1, 3, 5, 7 and 9 is 1
L.C.M of 2, 4, 6, 8 and 10 is 120
H.C.F. of given fractions =

1
120

4 . The L.C.M. of two number is 2310. Their


H.C.F. is 30. If one number is 210, the other
is:
Ans: The other number
=

L. C. M. xH. C. F.
2310 x 30
330
given number =
210

5 . The H.C.F. and L.C.M. of two numbers


are 44 and 264 respectively. If the first number is divided by 2, the quotient is 44, The
other number is
Ans: First number = 2x44 = 88
Second number =

44 x264
132
88

6 . The least square number which is divisible by 6, 8 and 15 is:


Ans: The least number divisible by 6, 8 and 15
is their L.C.M. which is 120
Now 120 = 2x2x2x3x5
To make it a perfect square, it must be
multiplied by 2x3x5
Required Number=120x2x3x5=3600
7 . The least number of square tiles
required to pave the ceiling of a room 15m
17cm long and 9m. 2cm broad is:
Ans: Size of largest square tile

= H.C.F. of 1517 cm and 902 cm


= 41 cm.
Least number of tiles required

3.

2 3
4
,
and
5 10
15

1)

1
30

2) 2

1517 x 902
814
=
41 x 41

3)

24
750

4)

2.

Find the L.C.M. of

Areaof the room


=
Areaof one tile

8. Find the least number which when divided


separately by 15, 20, 36 and 48 leaves 3 as remainder in each case.
Ans : Required number
= L.C.M. of (15,20,36 and 48) +3
= 720 + 3 = 723
9 . Find the greatest number that will divide
197 and 269 and leaves 5 as remainder in
each case.
Required number = H.C.F. of [(197-5)
and (269-5)]
= H.C.F. of (192 and 264) = 8
1 2 . Five bells begin to toll together and toll
respectively at intervals of 6,7,8,9 and 12 seconds. How many times they will toll together
in one hour, excluding the one at the start?
Ans: L.C.M. of 6,7,8,9 and 12
= 2x2x3x7x2x3 = 504
ie, The bells will toll together after each 504
seconds. In one hour, they will toll together

1.

4.

60 x60
7 times
504

PRACTICE TEST
Find the L.C.M of 12, 15, 18 and 27.
1) 1, 08 0
2) 5 4 0
3) 2 7 0
4) 7 6 0
Find the H.C.F. of 72, 48 and 30.
1) 3 0
2) 1 2
3) 6
4) 3
Find the L.C.M. of 2 2x33x53 and
2 3 x3 2 x5.
1) 27 , 0 00
2) 1 8 0
3) 3 6
4) 13 , 5 00

5.

6.
is.

Find the H.C.F. of

2
5

2
5

4 3
7
,
and
5 10
15

1)

1
5

2)

84
5

3)

84
30

4)

1
30

If the L.C.M of x and y is z, their H.C.F.

1)

xy
z

2) xy z

3)

x+ y
z

4)

z
xy

7 . H.C.F of two numbers is 24 and their


L.C.M is 1080. If one of the numbers is 120,
find the other.
1) 2 1 6 2) 5 3 2 3) 1 0 8 4) 8 2 0
8 . L.C.M. of 2.5, 0.5 and 0.175 = ?
1) 2 . 5
2) 0 . 5
3) 0. 17 5
4) 1 7. 5
9 . H.C.F. of two numbers is 24 and their
L.C.M is 1344. If the difference between the
numbers is 80, their sum is:
1) 3 6 8
2) 3 5 6
3) 3 3 2
3) 3 0 4
1 0 . Find the greatest number which can divide 1354, 1866 and 2762 leaving the same
remainder 10 in each case.
1) 6 4
2) 1 2 4 3) 1 5 6 4) 2 6 0
1 1 . Find the least perfect square which is di-

visible by 3, 4, 5, 6 and 8.
1) 2 5 0 0
2) 1 2 0 0
3) 3 6 0 0
4) 9 0 0
1 2 . The least number which when divided by
15, 27, 35 and 42 leaves in each case a remainder 7 is:
1) 1 8 9 7
2) 1 9 8 7
3) 1 8 8 3
4) 2 0 0 7
1 3 . Two containers contain 60 and 165 litres
of milk respectively. Find the maximum capacity of a container which can measure the milk
in each container an exact number of times
(in litres)
1) 1 5
2) 3
3) 5
4) 1 0
1 4 . Two baskets contain 195 and 250 bananas respectively, which are distributed in
equal number among children. Find the largest
number of bananas that can be given, so that
3 bananas are left over from the first basket
and 2 from the second.
1) 4
2) 1 8
3) 8
4) 6
Qn: (15- 18) :- Write in ascending order
15.

16.

17.

1 2 3 3
, , ,
2 5 4 2

1)

7 1 3 5
, , ,
8 3 4 6

2)

5 7 3 1
, , ,
6 8 4 3

3)

3 7 1 3
, , ,
4 8 3 4

4)

1 3 5 7
, , ,
3 4 6 8

Qn: 18- 20 Write in descending order


18.

19.

20.

1 2 3 1
, , ,
3 5 4 6
1)

1 2 3 1
, , ,
3 5 4 6

2)

1 2 1 3
, , ,
6 5 3 4

3)

2 3 1 1
, , ,
5 4 3 6

4)

3 2 1 1
, , ,
4 5 3 6

5 7 11 3
, ,
,
6 8 12 10

1)

5 7 11 3
, ,
,
6 8 12 10

2)

7 5 11 3
, ,
,
8 6 12 10

3)

11 7 5 3
, , ,
12 8 6 10

4)

7 5 11 3
, ,
,
8 6 12 10

5 11 5 7
, , ,
3 9 6 12

1)

2 1 3 3
, , ,
5 2 4 2

2)

3 1 2 3
, , ,
4 2 5 2

1)

5 11 5 7
, , ,
3 9 6 12

2)

11 5 7 5
, ,
,
9 3 12 6

3)

3 3 1 2
, , ,
2 4 2 5

4)

3 1 2 3
, , ,
2 2 5 4

3)

5 11 5 7
, , ,
3 9 6 12

4)

11 5 5 7
, , ,
9 6 3 12

5 11 5 7
, , ,
3 9 6 12

Qn 21-23 Find the greatest of the given fractions

1)

11 7 5 5
,
, ,
9 12 3 6

2)

7 5 11 5
, , ,
12 6 9 3

3)

5 7 11 5
,
, ,
6 12 9 3

4)

5 11 5 7
, , ,
3 9 6 12

5 7 3 1
, , ,
6 8 4 3

21.

2 4 3 3
,
, ,
3 15 5 4

1)

4
15

2)

3
4

3)

3
5

4)

2
3

22.

23.

5 6 13 9
, ,
,
8 11 22 13

1)

5
8

2)

6
11

3)

13
22

4)

9
13

3 5 2 8
, , ,
4 7 3 11
1)

3
4

2)

5
7

3)

2
3

4)

8
11

Qn: (24 - 26) Find the smallest of the given


fraction.
24.

25.

26.

2 5 9 9 7
, ,
,
,
3 7 13 14 4

1)

9
14

2)

2
3

3)

7
4

4)

5
7

11 14 17 23 29
,
,
,
,
14 17 20 26 32

1)

29
32

2)

11
14

3)

17
20

4)

14
17

5 3 5 6
, , ,
6 4 8 7

1)

3
4

2)

6
7

3)

5
8

4)

5
6

2 7 . A heap of stones can be made in groups


of 21 but when made up into groups of 16,
20, 25 and 45 there are 3 stones left in each

case, The number of stones in the heap is


1) 3 6 0 0
2)
3603
3) 7 2 0 0
4)
7203
2 8 . Three measuring rods are 64cm, 80cm
and 96 cm in length. The least length of cloth
(in metres) that can be measured exact number of times using any of the three rods is
1) 0.96m
2)
9.6m
3) 96 m
4)
96 0m
2 9 . The largest number, which exactly divides
the product of any three consecutive integers
is
1) 2
2) 3
3) 6
4) 1 2
3 0 . The L.C.M. of two numbers is 63 and their
H.C.F. is 9. If one of the numbers is 27, the
other number will be
1) 9
2) 2 1
3) 1 7
4) 1 8 9
3 1 . The HCF of two numbers is 32 and their
product is 10240. Find their L.C.M?
1) 6 4 0 2) 3 2 0 3) 3 2 4 4) 2 3 0
3 2 . A gardener had a number of shrubs to
plant in rows. At first he tried to plant 8, then
12 and then 16 in a row but he had always 3
shrubs left with him. On trying 7 he had none
left. Find the total number of shrubs.
1) 1 4 7 2) 1 5 0 3) 1 3 7 4) 1 5 4
3 3 . Six bells commencing tolling together and
toll at intervals of 2,4,6,8,10 and 12 seconds
respectively. In 30 minutes, how many times
do they toll together.
1) 1 7
2) 1 5
3) 1 6
4) 2 0
3 4 . In a seminar the number of participants
in Hindi, English and Mathematics are 60, 84
and 108 respectively. Find the minimum number of rooms required, where in each room
the same number of participants are to be
seated; and all of them being in the same subject.
1) 2 0
2) 2 2
3) 2 5
4) 2 1
3 5 . Find the least number that being increased
by 8 is divisible by 32, 36 and 40
1) 1 4 3 0
2) 1 4 0 0
3) 1 4 3 2
4) 1 4 2 5

3 6 . Find the least multiple of 11 which when


divided by 8, 12 and 16 leaves 3 as remainder.
1) 1 0 0 2) 9 0
3) 9 9
4) 8 8
3 7 . The least multiple of 7 which leaves a
remainder 4 when divided by 6, 9, 15 and 18
is
1) 7 4
2) 9 4
3) 1 8 4 4) 3 6 4
3 8 . The greatest number that will divide 187,

233 and 279 leaving the same remainder in


each case is
1) 3 0
2) 3 6
3) 4 6
4) 5 6
3 9 . The floor of a room of dimensions 6.5m
x 4.0 m is to be paved with square marble
slabs. The length of the largest possible slab
is:
1) 25 cm
2) 75 cm
3) 50 cm
4) 100 cm

ANSWERS TO PRACTICE TEST - 6


1.

(2)

2.

(3)

3.

(1)

4.

(2)

5.

(4)

6.

(1)

7.

(1)

8.(4)

9.

(1)

10. (1)

11. (3)

12. (1)

13. (1)

14. (3)

15. (1)

16.(2)

17. (4)

18. (4)

19. (3)

20. (1)

21.(2)

22. (4)

23. (1)

24. (1)

25. (2)

26. (3)

27. (4)

28.(2)

29.(3)

30. (2)

31. (2)

32.(1)

33. (2)

34. (4)

35. (3)

36. (3)

37. (4)

38. (3)

39. (3)

Bank Probationary Officer


Q uantitative Aptitude

PERCENTAGE

A fraction with its denominator as `100'


is called a percentage. Percentage means per
hundred.
So it is a fraction of the form

if a student secures y marks and fails by z


marks, then the maximum mark

6
37
151
,
and
and these fractions
100 100
100

A candidate scores x% in an examination fails by `a' marks while another candidate who scores y% gets `b' marks more than
the minimum required for a pass, then the

can be expressed as 6%, 37% and 151%


respectively.
In such a fraction, the numerator is called
rate percent.
To express x% as a fraction or a decimal, divide x by 100.
If the price of an item increases by r%,
then the reduction in consumption, so that the
expenditure remains the same is
r

x100%
r 100

If the price of the commodity decreases


by r%, the increase in consumption, so that
the expenditure remains the same is

If the length of a rectangle is increased


by x% and the breadth is decreased by y%,
then the area is increased or decreased by
xy
)% according to the (+) ve or
100
(-) ve sign obtained.
If the present population is P which increases R% annually, then
(i) the population after n years
(x y

100 R

100

x2

%
100
If pass marks in an examination is x% and

= P
(ii)

xy

x y
%

100
If there is a decrease instead of increase, a
negative sign is attached to the corresponding
rate percent.
If the value of a number is first increased
by x% and later it is decreased by x% then net
change is always a decrease which is equal to

100 (a b)
y x

maximum mark =

x100%
1 0 0 r

If the value is first increased by x% and


then by y%, the final increase is

1 0 0 ( y z)
x

the population n years ago


=

100
P

100 R

If the present value of a machine is P


which depreciates at R% per annum, then
(i) the value of the machine after n years
100 R

100

= P
(ii)

the value of the machine n years ago


=

100
P

100 R

If x% students failed in a particular subject, y% students failed in another subject, and


z% students failed in both subjects, then the
pass present = 100+z-( x+y)
Fractional Equivalents of important percents
1

1%

1
4
1
3

4%

100

50

25

40%
6

2%
60%

%
%

1
16
1
12

12
16

1
2
2
3

80%

%
%

25%

8
1

33

1
3

5%

Then 36% of x = 27 or

1
3
37 %
2
8

1
4
1
3

2
2
66 %
3
3

2 7 x1 0 0
75
36

Hence, maximum marks = 75


The answer can be arrived quickly by
Maximum marks =

2
25
1
50%
2
1
5
83 %
3
6
8%

1
10 %
10
1
5
62 %
2
8
87

1
2

7
8

20%

5
3
75%
48
2
1
4
133 %
3
3

Find 33

1
% of 600
3

2.

1
1
% of 600 =
x600 = 200
3
3

What percent of 144 is 36?


Ans: Let x% of 144 = 36
(ie)

x
x 144 36
100

(ie) x
3.

3 6 x1 0 0
25
144

2.5 is 5% of what?
Ans : Let the number be x

5% of x 2.5
5
x x 2 .5 x 5 0
100

100 x 27
75
36

40
x 3 0 (ie) x 1 3 0

5
100

x 30 x

Ans: 33

1 0 0 (1 7 1 0 )
36

5 . Subtracting 40% of a number from the


number, we get the result as 30. Find the number.
Ans: Let the number be x.

SOLVED EXAMPLES:
1.

36
x x 27
100

20
100% 1

4
In an examination 36% are pass marks.
If an examinee gets 17 marks and fails by 10
marks, what are the maximum marks?
Ans : Pass mark=(17+10)= 27
Let maximum marks be x

5
50
3

6 . If the price of sugar be increased by


25%, find by how much percent must its consumption be decreased to keep the expenditure fixed on sugar?
Ans:
Decrease in consumption

25

2 5 x1 0 0
100% =
% 20%
=
100 25
125
7 . The salary of a worker was first increased
by 10% and thereafter decreased by 5%.
What was the effect in his salary?
10 x5

%
Ans: % effect = 10 5
100

His salary is increased by 4.5% (because


the sign is +ve.)

8 . The value of a machine depriciates at the


rate of 10% per annum. If its present value is
Rs. 81,000 what will be its worth after 2
year s?
Ans: The value of the machine after

2 years = Rs. 81,000x 1


= Rs. 81000 x

10

100

9
9
x
Rs. 6 5 , 6 1 0
10 10

9 . Due to fall of 10% in the rate of sugar,


500 gm more sugar can be purchased for
Rs.
140. Find the original rate and reduced rate.
Ans : Money spent originally=Rs. 140
Less Money to be spent now
= 10% of 140
= Rs. 14

Rs. 14 now yield 500gm sugar


Present rate of sugar = Rs. 28 per kg.
If the present value is Rs. 90, the original value
= Rs. 100
If the present value is Rs. 28 the original value
= Rs.

100
x 28
90

= Rs. 31.11
1 0 . In an examination, 42% students failed
in History and 52% failed in Geography. If 17%
students failed in both subjects, find the percentage of those students who passed in both
the subjects.
Ans:- Pass percent=100+17-(42+52)
= 117 - 94
= 23
PRACTICE TEST
1.

65% of 7+35% of 3 = ?% of 56
1) 1
2) 1 0
3) 5 0
4) 1 0 0

2 . What is 20% of a number whose 200%


is 360?
1) 7 2
2) 3 6
3) 5 2
4) 1 4 4

3.

What percent of

4
2
is
?
7
35

1) 2 . 5 %
2) 1 0 0 0 %
3) 2 5 %
4) 1 0 %
4 . The total income of A and B is Rs. 6000.
A spends 60% of his income and B spends
80% of his income. If their savings are equal,
then the income of A is,
1) Rs. 3500
2) Rs. 2000
3) Rs. 4000
4) Rs. 3000
5 . With an increase of Rs. 2,000, Vishnu's
monthly salary became Rs. 12,000. What is
the percent increase in his salary?
1) 2 0
2) 2 5
3) 4 0
4) 8 0
6 . if 75% of the students in a school are
boys and the number of girls is 420, the number of boys is
1) 1 1 7 6
2) 1 3 5 0
3) 1 2 6 0
4) 1 1 2 5
7 . The salary of a worker is first increased
by 10% and therafter it was reduced by 10%.
What was the change in his salary?
1) 1% increase
2) 5% increase
3) no change
4) 1% decrease
8 . A water tank contains 5% salt by
weight. x litres of fresh water is added to 40
litres of tank water, so that the solution contains 2% salt. The value of x is
1) 4 0
2) 5 0
3) 5 5
4) 6 0
9 . The population of a town increases 5%
annually. If it is 15,435 now, what was it 2
years ago?
1) 14 , 0 00
2) 13 , 4 73
3) 12 , 3 45
4) 10 , 1 45
1 0 . Navin spends 15% of his salary on cloths,
30% on food and 10% on transport. After this
if he is left with Rs. 900/- what is his salary?
1) Rs. 1,500
2) Rs. 2000
3) Rs. 1,635
4) Rs. 2500
1 1 . When the price of an article was reduced
by 15% the sale of the article is increased by

20%. What was the effect on the sales?


1) 2% increase
2) 1% increase
3) 2% decrease
4) 1% decrease
1 2 . In an election between two candidates,
the one gets 35% of the votes polled is defeated by 15000 votes. The number of votes
casted by the winning candidate is
1) 15 , 0 00
2) 1, 75, 000
3) 32 , 5 00
4) 52 , 5 00
1 3 . In an examination, 70% students passed
in English and 75% in Hindi while 20% failed
in both the subjects. If 260 students passed in
both the subjects, the total number of students
is
1) 4 0 0
2) 5 0 0
3) 3 4 0
4) 4 6 0
1 4 . If the radius of a circle is diminished by
10%, the area is diminished by
1) 3 6 %
2) 2 0 %
3) 1 9 %
4) 1 0 %
1 5 . The price of an article is cut by 10%. In
order to restore it to its former value, the new
price must be increased by
1)
3)

10

1
%
3

1
11 %
9

1 8 . In an employment exchange, 40% of the


job seekers are graduates, 20% are postgraduates and remaining 6000 are non-graduates. How many post-graduate job seekers are
there?
1) 3, 00 0
2) 6, 00 0
3) 9, 00 0
4) 12 , 0 00
1 9 . A company hired a salesman on a monthly
salary of Rs. 3,000. In addition to it, the salesman was entitled for 20% commission on the
monthly sale. How much sale the salesman
should do if he wants his monthly income as
Rs. 10,000?
1) Rs. 50,000
2) Rs. 15,000
3) Rs. 35,000
4) Rs. 21,000
2 0 . In a public sector company, 30% employees opted for pension and 50% employees
opted for provident fund. The remaining employees were uncertain. If the difference between those who opted for provident fund and
those who were uncertain was 1440, how
many employees were there in the company?
1) 7, 20 0
2) 2, 40 0
3) 2, 88 0
4) 4, 80 0
2 1 . Prasanna spends 25% of her monthly in-

2) 1 1 %
4) 12

1
%
9

1 6 . The breadth of a rectangular field is 60%


of its length. If the perimeter of the field is
800m, What is the area of the field?
1) 37,500 sq.m.
2) 4,800 sq.m
3) 18,750 sq.m
4) 40,000 sq.m
1 7 . In a factory, 60% of the employees are
males. Among them 20% are matriculates and
the remaining are graduates. Among the females 40% are matriculates and the remaining are graduates. If the total number of female employees in the factory is 640, how
many graduates are there in the factory ?
1) 1024
2) 896
3) 1,152
4) 1 0 0 0

come on petrol for her car,

2
rd of the remain3

ing income on house hold items, rent, etc. If


she is left with Rs. 1,800 with her at the end
of the month how much does she spend on
petrol?
1) Rs. 1,800
2) Rs. 720
3) Rs. 2,500
4) Rs. 1,440
2 2 . Rajesh earns Rs. 2,300 per month. He
spends Rs. 1,200 on food, Rs. 630 on conveyance, 10% of his monthly income on other incidentals and saves the remaining amount. How
much money will he save in one year?
1) Rs. 2300
2) Rs. 2880
3) Rs. 2600
4) Rs. 2400
2 3 . In an examination, Hari got 8 marks less
than 80% of the full marks and Ravi got 5
marks more than 70% of the full marks. Hari

beats Ravi by 2 marks. The marks scored by


Ravi is
1) 9 0
2) 1 1 0 3) 1 3 0 4) 1 4 0
2 4 . A candidate secured 20% marks in a test
and failed by 10 marks. Another candidate
secured 42% and got one mark more than the
bare minimum to pass. The maximum mark is
1) 5 0
2) 6 0
3) 6 5
4) 7 0
2 5 . A's salary is 20% less than B's salary and
B's salary is 20% more than C's salary. If the
sum of the salaries of A and B is Rs. 5,400
then C's salary is
1) Rs. 3,000
2) Rs, 2,880
3) Rs. 2,500
4) Rs. 2,700
2 6 . The price of some commodity was reduced by 20%. To bring the price of that commodity to the original level, by how much percentage of the increase in the price of that
commodity will have to be made?
1) 1 2. 5 %
2) 2 0 %
3) 2 5 %
4) 3 7. 5 %
2 7 . In a college election between two candidates, the candidate who got 62% of the
votes, won by 144 votes. The total number of
votes is
1) 6 0 0
2) 8 0 0
3) 9 2 5
4) 1 2 0 0
2 8 . In a class, 30% of the boys play football,
40% of the remaining play cricket and the
remaining 21 boys play different other games.

How many boys are there in the class?


1) 5 0
2) 1 0 0 3) 4 8
4) 9 6
2 9 . In an examination 40% of the students
failed in English, 60% passed in Mathematics.
If 10% of the students failed in both the subjects, what is the pass percent?
1)

33

1
3

2) 3 0

3)

36

2
3

4) 5 0

3 0 . In a library, 30% of books are on computers, 5% on English, 35% on Science and


remaining 900 are on various other fields. How
many books on English are there in the library?
1) 3 0 0 0
2) 3 0 0
3) 1 5 0
4) 2 0 0
3 1 . Ramu spends 40% of his income on food,
1/3 rd of the remaining on transport and 10%
of the remaining on books. If he spends Rs.
250 for rent of his house, what is his salary?
1) Rs. 6,000
2) Rs. 625
3) Rs. 62,500
4) Cannot be
determined
3 2 . In an examination, A got 10% marks less
than B, B got 25% more than C, and C got
20% less than D. If A got 360 out of 500, the
percentage marks obtained by D was
1) 7 0
2) 7 5
3) 8 0
4) 8 5

ANSWERS TO PRACTICE
1.

(2)

2.

(2)

9.

(1)

1 0 . (2)

1 7 . (3)
2 5 . (3)

3.

(4)

4.

(2)

5 . (1)

6.

(3)

7.

(4)

8.(4)

1 1 . (1)

1 2 . (3)

13. (1)

1 4 . (3)

1 5 . (3)

16.(1)

1 8 . (1)

1 9 . (3)

2 0 . (4)

21.(1)

2 2 . (2)

2 3 . (2)

2 4 . (1)

2 6 . (3)

2 7 . (1)

28.(1)

29.(2)

3 0 . (3)

3 1 . (4)

32.(3)

Bank Probationery Officer

P RO FI T & L OS S

Quantitative Aptitude

Cost Price: The price for which an article is


purchased is called the Cost Price (C.P.)
Selling price : The price at which an article is
sold is called the Selling Price (S.P.)
Profit (Gain) : The difference between the selling price and the cost price (when S.P. is more
than C.P) is called the Profit.
Loss:The difference between the cost price
and selling price (when C.P. is more than S.P.)
is called the Loss.
Points to remember:
1 . Gain = (S.P) - (C.P); Loss = (C.P) - (S.P)
2.

Gain%=

G ain x 1 0 0
SP CP
= CP x 1 00
C. P.

Lo ssx 1 0 0
Loss%=
=
C. P.

CP S P

x 100
CP

3 . When the cost price and gain percent are


given,
100 Gain%

100

S.P. = C.P. x

4 . When the cost price and loss percent are


given
100 Loss%

100

S.P. = C.P. x

5 . When the selling price and gain percent


are given

100
C.P. = S.P. x 100 Gain%

6 . When the selling price and loss percent


are given
100

C.P. = S.P. x (1 0 0 Lo ss% )

7 . The discount percent is calculated on the


marked price.
Discount percent
=

Disc ount
x100
M arked price

8 . If there are two successive profits of x%


and y% in a transaction then the resultant
profit percent is
xy

xy
100

9 . If there is a profit of x% and loss of y% in


a transaction, then the resultant profit and loss
percent is
xy
according to the (+)ve and
100
the (-)ve signs respectively.
xy

1 0 . If cost price of x articles is equal to the


selling price of y articles, then profit percent.
=

x y

x 100
y

SOLVED EXAMPLES
1 . A man buys a toy for Rs. 25 and sells it
for Rs. 30. Find his gain percent.
Ans: Gain = 30 - 25 = Rs. 5
Gain % =

5
x 1 0 0 Rs. 2 0 %
25

2 . By selling a watch for Rs. 144 a man


loses 10%. At what price should he sell it in
order to gain 10%?
Ans: S.P. = Rs. 144; Loss = 10%

100
C. P. Rs
x 144
100 10

= Rs.

100
x 1 4 4 Rs. 1 6 0
90

If S.P = Rs. 50, loss = Rs. ( x-50)


(X 4 0 ) (X 5 0 )

Gain required = 15%


S. P. Rs.

115
x 1 6 0 Rs. 1 8 4
100

Short cut:
Required SP =

144
x 1 1 5 Rs.1 8 4
90

3 . I sold a book at a profit of 16%. Had I


sold it for Rs. 18 more, 20% would have been
gained. Find the cost price.
Ans: Here 120% of C.P. _ 116% of C.P.
= Rs. 18

4% of cos t Rs.18

10

5
x X
100

5
x X X R s. 2 0 0 0
100

6 . A grocer sells rice at a profit of 10% and


uses weights which are 20% less than the
marked weight. The total gain earned by him
will be
Ans: Let us consider a packet of rice marked
1 Kg.
Then its actual weight=80% of 1Kg.=0.8 Kg.
Let C.P. of 1 Kg be Rs. x
Then C.P. of 0.8Kg = Rs. 0.8 x
Now, S.P. = 110% of C.P. of 1 Kg
110

18 x100
C. P.
Rs.450
4
Formula :
M o re g ain x 1 0 0

C.P = D if f . in p erc en t ag e p ro f it
=

1 8 x1 0 0
Rs. 4 5 0
20 16

4 . A shopkeeper mixes two varieties of tea,


one costing Rs. 35 per kg. and another at
Rs. 45 per kg. in the ratio 3:2. If he sells the
mixed variety at Rs. 41.60 per kg. what is his
gain or loss percent?
Ans: - C.P of 5 kg. mix
= Rs. (35x3+45x2) = Rs. 195
S.P. of 5 kg. mix = Rs. (41.60x5)
= Rs. 208
Gain = Rs. (208-195) = Rs. 13
13
2
x 100 6 %
Gain% =
195
3

5 . By selling a table for Rs. 40 instead of


Rs. 50, 5% more is lost. Find the cost of the
table.
Ans : Let the C.P. be Rs. x
Then if S.P.=Rs. 40, loss=Rs. ( x-40)

. X
= 1 0 0 x X Rs. 11
0 .3 X

Gain %= 0 .8 X x 100% = 37.5%


7 . The cost price of 10 articles is equal to
the selling price of 9 articles. Find the gain
percent.
Ans: Let the cp of 1 article = Rs. 1
Then CP of 9 articles = Rs. 9
CP of 10 articles = Rs. 10
SP of 9 articles = Rs. 10
100
1
10 9
11 %
100
9
9
9

Gain =

8 . A boy buys oranges at Rs. 2 for 3 oranges and sells them at one rupee each. To
make a profit of Rs. 10 he must sell:
Ans: Suppose he sells x oranges.

2
x
3
SP of x oranges = Rs. x
CP of x oranges = Rs.

Profit on x oranges = x

x
10 x = 30
3

2x
x
=
3
3

9 . A man sells two horses for Rs. 4000 each


neither losing nor gaining in the deal. If he sold
one horse at a gain of 25%, the other horse
would be sold at a loss of:
Ans: Let the other horse be sold at x% loss
Then 25 - x-

25 x
0
100

x 20%
1 0 . A discount series of 10%, 20% and 40%
is equal to a single discount of .....
Ans: Equivalent single discount
= 100

60
80
x
x 90 56.8%
100 100

PRACTICE TEST
1 . By selling an article for Rs. 100, one gains
Rs. 10. Then the gain percent is
1) 1 0 %
2) 9 %
3)

1
11 %
9

4) 10

1
%
2

2 . A loss of 5% was suffered by selling a


plot for Rs. 4,085. The cost price of the plot
wa s:
1) Rs. 4350
2) Rs. 4259.25
3) Rs. 4200
4) Rs. 4300
3 . A dealer sold a mixer for Rs. 420 at a
loss of 12.5%. At what price should he have
sold it to gain 12.5%?
1) Rs. 620
2) Rs. 540
3) Rs. 650
4) Rs. 750 4 .
4 . On selling 33m. of cloth, a person gained
an amount equivalent to the S.P. of 11m. of
cloth. The profit in the deal is
1) 5 0 %
2) 2 0 %
3) 7 0 %
4) 3 0 %
5 . There are two consecutive discounts of
35% and 10% on a saree. If a person paid Rs.
1170 for that, then what was the original price
of the saree before the discounts?
1) Rs. 2,000
2) Rs. 1,800
3) Rs. 1,900
4) Rs. 1,700

6 . A dishonest dealer claims to sell his goods


at cost price, but he uses a weight of 960 gm
for the kg weight. His gain percent is
1)

4%

3)

1
%
2

2) 4

1
%
6

4) 3

3
%
4

7 . By selling a vehicle for Rs. 36,300, a person gains 21% profit. What was his gain in
Rupees?
1) Rs. 3,000
2) Rs. 7,623
3) Rs. 3,600
4) Rs. 6,300
8 . Hameed bought a calculator for Rs. 520
and sold it with 15% profit on the price he
bought. At what price did he sell the calculator?
1) Rs. 598/2) Rs. 542/3) Rs. 528/4) Rs. 780/9 . Ramesh purchased four old cycles at the
rate of Rs. 625 for each. He spent Rs. 175 on
each cycle for repairing and painting. At what
price should he sell each cycle in order to make
35% profit on the money he spent?
1) Rs. 1,019
2) Rs. 1,000
3) Rs. 1,080
4) Rs. 844
1 0 . Raghu bought 10kg. of sugar at the rate
of Rs. 14 per kg and 15kg. of sugar at the
rate of Rs. 16 per kg. He mixed the two varieties and sold the mixture at the rate of Rs.
18 per kg. What was his total gain by doing
so?
1) Rs. 50
2) Rs. 70
3) Rs. 40
4) Rs. 80
1 1 . Ravi buys a radio three-fourth of its value
and sells it for 20% more than its value. What
will be the profit percent?
1) 5 0 %
2) 4 0 %
3) 6 0 %
4) 7 0 %
1 2 . A man buys pencils at 10 for Rs. 3 and
sells at 8 for Rs. 3. His gain percent is
1) 20
2) 25
3) 30
4) 27

1 3 . A fruit seller buys lemons at 2 for a rupee and sells them at 5 for three rupees. His
gain percent is
1) 1 0 %
2) 1 5 %
3) 2 0 %
4) 1 2 %
1 4 . Toffee are bought at a rate of 8 for one
rupee. To gain 60% they must be sold at
1) 6 for Re. 1
2) 5 for Re. 1
3) 9 for Re. 2
4) 24 for Re. 5
1 5 . By selling sugar at Rs. 5.58 per kg. a man
loses 7%. To gain 7% it must be sold at the
rate of Rs.
1) 5.62 per kg
2) 6.42 per kg
3) 7.32 per kg
4) 6.62 per kg
1 6 . A tradesman's prices are 20% above
C.P. He allows his customers some discount
on his bill and makes a profit of 8%. The rate
of discount is :
1) 1 0 %
2) 1 2 %
3) 1 4 %
4) 1 6 %
1 7 . An article was sold at a loss of 5%.If it
were sold for Rs. 30 more ,the gain would have
been 1.25%. The cost price of the article is
1) Rs. 488
2) Rs. 480
3) Rs. 460
4) Rs. 420
1 8 . Anitha sold a painting at a profit of 11%.
Had she sold it for Rs.175 more, she would
have gained 18%. The C.P of the painting is
1) Rs. 2250
2) Rs. 2350
3) Rs. 2500
4) Rs. 2550
1 9 . Pradeep bought a toy with 20% discount
on its labelled price. He sold it for Rs. 468 at
4% profit on the labelled price. At what price
did he by the toy ?
1) Rs. 450
2) Rs. 360
3) Rs. 390
4) Rs. 380
2 0 . Arun purchased a bag with 25% discount
on the labelled price. At what percentage
profit on the price he bought should he sell it
to make 20% profit on the labelled price?
1) 6 0 %
2) 4 0 %
3) 4 5 %
4) 5 0 %

2 1 . Nimesh bought a cycle for Rs. 1,850.He


spent Rs.380 for buying different accessories. Approximately, at what price should he
sell the cycle to make 20% profit in the transaction?
1)

Rs. 2900

2) Rs. 2676

3)

Rs. 3000

4) Rs. 3125

22. A man buys an article and sells it at a profit


of 20%.If he bought at 20%less and sold it
for Rs. 75 less, he would have gained 25%.
What is the cost price ?
1)

Rs. 375

2) Rs. 425

3)

Rs. 350

4)Data inadequate

23. A man sells a car to his friend at 10%


loss. If the friend sells it for Rs. 54,000 and
gains 20%, the C.P. of the car was:
1)

Rs. 25,000

2) Rs. 37,500

3)

Rs. 50,000

4) Rs. 60,000

24. Listed price of an article is Rs. 65. A


customer pays Rs. 56.16 for it. He was given
two successive discounts. If the first discount
is 10% find the second .
1)

8%

2) 6%

3) 5 %

4) 4%

25. A single discount equivalent to a discount


series of 20%,10% and 25% is
1)

55%

2) 54%

3)

46%

4) 42%

2 6 . A trader allows two successive discounts


of 20% and 10%. if he sells an article for Rs.
108, then the market price of the article is
1)

Rs. 140

2) Rs. 142

3)

Rs. 148

4) Rs. 150

2 7 . A person bought 20 litres of milk at the


rate of Rs. 8 per litre. He got it churned after
spending Rs. 10 and got 5 kg. of cream and
20 litres of toned milk. If he sold the cream at
Rs. 30 per kg. and toned milk at Rs. 4 per litre, his profit in the transaction is
1)

25%

2) 35.3%

3)

3 7. 5 %

4) 42%

2 8 . A dishonest dealer sells his goods at the


cost price but still earns a profit of 25%. by
underweighing. What weight does he use for
a kg ?
1) 750gm.
2) 800gm.
3) 825gm.
4) 825gm.
2 9 . On selling a certain commodity
for
Rs 425, there is as much gain as loss on selling it for Rs 355. the C.P of the commodity
1) Rs. 370
2) Rs. 385
3) Rs. 390
4) Rs. 400
30. A person bought some oranges @ Rs. 10
per dozen and bought the same number of oranges @ Rs. 8 per dozen. He sold these oranges @ Rs. 11 per dozen and gained Rs 120.
The total number of oranges bought by him
wa s
1) 30 dozen
2) 40 dozen
3) 50 dozen
4) 60 dozen
3 1 . The marked price of a table is Rs 3000
and is available at successive discounts of 20%
and 10% respectively. If there is an additional
discount of 5% on cash payment, then what
is the cash price of the table?
1) Rs. 2400
2) Rs. 2052
3) Rs. 2100
4) Rs.1640
3 2 . Ramesh bought 12kg . of sugar at the
rate of Rs 9.50 per kg. and 18 kg of sugar at
the rate of Rs. 12.50 per kg. He mixed the
two varieties and sold the mixture at the rate
of Rs.12.00 per kg. Approximately what was
his percentage gain in this transaction?
1) 6
2) 8
3) 4
4) Data inadequate

3 3 . A shop keeper bought 16 dozen toys at


the rate of Rs. 703.20 per dozen. He sold them
with 20% profit on the price he bought. Approximately , at what price did he sell each toy?
1) Rs.70
2) Rs.75
3) Rs. 65
4) Rs.80
3 4 . Prashanth buys mangoes at 3 per kg.for
Rs 21 and sells them at 5 kg. for Rs.50. To
earn Rs. 102 as profit ,he must sell
1) 34 kg.
2) 52 kg.
3) 26 kg.
4) 32 kg.
3 5 . A man loses Rs. 50 by selling toys at the
rate of Rs 3 per toy but gains Rs .75. If he
sells them at Rs 3.25 per toy. The number of
toys sold by him is
1) 5 0 0
2) 6 0 0
3) 7 5 0
4) 1 0 0 0
3 6 . A radio is sold at a gain of 16%. If it had
been bought at 10% less and sold for Rs. 14
less; there would have been a gain of 25%.
The cost price of the radio is
1) Rs. 500
2) Rs. 375
3) Rs. 400
3) Rs. 500
3 7 . Price of food grains have risen by 10%
and of other items of consumption by 15%. If
the ratio of an employee's expenditure on food
grains and other items is 2:5, by how much
should his salary be increased so that he may
maintain the same level of consumption as
before, assuming that his present salary is Rs.
3500?
1) Rs. 300
2) Rs. 350
3) Rs. 375
4) Rs. 475

ANSWERS TO PRACTICE
1.

(3)

2.

(4)

9.

(3)

1 0 . (2)

1 7 . (2)

3.

(2)

4.

(1)

5 . (1)

6.

(2)

7.

(4)

8.(1)

1 1 . (3)

1 2 . (2)

13. (3)

1 4 . (2)

1 5 . (2)

16.(1)

1 8 . (3)

1 9 . (2)

2 0 . (1)

21.(2)

2 2 . (1)

2 3 . (3)

2 4 . (4)

2 5 . (3)

2 6 . (4)

2 7 . (2)

28.(2)

29.(3)

3 0 . (4)

3 1 . (2)

32.(1)

3 3 . (1)

3 4 . (1)

3 5 . (1)

36.(3)

37.(4)

Bank Probationary Officer


Quantitative Aptitude

AVERAGE

An average, or an arithmetic mean, is the


sum of `n' different data divided by `n'
su m o f d at a

Average = N o. o f d at a
sum of data
No. of data = Average

Sum = Average x No. of data


Points to remember:
1 . Age of new entrant = New average +
No. of old members x change in average
2 . Age of one who left = New average No. of old members x change in average
3 . Age of new person = Age of the removed
person + No. of members x change in avera ge
In all the above three cases, if there is a
decrease in the average, the sign of change in
average will be negative.
4 . If a certain distance is covered at x km/hr
and the same distance is covered by y km/hr,
then the average speed during the whole journey is
2 xy
km/hr
x y

Examples
1 . The average age of 30 boys of a class is
equal to 14 years. When the age of the class
teacher is included the average becomes 15
years. Find the age of the class teacher.
Total age of 30 boys = 14x30=420 years
Total age when the teacher is included
= 15x31 = 465 years
Age of the class teacher
= 465 - 420 = 45 years

Direct Formula
Age of new entrant = New average + No.
of old members x change in average
= 15+30(15-14) = 45 years.
2 . The average weight of 8 men is increased
by 1.5 g. when one of the men who weighs
65 kg is replaced by a new man. The weight
of the new man is:
Weight of the new man = Weight of the man
replaced + (Number x change in average)
= 65 + (8x1.5) = 65+12 = 77 kg.
3 . The average of 11 results is 50. If the
average of first six results is 49 and that of
last six is 52, find the sixth result.
The sum of 11 results = 11x50 = 550
The sum of first 6 results = 6x49 = 294
The sum of last 6 results = 6x52 = 312
Sixth results = 294+312-550 = 56
4 . There were 35 students in a hostel. If
the number of students increased by 7, the
expenses of the mess were increased by Rs.
42 per day, while the average expenditure per
head diminished by Re. 1. The original expenditure of the mess was:
Ans: Let the original expenditure per head be
Rs. x.
Then 35 x + 42 = ( x-1) 42
35 x + 42 = 42 x - 42 or x = 12
5 . The average expenditure of a man for the
first five months was Rs. 120 and for the next
seven months is Rs. 130. What was his
monthly average income if he saved Rs. 290
in that years.
Total income for 12 months.
= Rs. (120x5+130x7+290)
= Rs. 1800

Average monthly income


=

1800
Rs.150
12

6 . There are 50 boys in a class. Their average weight is 45 kg. When one boy leaves the
class, the average reduces by 100 gms. Find
the weight of the boy who left the class.
Weight of the boy left = New average No. of old members x change in average
= 44.9-50x(-0.1) = 44.9+5 = 49.9kg.
7 . The average attendance in a school for
the first 4 days of the week is 30 and for the
first 5 days of the week is 32. The attendance
on the fifth day is:
Total attendance for the first 4 days
= 4x30 = 120
Total attendance for the first 5 days
= 5x32 = 160
Attendance on the fifth day
= 160-120 = 40
PRACTICE TEST
1 . The marks obtained by a student in five
subjects are 68,73,62,85 and 79. Find the
average score.
1) 7 3
2) 7 3. 4
3) 7 5
4) 7 4. 5
2 . The average income of a group of 9 workers is Rs. 137.30 and that of another group of
7 workers is Rs. 95.06. The average income
of all the persons is:
1) Rs. 118.82
2) Rs. 116.18
3) Rs. 125.18
4) Rs. 128.15
3 . There are 40 boys in a class. One boy
weighing 40 kg goes away, and at the same
time another boy joins the class. If the average weight of the class is thus increased by
100 gm, the weight of the new boy is.
1) 39.9 kg
2) 44.1 kg
3) 40.1 kg
4) 44 kg
4 . My average expenses for 4 days is Rs.
6.00. I spend Rs. 7.70 on first day and Rs.

6.30 on second day. If I spent Rs. 10 on third


day, what did I spend on the 4th day?
1) Rs. 2/2) Rs. 3/3) Rs. 4/4) Nothing
5 . The average temperature on Tuesday,
Wednesday and Thursday was 37 centigrade.
The average tempeature on Wednesday,
Thursday and Friday was 38 centigrade. If
the temperature on Friday was 39 centigrade,
the temperature on Tuesday was:
1) 35 C
2) 36 C
3) 37 C
4) 38 C
6 . The average age of students in two
classes of 40 students each is 10 years and 8
years respectively. The average age of students in both the classes taken together is:
1) 8 years
2) 9 years
3) 10 years
4) 11 years
7 . The average age of 50 soldiers in troop
is 25 years. If the captain's age is included,
the average age of all of them still remains
the same. What is the captain's age in years?
1) 2 5
2) More than 25
3) Less than 25
4) Cannot be
determined
8 . Two towns A and B are some distance
apart. A girl cycles from A to B at a speed of
10 km/hr and then back from B to A at the
rate of 15 km/ hr. The average speed during
the journey is:
1) 12.5 km/hr
2) 15 km/hr
3) 12 km/hr
4) 13.5 km/hr
9 . An employee's average contribution to
his provident fund for the first 9 months was
Rs. 3,500 each and for each of the remaining
3 months, the contribution was Rs. 5,500. By
what amount was his total contribution short
of Rs. 58,000?
1) Rs. 4,000
2) Rs. 16,500
3) Rs. 8,000
4) Rs. 10,000
1 0 . What fraction must be subtracted from
the sum of

1
1
and
to have an average of
6
4

1
of these the two fractiaons?
12

1)

1
3

2)

1
2

3)

1
4

4)

1
8

1 1 . The average marks of 12 students was


calculated as 40. But it was later found that
marks of one student had been entered
wrongly as 42 instead of 54 and of another as
74 instead of 50. The correct average is:
1) 3 9
2) 4 0
3) 4 1
4) 4 3
1 2 . The average salary of workers in an industry is Rs. 2000, the average salary of 150
technicians being Rs. 4000, and the non-technicians being Rs. 1,250. The total number of
workers is
1) 4 5 0
2) 3 0 0
3) 5 5 0
4) 500
1 3 . The average age of a husband and a wife
who were married four years ago was 20 years
then. What will be the average age of the family now if they have a three years old child?
1)

15

2
3

years

2) 1 6

1
years
3

3) 17 years
4) 16 years
1 4 . The average of three consecutive odd
numbers is 39. What is the sum of the first
two of these numbers?
1) 7 8
2) 7 6
3) 2 4
4) 1 1
1 5 . If the average of 9 consecutive numbers
is 20, the highest of these numbers is:
1) 2 0
2) 2 1
3) 2 4
4) 2 6
1 6 . The sum of two consecutive even numbers is 23 more than the average of these two
numbers. What is the second number?
1) 2 2
2) 2 4
3) 2 6
4) Data indequate
1 7 . The average of 17 numbers is 10.9 If the
average of first nine is 10.5 and that of the
last nine is 11.4, the middle number is
1) 1 1. 8
2) 1 1. 4
3) 1 0. 9
4) 1 1. 7

1 8 . The average monthly expenditure of a


family was Rs. 2,200 during first 3 months,
Rs. 2,550 during next 4 months and Rs. 3,120
during last 5 months of the year. If the total
saving during the year was Rs. 1,260, find
average monthly income.
1) Rs. 3,960
2) Rs. 760.8
3) Rs. 2,805
4) Rs. 3,125
1 9 . 30 pens and 75 pencils were purchased
for Rs. 510. if the average price of a pencil
was Rs. 2.00, find the average price of a pen.
1) Rs. 12
2) Rs. 15
3) Rs. 19
4) Rs. 25
2 0 . the average age of the husband and wife
who were married 7 years ago was 25 years
then. The average age of the family including
the husband, wife and the child born during
the interval is 22 years, now. How old is the
child now?
1) 2 years
2) 3.5 years
3) 1 years
4) 4 years
2 1 . Average monthly income of a family of
four earning members was Rs. 735. One of
the earning members died and therefore the
average income came down to Rs. 650. The
income of the deceased was:
1) Rs. 820
2) Rs. 990
3) Rs. 692.50
4) Rs. 1,385
2 2 . A batsman has certain average runs for
20 innings. In the 21st inning, he served 107
runs thereby increasing his average by 2. What
is his average after 21 innings?
1) 6 7
2) 6 5
3) 6 0
4) 7 2
2 3 . Nine men went to hotel. 8 of them spent
Rs. 3 each over their meals and the ninth spent
Rs. 2 more than the average expenditure of
all the nine. The total money spent by all of
them was
1) Rs. 26
2) Rs. 40
3) Rs. 27
4) Rs. 29
2 4 . Keshav is given 12 days to drive a destination 1200 km. away. for the first 6 days he
goes 100km a day. Due to accident, he can-

not drive for 2 days. What is the average distance per day that he has to drive to reach his
destination in time?
1) 50 km
2) 100 km.
3) 150 km
4) 200 km.
2 5 . In a cricket team eleven, the average age
of eleven players is 28 years. Out of these,
the average of three groups of three players
each are 25 years, 28 years and 30 years.
The captain and the youngest player who is
11 years younger than the captain are not included in these groups. The age of the captain
is
1) 33 years
2) 34 years
3) 35 years
4) 36 years
2 6 . Average income of A and B is Rs. 3000
and of C and D is Rs. 500. What is the average income of A, B, C and D.
(1) 1 5 0 0
(2) 1 7 5 0
(3) 1 7 0 0
(4) 2 0 0 0
2 7 . Five years ago average age of P and Q
was 15 years. Average age of P, Q and R today is 20 years. How old R will be after 10
years.
(1) 32 yrs.
(2) 25 yrs.
(3) 35 yrs.
(4) 30 yrs.
2 8 . One of the two buses complete a journey

of 300 km in 7

1
hrs. and the other a journey
2

of 450 km in 10 hrs. Find the ratio of their


average speeds.
(1) 4: 5
2) 9: 8
(3) 8: 9
4) 5: 4
2 9 . A man spends Rs. 1800 per month on an
average for the first four months and Rs. 2000
per month for the next 8 months and saves
Rs. 5600 a year. What is his average monthly
income?
1) Rs. 2400
2) Rs. 2000
3) Rs. 1800
4) Rs. 2500
3 0 . In a class there are three divisions. The
number of students and the average marks in
mathematics in the three divisions are 30, 40,
30 and 40%, 30% and 50% respectively.
What are the average marks in mathematics
of the class?
1) 3 9
2) 4 0
3) 4 1
4) 5 1
3 1 . Visitors to a show were charged Rs. 15.00
each on the first day, Rs. 7.50 on the second
day, Rs. 2.50 on the third day and total attendance on the three days were in the ratio
2:5:13 respectively. The average charge per
person for the whole show is
1) Rs. 3.00
2) Rs. 4.50
3) Rs. 5.00
4) Rs. 7.50

ANSWERS TO PRACTICE TEST

1.

(2)

2.

(1)

9.

(4)

1 0 . (3)

1 7 . (1)
2 5 . (3)

3.

(4)

4.

(4)

5 . (2)

6.

(2)

7.

(1)

8.(3)

1 1 . (1)

1 2 . (3)

13. (3)

1 4 . (2)

1 5 . (3)

16.(2)

1 8 . (3)

1 9 . (1)

2 0 . (1)

21.(2)

2 2 . (1)

2 3 . (4)

2 4 . (3)

2 6 . (2)

2 7 . (4)

2 8 . (3)

2 9 . (1)

3 0 . (1)

3 1 . (3)

Bank Probationary Officer


Quantitative Aptitude

RATIO & PROPORTION


Ratio: The number of times one quantity contains another quantity of the same kind is called
ratio of the two quantities. The ratio of a to b
is written as
a : b a a b
b
In the ratio a : b, a and b are called the terms
of ratio, `a' is the antecedent and `b' is the
consequent.

ii)
In a:b : : c:d, d is the fourth proportional
to a,b and c.
iii) If x is the third proportional to a and b
then a: b : : b: x.
iv) Mean proportional between a and b is

(ab).
Other properties
If

a c

`or' a: b : : c:d.
b d

Points to remember:
i)
ii)
iii)

iv)

The order of the terms in a ratio is very


important
The quantities of a ratio must be expressed in the same units.
The ratio is unaltered if each term is
multiplied or divided by the same number.
When a certain quantity`q' is divided in
a given ratio a:b, the two parts are

aq
bq
and
ab
a b
v) If a: b and c : d are two ratios, then
ac: bd is called the compounded ratio of the
given ratios.
Proportion: The equality of the two ratios is
called proportion. Suppose the two ratios a:b
and c:d are equal, i.e, a:b = c:d, then we write,
a:b: : c:d
Here, a and d are called as extremes and b, c
are called means.
Rule:
i)
ad = bc `or'
Product of extremes = Product of
means.

i)

ab cd

b
d

ii)

ab c d

b
d

iii)

ab c d

ab c d

iv)

a c a c ka kc

b d b d kb kd

Solved examples:
1 . If a: b = 4:5 and b:c = 6:7, find the
ratios a:c and a:b:c
Given,

a 4 b 6
;
b 5 c 7

a b 4 6
a 24
x x (ie )
b c 5 7
c 35

a: c 24 : 35
Here `b' term is common to both the equations and so their corresponding values should
be made equal.
(ie) a:b
= 24 : 3 0
b:c =
30 : 35
a : b: c 24 : 30 : 35

2.

Divide Rs. 54 in the ratio 4 : 5


Sum of ratios = 4+5 = 9
4

First part = 54 x 9 = Rs. 24


5
Second part = 54 x
= Rs. 30
9

3 . In a ratio, which is equal to 7 : 8 , if the


antecedent is 35, what is the consequent?
Let the consequent be x

8x 35 ; x

4.

8x3 5
40
7

3
x 40 30 litres
4

Quantity of water = 40-30=10 litres


Suppose x litres of water be added in 40 litres
of mixture.
30

1 0 x 1 2 10 x 30 x 5 litres
7 . Two numbers are such that the ratio between them is 3:5 but if each is increased by
10, the ratio between them becomes 5:7. Find
the numbers.
Let the numbers be 3 x and 5x

The sides of a triangle are in the ratio of

3 x 10 5

5 x 10 7

1 1 1
: :
. If the perimeter is 104 cms, find
2 3 4

Then

the length of the smallest side.

7 (3 x+10)=5(5 x+10) x = 5

Given ratio is

1 1 1
: : 6: 4:3
2 3 4

(Multiplying with the L.C.M. of


2,3, & 4)
Sum of ratio = 6+4+3= 13
3
de
x 1 0 4 2 4 cms.
Smallest side
13

5 . The incomes of A and B are in the ratio


2:3 and their expenditure are in the ratio 1:2,
If each saves Rs. 2,400, find A's income.
Let the income of A and B be 2x and 3x
Since, Income - Savings = Expenditure,
(2x - 2400) : (3 x - 2400) = 1:2

2 (2 x-2400) = 3 x - 2400
x = 2400
As income 2 x=2x2400
= Rs. 4800.
6 . In 40 litres mixture of milk and water,
the ratio of milk and water is 3:1. How much
water should be added in the mixture so that
the ratio of milk to water becomes 2:1?
In 40 litres of mixture, quantity of milk

The numbers are 15 and 25


8 . A bag contains rupees, fifty paise, and
twenty five paise coins in the proportion 5:6:8.
If the total amount is Rs. 210. Find the number of coins of each kind.
Ans: Let there be 5 rupee coins, 6 fifty paise
coins, and 8 twenty five paise coins the value
of 6 fifty paise coins
= Rs. 3
The value of 8 twenty five paise coins
= Rs. 2
The number of rupee coins
=

5 x2 1 0
105
10

The number of 50 paise coins


=

6 x2 1 0
126
10

The number of 25 paise coins


=

8 x2 1 0
168
10

PRACTICE TEST
1.

If A:B = 3:2 B:C= 4:3 then A:B:C=?


1) 6:4:3
2) 3:2:3
3) 3:4:3
4) 3:2:1
2 . Ratio between two numbers is 3:2 and
their difference is 225, then the smaller number is:
1) 9 0
2) 6 7 5
3) 1 3 5
4) 4 5 0
3 . If 2x=3y = 4z, then x : y : z is
1) 4:3:2
2) 6:3:4
3) 3:4:2
4) 6:4:3
4 . The mean proportion between 9 and 36
is
1) 2 2. 5
2) 1 8
3) 6
4) 3 6
5 . The fourth proportion to 3,6,15 is
1) 1 5
2) 3 0
3) 5
4) 1 8
6 . Two numbres are in the ratio 7:9. If 12
is subtracted from each of them, the ratio
becomes 3:5. The product of the numbers is:
1) 4 3 2
2) 5 6 7
3) 1 5 7 5
4) 1 2 6 3
7 . What must be added to each term of the
ratio 7:13 so that the ratio becomes 2:3?
1) 1
2) 2
3) 3
4) 5
8 . A total amount of Rs. 1800 is to be divided among A,B and C in such a way that
half of A's part, one third of B's part and onefourth of C's part is equal. The A's part is
1) Rs. 400
2) Rs. 600
3) Rs. 800
4) Rs. 900
9 . A sum of Rs. 53 is divided among A,B,C
in such a way that A gets Rs. 7 more than B
and B to gets Rs. 8 more than C. Then the
ratio of their shares is
1) 10: 18:25
2) 18: 25:10
3) 25: 18:10
4) 15: 18:20
1 0 . The ratio of number of boys and girls in a
school of 720 students is 7:5. How many more

girls should be admitted to make the ratio 1:1?


1) 9 0
2) 1 2 0
3) 2 2 0
4) 2 4 0
1 1 . The ratio of the number of boys and girls
at a party was 1:2 but when 2 boys and 2
girls left, the ratio became 1:3. then the number of persons initially in the party was
1) 2 4
2) 3 6
3) 1 2
4) 1 5
1 2 . A sum of Rs. 3400 has been divided
among A,B and C in such a way that A gets
2
3

of what B gets and B gets

1
of what C
4

gets. Then, B's share is


1) Rs. 600
2) Rs. 340
3) Rs. 400
4) Rs. 500
1 3 . Two numbers are in the ratio 3:5, If 8 is
subtracted from each, then they are in the ratio
1:3. Then, the second number is
1) 1 5
2) 2 0
3) 4
4) 1 2
1 4 . The proportion of copper and zinc in brass
is 13:7. How much zinc will be there in 100
kg of brass?
1) 20 kg
2) 35 kg
3) 45 kg
4) 50kg
1 5 . The ratio of the father's age to son's age
is 4:1. The product of their ages is 196. The
ratio of their ages after 5 years will be:
1) 3: 1
2) 10 : 3
3) 11 : 4
4) 14 : 5
1 6 . The ages of Manoj and Amit are in the
ratio 2:3. After 12 years, their ages will be in
the ratio 11:15. The age of Amit is:
1) 32 years
2) 40 years
3) 48 years
4) 56 years
1 7 . Rs. 780 is divided among 2 men, 6 women
and 8 boys so that the share of a man, a woman
and a boy are in the ratio 3:2:1. Then, how
much does a boy get?
1) Rs. 130
2) Rs. 60
3) Rs. 240
4) Rs. 40

1 8 . The ratio between the annual incomes of


A and B is 5:4 and between their expenditures
is 4:3. If at the end of the year, A and B respectively save Rs. 400 and Rs. 500, then the
income of A is:
1) Rs. 4,000
2) Rs. 3,200
3) Rs. 3,700
4) Rs. 4,800
1 9 . A bag contains one rupee, 50 paise and
25 paise coins in the ratio 5:7:9. If the total
amount in the bag is Rs. 430, find the number
of coins of 25 paise.
1) 2 0 0
2) 2 8 0
3) 3 6 0
4) 3 0 0
2 0 . A mixture contains milk and water in the
ratio 3:2. If 4 litres of water is added to the
mixture, milk and water in the mixture becomes
equal. The quantity of milk in the mixture in
litre is.
1) 1 8
2) 4
3) 6
4) 1 2
1
2
2 1 . Two equal glasses are
and
full of
2
3

milk respectively. The two are completely filled


up with water. The contents of the two glasses
are then mixed in another vessel. The ratio of
milk and water in the vessel is
1) 5: 7
2) 7: 5
3) 1: 1
4) 2: 3
2 2 . An amount is to be distributed among A,B
and C in the ratio 3:7:5 respectively. If the
difference in the shares of A and B is Rs. 7,600/
- what will be the share of C?
1) Rs. 5,700
2) Rs. 19,000
3) Rs. 9,500
4) Rs. 10,000
2 3 . Two varieties of oil are mixed in the ratio
4:3 to produce first quality and if they are
mixed in the ratio 2:3 second quality is obtained. How many kg. of the first quality be
mixed with 10kg of the second quality so that
a third quality having the two verieties in the
ratio 5 : 4 may be produced?
1) 48 kg
2) 42 kg
3) 88 kg
4) 98 kg

2 4 . The ratio of the number of gents to ladies in a party was 2:3. When 20 more gents
joined the group, the ratio was reversed. The
number of ladies in the party was
1)

16

2) 2 4

3) 3 0

4) 3 6

2 5 . The HCF of three numbers is 12. If they


are in the ratio of 1:2:3, the numbers are
1)

12, 24, 36

2) 10, 20, 30

3)

5, 10, 15

4) 4, 8, 12

2 6 . If the ratio of the areas of two squares is


1:4, the ratio of their perimeters is
1)

1: 2

2) 1: 4

3)

1: 6

4) 1: 8

2 7 . Two numbers are such that their difference, their sum and their product are in the
ratio of 1:7:24. The product of the numbers
is
1)

2) 1 2

3) 2 4

4) 4 8

2 8 . The incomes of A, B and C are in the ratio 7:9:12 and their spending are in the ratio
8:9:15. If A saves

1
th of his income, then
4

the savings of A,B and C are in the ratio


of
1)

56: 99:69

2) 99: 56:69

3)

69: 56:99

4) 99: 69:56

2 9 . Rs. 180 contained in a box is made up of


one rupee, 50 paise, and 25 paise coins in the
proportion of 2:3:4. What is the number of 50
paise coins?

30.
half
part
The

1)

150

2) 1 8 0

3)

240

4) 120

81 is divided into three parts, such that


of the first part, one-third of the second
and one-fourth of the third part are equal.
third part is more than the first by
1)

2) 1 8

3)

27

4) 3 6

3 1 . 94 is divided into two parts in such a way


that the fifth part of the first and eight parts

of the second are in the ratio 3:4. The first


part is.
1) 2
2) 3 0
3) 3 6
4) 4 8
3 2 . If a carton containing a dozen mirrors is
dropped, which of the following cannot be the
ratio of broken mirros to unbroken mirros?
1) 2: 1
2) 3: 1
3) 3: 2
3) 7: 5
3 3 . Two-third of Reya's money is equal to
one-fifth of Sobha's ,money. What is the ratio
between their share of money?
1) 2: 5
2) 3: 10
3) 4: 5
4) 10 : 3
3 4 . The ratio of number of boys and girls in a
school is 3:2. If 20% of the boys and 30% of
the girls are scholarship holders, the percentage of students who do not get scholarship is
1) 5 0
2) 7 2
3) 7 5
4) 7 6
3 5 . 20 litres of a mixture contains milk and
water in the ratio of 5:3. If 4 litres of this
mixture is replaced by 4 litres of milk, the ratio of milk to water in the new mixture will be
1) 2: 1
2) 5: 3
3) 7: 3
4) 7: 4
3 6 . A sum of Rs. 427 is to be divided among
A,B and C in such a way that 3 time A's share,
4 times B's share and 7 times C's share are all
equal. The share of C is
1) Rs. 84
2) Rs. 147
3) Rs. 196
4) Rs. 240
3 7 . An employer reduces the number of his
employees in the ratio of 9:8 and increases
their wages in the ratio 14:15. in what ratio

the wages bill is increased or decreased.


1) 21: 20
2) 20: 21
3) 25: 26
4) 26: 25
3 8 . A and B are two alloys of gold and copper prepared by mixing metals in the ratio 7:2
and 7:11 respectively. If equal quantities of
alloys are melted to form a third alloy C. Find
the ratio of gold and copper in C.
1) 5: 7
2) 7: 5
3) 6: 5
4) 5: 6
3 9 . A shopkeeper mixes two kinds of flour,
one costing Rs. 3.50 per kg and the other Rs.
2.25 per kg. so that the price of the mixture is
Rs. 2.75 per kg. The ratio of first kind of flour
to that of the second must be
1) 3: 4
2) 4: 3
3) 3: 2
4) 2: 3
4 0 . In what ratio must 25% alcohol be mixed
with 60% alcohol to get a mixture of 40%
alcohol strength?
1) 1: 2
2) 2: 1
3) 4: 3
4) 3: 2
4 1 . In what proportion must water be mixed
2
3

with spirit to gain 16 % % by selling it at cost


price?
1) 6: 1
2) 2: 1
3) 1: 2
4) 1: 6
4 2 . 15 litres of a mixture contains 20% alcohol and the rest water. If 3 litres of water
be mixed in it, the percentage of alcohol in the
new mixture will be
2
1) 1 7
2) 1 6
3
1
3) 1 8
4) 1 5
2

ANSWERS TO PRACTICE TEST


1.

(1)

2.

(4)

9.

(3)

1 0 . (2)

1 7 . (2)

3.

(4)

4.

(2)

5 . (2)

6.

(2)

7.

(4)

8.(1)

1 1 . (3)

1 2 . (1)

13. (2)

1 4 . (2)

1 5 . (3)

16.(3)

1 8 . (1)

1 9 . (3)

2 0 . (4)

21.(2)

2 2 . (3)

2 3 . (4)

24.(2)

2 5 . (1)

2 6 . (1)

2 7 . (4)

2 8 . (1)

29.(4)

3 0 . (2)

3 1 . (2)

3 2 . (3)

3 3 . (2)

3 4 . (4)

3 5 . (3)

3 6 . (1)

37.(1)

3 8 . (2)

39. (4)

40.(3)

41 (4)

4 2 (2)

BANK PROBATIONARY OFFICER


QUANTITATIVE APTITUDE
PARTNERSHIP

Sometimes two or more than two persons agree to run a business jointly. They are
called partners and the deal is known as partnership.
1 . Simple partnership: When capitals of all
the persons are invested for the same time
intervals, the gain or loss in the business is divided among the partners in the ratio of their
investments.
2 . Compound partnership: When capitals of
partners are invested for different time intervals then equivalent capitals are obtained for
1 unit of time by multiplying the capital with
the number of units. The gain or loss is now
divided among partners in the ratio of these
capitals.

Divide the net profit of Rs. 2,40, 000 among


the partners.
Ans: Ratio of investment
= 3,80,000:4,00,000 : 4,20,000
= 19:20:21
Ratio of profit = 19 : 20 : 21
Vinod's share=Rs. 2,40,000 x

19
60

=Rs. 76,000
Ramesh's share = Rs. 2,40,000x

20
60

= Rs. 80,000
Arun's share = Rs. 2,40,000x

Formula for compound partnership is

21
60

= Rs. 84,000
A' s Capital x A' s time in partnership
B' s capital x B' s time in partnership

A' s profit
B' s profit

Solved Examples
1 . Three partners A, B and C agree to divide the profit or loss in the ratio 1.50 : 1.75 :
2.25. In a particular year, they earn a profit of
Rs. 66,000. Find the share of each.
Ans: Ratio of profits
=1.50:1.75:2.25 =150:175:225 = 6:7:9
A's share = Rs. 66,000 x

6
= Rs. 18,000
22

B's share = Rs. 66,000 x

7
= Rs. 21,000
22

C's share = Rs. 66,000 x

9
= Rs. 27,000
22

2 . Vinod, Ramesh and Arun started a business jointly by investing Rs. 3,80,000,
Rs. 4,00,000 and Rs. 4,20, 000 respectively.

3 . A,B and C enter into a partnership. A contributes Rs. 3,20,000 for 4 months, B contributes Rs. 5,10,000 for 3 months and C contributes Rs. 2,70,000 for 5 months. If the total profit be Rs. 1,24,800, whast is C's share?
Ratio of capitals of A,B &C
=

(3,20,000x4):(5,10,000x3): (2,70,000x5)

12,80,000 : 15,30,000 : 13,50,000

128 : 153 : 135

C's share=1,24,800x

135
=Rs. 40,500
416

4 . A,B and C enter into partnership . A invests some money at the beginning. B invests
double the amount after 6 months and C invests thrice the amount after 8 months. If the
annual profit is Rs. 18,000, what is C's share?
Ans: Suppose the investment of A is Rs. X
Ratio of investment of A, B, and C

= (12x X) : (6x2 X) : (4x3 X) = 1:1:1

1
C's share = Rs. 18,000 x
=Rs. 6,000
3
5 . A and B enter into a partnership. A invests Rs. 16,000 for 8 months and B remains
in the business for 4 months. Out of a total
profit, B claims

2
of the profit. Find the con7

tribution of B.
Ans:
A' s capital x A' s timeinpartnership A' s capital

B' scapitalx B' stimeinpartnership B' scapital

16,000 x 8

ie.

B' s Capit alx 4

5/7
2 /7

16,000 x 8 x2
Rs.12,800
4 x5
6 . A and B enter into a speculation. A puts
in Rs. 50 and B puts in Rs. 45. At the end of 4
months A withdraws half his capital and at
the end of 6 months B withdraws half of his
capital, C then enters with a capital of
Rs. 70. At the end of 12 months in what ratio
will the profit be divideed?
Ans: A's share : B's share : C's share

B's capital =

50
45
x 8 ):(45x6+
x 6 ) : 70x6
2
2
= 400 : 405 : 420 = 80 : 81 : 84
7 . A,B and C enter into a partnership and
their capitals are in the proportion of
= (50x4+

1 1 1
: : . A withdraws half of his capital at
3 4 5
the end of 4 months. Out of a total annual
profit of Rs. 847, A's share is:
Ans: Ratio of capitals in the beginning

1 1 1
: : 20 :15 :12
3 4 5
Ratio of investment for the whole year
= (20x4+10x8) : (15x12) : (12x12)
= 40 : 45 : 36
=

A's share = 847 x

40
= Rs. 280
121

PRACTICE TEST
1 . In business A, B and C invested Rs. 380,
Rs. 400 and Rs. 420 respectively. What is the
share of A if they got Rs. 180 as profit.
1) Rs. 57
2) Rs. 60
3) Rs. 63
3) Rs. 62
2 . A started a business with a capital of Rs.
10,000 and 4 months later, B joined him with
a capital of Rs. 5,000. What is the share of A
of a total profit of Rs. 2,000 at the end of the
year ?
1) Rs. 2,000
2) Rs. 1,600
3) Rs. 1,500
4) Rs. 1,000
3 . A and B together invested Rs. 12,000 in
a business. At the end of the year, out of a
total profit of Rs. 1,800, A's share was
Rs.
750. What was the investment of A?
1) Rs. 5,000
2) Rs. 4,000
3) Rs. 3,500
4) Rs. 5,300
4 . Rs. 750 is distributed among A, B and C
such that A's share : B's share = 2:3 and B's
share: C's share = 6:5. The share of A is
1) Rs. 150
2) Rs. 175
3) Rs. 200
4) Rs. 250
5 . A profit of Rs. 450 is divided between
two partners, one of whom has contributed
Rs. 1200 for 5 months and the other Rs. 750
for 4 months. How much amount did the second partner receive?
1) Rs. 300
2) Rs. 425
3) Rs. 150
4) Rs. 175
6 . A, B and C can do a work in 20, 25 and
30 days respectively. They undertook to finish the work together for Rs. 2,220. Then the
share of A exceeds that of B by:
1) Rs. 120
2) Rs. 180
3) Rs. 300
4) Rs. 600
7 . Three partners A, B and C invest
Rs. 13,000, Rs, 17,000 and Rs. 5,000
respetively in a business. They have a profit
of Rs. 1,750. B's share of profit is:
1) Rs. 650
2) Rs. 850
3) Rs. 250
4) Rs. 750

8 . A starts a business with Rs. 30,000 and


4 months after B joins. If at the end of the
year, the profits are divided by A and B in the
proportion of 9:4, B's capital was:
1) Rs. 20,000
2) Rs. 25,000
3) Rs. 20,700
4) Rs. 18,000
9 . A invests Rs. 3,000 for one year in a
business. How much should B invest in order
that the profit after 1 year may be divided in
the ratio 2:3?
1) Rs. 2,000
2) Rs. 1,800
3) Rs. 3,600
4) Rs. 4,500
1 0 . A and B start a business with initial investments in the ratio 12:11 and their annual
profits are in the ratio 4:1. If A invested the
money for 11 months, B invested the money
for:
1)

3 months

2) 3

2
months
3

3) 4 months
4) 6 months
1 1 . Anoop and Manoj invested Rs. 15,000 and
Rs. 20,000 respectively and opened a shop.
After one year they had a profit of
Rs.
23,100. How much more will Manoj get than
Anoop?
1) Rs. 3,300
2) Rs. 9,000
3) Rs. 13,200
4) Rs. 5,000
1 2 . A and B enter into a partnership and invest Rs. 1,20,000 and Rs. 2,00,000 respectively. At the end of two years, they make a

profit of Rs. 64,000. What is A's share in the


profit?
1) Rs. 24,000
2) Rs. 25,000
3) Rs. 40,000
4) Rs. 32,000
1 3 . A, B and C enter into a partership. A initially invests Rs. 25 lakhs and adds another
Rs. 10 lakhs after one year. B initially invests
Rs. 35 lakhs and withdraws Rs. 10 lakhs after
two years and C invests Rs. 30 lakhs. In what
ratio should the profits be divided at the end
of three years?
1) 20: 19:18
2) 19: 19:18
3) 20: 20:19
4) Data inadequate
1 4 . A and B entered into a partnership investing Rs. 16,000 and Rs. 12,000 respetively.
After 3months A withdrew Rs. 5,000 while B
invested Rs. 5,000 more. After three months
C joins the business with a capital of Rs.
21,000. The share of B exceeds that of C, out
of a total profit of Rs. 26,400 after one year
by
1) Rs. 1,200
2) Rs. 2,400
3) Rs. 3,600
4) Rs. 4,800
1 5 . Madhu and Sunu entered into a partnership by investing Rs. 12,000 and Rs. 9,000
respectively. After 3 months Sony joined them
with an investment of Rs. 15,000. What is
the share of Sony in a half yearly profit of Rs.
9, 5 00?
1) Rs. 3,500
2) Rs. 3,000
3) Rs. 2,500
4) Rs. 4,000

ANSWERS TO PRACTICE TEST


1.

(1)

2.

(3)

9.

(4)

10. (1)

3.

(1)

11. (1)

4.

(3)

12. (1)

5.

(3)

13. (2)

6.

(2)

14. (3)

7.

(2)

15. (3)

8.(1)

Bank Probationary Officer

TIME & DISTANCE

Quantitative Aptitude

Important Formulae
1.

Speed =

2.

Time =

3.

Dis t an ce
Time

and ii)

they walk in the same direction

Ans : i)

If they walk in opposite directions,


they will be 8+10 = 18 km apart in
one hour.

Dis t an ce
Speed
Distance = Speed x Time

in 3

4 . If the speed of a body is changed in the


ratio a:b, the time taken changes in the ratio
b:a.

18 x
ii)

5 . If a certain distance is covered at x km/hr


and the same distance is covered at y km/hr
then the average speed during the whole jour-

6.

To convert a speed in metres per second

to km per hour, multiply it by

18
5

Solved Examples

2 x15 x10
12km / hr
10 15

2 . Two men starting from the same place


walk at the rate of 10km/hr and 8 km/hr respectively. How many km. will they be apart
at the end of 3
i)

1
7
hrs, they will be 2 x 7 km
2
2

3 . A train travelling 25 km/hr leaves Delhi


at 9 am and another train travelling 35 km/hr
starts at 2 pm in the same direction. How
many km. from Delhi will they be together.
Ans: Let the two trains be together x hrs after
9 am.
Distance travelled by the first train = 25 x km
Distance travelled by the second train

1 . A and B are two cities. A man travels


from A to B at a speed of 15 km/hr. and returns back at the rate of 10km/hr. Find his
average speed for the whole journey.
Ans : Average speed =

If they walk in the same direction,


their they are 10-8=2 km. apart in
one hour.

apart.

To convert a speed in km.per hour to

5
metres per second, multiply it by
18

7.

7
63 km. apart.
2

in 3

2 xy
km / hr
ney is
x y

1
hrs, they will be
2

1
hrs, if
2

they walk in opposite directions.

= 35 (x-5) km
ie 25x = 35 (x-5) x = 17.5 hrs.
Required distance = 25x17.5 = 437.5 km.
Short cut:
Required distance =
Pr oduct of speed
x Dif ferencein time
Dif ferenceof speed

25 x 35 x5
25 x 35 x5

437.5 km
35 25
10

4 . Two towns P and Q are 110 km. apart.


A motorcycle rider starts from P towards Q
at 8 am at a speed of 20 km/hr. Another motor cycle rider starts from Q towards P at
9
am at a speed of 25 km/hr. Find when will
they cross each other.
Ans : - Suppose they meet x hours after 8 am
Then 20x + ( x-1) 25 = 110 x= 3 hrs.
two riders meet at 8+3 = 11 am.

5 . A boy goes to school at a speed of 3km/hr


and returns to the village at a speed of
2km/
hr. If he takes 5 hrs in all, what is the distance
between the village and the school.
Ans:- Let x km be the distance between the
village and the school.
Then

x x
5 x 6km
3 2

6 . Joshy travels at the rate of 3km/hr and


he reaches his office 15 mts late. If he travels
at the rate of 4km/hr. he reaches 15 mts earlier. The distance Joshy has to travel is
Ans: Let x km be the distance that Joshy has
to travel.
Then

x x 15 15

3 4 60

x = 6 km
3
7 . By walking at
of his usual speed a man
4
reaches his office 20 mts later than usual time.
His usual time is
3
Ans : New Speed =
usual speed
4
4
usual time
3
= usual time + 20 mts

New time =

4
ie
usual time - usual time = 20 mts
3
1
usual time = 20 mts
3
usual time = 3x20 = 60 mts

PRACTICE TEST
1 . A man crosses a street 600m long in 5
minutes. What is his speed in Kilometres per
hours?
1)

9km/hr.

2) 7km/hr.

3)

9.6 km/hr.

4) 7.2 km/hr.

2 . A man goes to a place at the rate of


13 km/hr. He comes back on a bicycle at
7 km/hr. His average speed for the entire journey is:
1)

5 km/hr.

2) 6.5 km/hr.

3)

8.2 km/hr.

4) 9.1 km/hr.

3 . A person walks at 5 kmph. and reaches


his destination 10 minutes late. If he increases
his speed by 1 kmph., he would have reached
15 minutes earlier. The distance he travelled
from his house is
1)

10 km.

2) 5.5 km

3)

12.5 km.

4) 10.5 km

4 . A and B start from the same place in opposite directions with 25 kmph. and 30 kmph.
respectively. In what time will they be
110 km apart?
1)

2 hrs 5 min.

2) 5 hours

3)

3 hrs. 20 min.

4) 2 hours

5 . A motor cyclist travels for 10 hours, the


first half at 21 km/hr. and the other half at 24
km/hr. Find the distance travelled.
1)

225 km

2) 224 km

3)

200 km

4) 324 km

6 . Two towns A and B are 160 km apart. A


bus starts from A to B at 7 A.M at a speed of
50 kmph. Another bus starts from B to A at 8
A.M at a speed of 60 kmph. The time of their
meeting is:
1)

9 A.M.

2) 9.30 A.M

3)

10. A.M.

4) 8.30 A.M.

7 . Two trains starts at the same time from


places A and B and proceed towards each other
at 72 km/hr. and 60 km/hr. respectively. At

the time of their meeting, one train has travelled 48 km more than the other. Then the
distance between A and B is

time of their meeting the second train has travelled 120 km more than the first. The distance
between A and B is

1)

132 km.

2) 704 km

1)

990 km

2) 1,200 km

3)

470 km

4) 528 km

3)

1,320 km

4) 1,440 km

8 . Two men start together to walk a certain distance at 3.75 km./hr. and 3 km./hr.
respectively. The former arrives 30 mts before the latter. Find the distance they walked.

1 4 . A walks at 4 km an hour, and 4 hrs after


his start, B bicycles after him at 10 km an
hour. Find out how far from the start does B
catch up with A.

1)

7.5 km

2) 10 km

1)

16.7 km

2) 18.6 km

3)

12.5 km

4) 15 km

3)

21.5 km

4) 26.7 km

9 . A car went 52 km in the first hour, 60


km in the second hour and 54 km in the third
hour. There was trouble in the fourth hour with
the result that its speed was 26 km. Its average speed was
1)

51 km/hr

2) 57 km/hr

3)

45 km/hr

4) 48 km/hr

1 0 . Travelling at a uniform speed, a car covers a distance of 6 km. in 4 mts. what is the
speed of the car in km.hr?
1)

90

2) 4 0

3) 2 4

4) 6 0

1 1 . A car completes a journey in 11 hrs. It


covers the first half of the journey at the rate
of 50 km/hr. and the second half at the rate
of 60 km/hr. The distance of total journey is
1)

605 km

2) 300 km

3)

500 km

4) 600 km

1 2 . A student walks from his house


@5 km/hr. and thus reaches the school 10 mts
late. If his speed had been 6 km/hr. he would
have reached 15 mts earlier. Then the distance
of the school from his house is
1)

5
km
2

2)

5
km
22

3)

25
km
2

4)

25
km
22

1 3 . Two trains start from stations A and B


and travel towards each other at a speed of
50 km/hr. and 60 km/hr. respectively. At the

1 5 . Mohan rides a cycle at 8 km/hr. After


every 10 km he rests for 20 mts. He will cover
40 km in
1)

5 hrs

2) 5 hrs 20 mts

3)

6 hrs

4) 6 hrs 20 mts

2
, he takes
3
1 hr more to walk a certain distance. The time
(in hours) to cover the distance with his normal speed is

1 6 . If a man reduces his speed to

1)

2) 1

3)

4) 1 . 5

1 7 . Sound travels at 330 m a second. How


many kilometres away is a thunder cloud when
its sound follows the flash after 10 seconds?
1)

3.3

2) 3 3

3)

0.33

4) 3 . 3 3

1 8 . Car A moves 175 km in 8 hrs whereas


car B moves 189 km in 12 hrs. The ratio between the speed of car A and car B is
1)

3: 2

2) 4: 3

3)

5: 4

4) 25: 18

1 9 . If a boy walks from his house to the school


@ 4 km/hr. he reaches the school 10 mts earlier than the school time. However, if he walks
@ 3 km/hr, he reaches 10mts late. The distance of school from his house is
1)

6 km

2) 4.5 km

3)

4 km

4) 3 km

2 0 . A man walks a certain distance at


8 km/hr. and returns at 6 km/hr. If the total
1
time taken by him is 3
hrs, the total dis2
tance walked is

1)

28 km

2) 24 km

3)

14 km

4) 16 km

2 1 . A cart runs at the rate of 4 km/hr. during


the first 10 km and 9 km/hr. during the second 10 km. The average speed of the cart in
km/hr. is
1)

5.0

2) 5 . 5

3) 6 . 0

4) 6 . 5

2 2 . A car starting from airport reaches the


bus station in 45 mts with an average speed
of 40 km/hr. If the speed of the car is increased by 10 km/hr. how much less time will

the car take to cover the distance?


1)

19 mts

2) 10 mts

3)

9 mts

4) 24 mts

2 3 . A thief steals a car at 1.30 pm and drives


it at 40 km/hr. The theft is discovered at 2 pm
and the owner sets off in another car at 50
km/hr. He will overtake the thief at
1)

3.30 pm

2) 4 pm

3)

4.30 pm

4) 6 pm

24. A car covers a distance of 715 km at a


constant speed. If the speed of the car had
been 10 km/hr. more, then it would have taken
2 hrs. less to cover the same distance. What
was the original speed of the car in km/hr?
1)

45

2) 5 0

3)

55

4) 6 5

ANSWERS TO PRACTICE TEST


1.

(4)

2.

(4)

9.

(4)

1 0 . (1)

1 7 . (1)

1 8 . (4)

3.

(3)

4.

(4)

5 . (2)

6.

(1)

7.

(4)

8.(1)

1 1 . (4)

1 2 . (3)

13. (3)

1 4 . (4)

1 5 . (3)

16.(1)

1 9 . (3)

2 0 . (2)

21.(2)

2 2 . (3)

2 3 . (2)

2 4 . (3)

Bank Probationary Officer


Quantitative Aptitude

P R O B LE M S O N TR A I N S

In solving problems on trains, the following points should be kept in mind.


Time taken by a train to cross a pole (or Signal
post or, a standing man)
=

Length of thetrain Length of thebridge


Speedof thetrain

Time taken by a train to cross a moving object


a)

L1 L2
In the opposite direction = V V
1
2

b)

L1 L2
In the same direction = V V
1
2

where L1 and V1 are the length and speed of


the train and L2 and V2 are the length and speed
of the moving object.

80 120
5
35 25 x

Speed of the train = 54 x

5
m/s
18

= 15 m/s
Time taken to cross the pole

300
20 seconds
15

A train 220 m long is running with a speed

200 x18
10 x5

18

= 72 seconds
4 . Two trains running in the same direction
at 40 km/hr. and 22 km/hr. completely pass
one another in 1 minute. If the length of the
first train is 125m., find the length of the second train.
Ans: Let the length of the second train be x
125 x
(40 22 )x

1 . A train 300 m long is running with a speed


of 54 km/hr. In what time will it cross a telephone pole?

2.

Ans: Time taken =

Then,

Solved Examples

220 x 18

12 seconds
5
66 x 5
(60 6) x
18
3 . Two trains 80 metres and 120 metres
long are running at the rates of 25 km/hr. and
35 km/hr. respectively on parallel rails. If they
are moving in same directions, how long will
they take to pass each other?
220

Lengt h of the train


Speed of the train

Time taken by a train to cross a bridge (or


tunnel or a train at rest or a platform)
=

of 60 km/hr. In what time will it pass a man


who is running at 6 km/hr in the direction opposite to that in which the train is going?
Ans: Time taken to pass the man

5
18

60

(125x)18
60 125 x 5 x60
18 x5

x = 300 125 175m


5 . A train 280m long is moving at a speed
of 60 km/hr. The time taken by the train to
cross a platform 220m long is
Ans : Time taken to cross the platform
=

280 220 500 x18 30 sec


60 x

5
18

60 x5

PRACTICE TEST
1 . How long will it take for a train 120 m
long running at 40 km/hr. to cross a bridge 80
metres long?
1)

18 min

2) 18 sec.

3)

20 sec

4) 19 sec.

2 . A train running at 50 m/s takes 30 seconds to cross a platform of 800m. long. The
length of the train in metres is
1)

900

2) 3 0 0

3)

550

4) 7 0 0

3 . A person standing on a platform of length


200 metres observed that a train takes 10
seconds to pass him, passed completely
through the platform in 30 seconds. Then the
length of the train is
1)

100 metres

2) 150met r es

3)

80 metres

4) 105 metres

4 . A train running at uniform speed takes


54 seconds to pass a platform of 324 m. long
and 45 seconds to pass another platform of
240m. long. The length of the train in metres
is
1)

180

2) 1 7 0

3)

190

4) 1 0 0

5 . A train 300 m long is running at a speed


of 68 km/hr. It will cross a man coming from
the opposite direction at a speed of 4km/hr.
in
1)

30 sec.

2) 15 sec.

3)

25 sec

4) 21.6 sec

6 . A train 240 metres long, takes 24 seconds to cross a man running at 10 km/hr in
the same direction as the train. The speed of
the train is
1)

32 km/hr

2) 46 km/hr

3)

38 km/hr

4) 24 km/hr

7 . A train crosses a platform in 60 seconds


travelling at a speed of 54 km/hr. If the length
of the platform is 500 metres, what is the

length of the train?


1)

40 0m

2) 35 0m

3)

450 m

4) 50 0m

8 . A man running at 18 km/hr crosses a


bridge in 2 minutes. The length of the bridge
is
1)

1.2 km

2) 0.6 km

3)

1 km

4) 3.6 km

9 . A railway passenger counts the telegraph


posts on the line as he passes them. If they
are 50 m apart from each other and the train
is running at the rate of 48 km/hr. how many
posts will he pass per minute?
1)

16

2) 1 7

3)

18

4) 1 3

1 0 . Two trains are running at 40 km/hr. and


20 km/hr. respectively in the same direction.
The faster train completely passes a man sitting in the slower train in 5 seconds. What is
the length of the faster train?
1)

23

2
m
9

2) 27 m

3)

27

7
m
9

4) 23 m

1 1 . A train 150 m long running at a speed of


60 km/hr. takes 30 seconds to cross a bridge.
The length of the bridge is
1)

3,500 m

2) 500 m

3)

200 m

4) 35 0m

1 2 . A train moving with a uniform speed of


60 km/hr. crosses a pole in 6 seconds. The
length of the train (in metres) is
1)

200

2) 1 5 0

3)

120

4) 1 0 0

1 3 . The time taken by a train 180 m long,


travelling at 42 km/hr. to pass a man walking
in the same direction at 6 km/hr. will be
1)

18 seconds

2) 21 seconds

3)

24 seconds

4) 25 seconds

1 4 . Two trains whose lengths 180 m and


220m respectively are running in directions opposite to one another with respective speeds
of 40 kmph and 50 kmph. Time taken by them
to cross one another will be
1)

16 seconds

2) 17 seconds

3)

18 seconds

4) 22 seconds

1 5 . Two trains running in the same direction


at 40 km/hr. and 22 km/hr. completely pass
one another in one minute. If the length of the
first train is 125 m, the length of the second
train is
1)

125 m

2) 15 0m

3)

175 m

4) 20 0m

1 6 . A train takes 18 seconds to pass completely through a station 162 m long and 15
seconds through another station 120 m long.
The length of the train is
1) 70 m

2) 80 m

3) 90 m 4) 100 m

1 7 . A man standing on a railway platform


observes that a train going in one direction
takes 4 seconds to pass him. Another train of
same length going in the opposite direction
takes 5 seconds to pass him. The time taken
by the two trains to cross each other will be
1)

31 sec
9

2) 40 9 sec

3)

49

4) 50 9 sec

9 sec

1 8 . A train 120 m long is moving at a speed


of 54 km/hr. The time taken to pass a bridge
180 m long is
1)

12 seconds

2) 18 seconds

3)

30 seconds

4) 20 seconds

1 9 . A train crosses a pole in 15 seconds,


while it crosses 100 m long platform in 25
seconds. The length of the train is
1)

125 m

2) 135 m

3)

150 m

4) 175 m

ANSWERS TO PRACTICE TEST


1.

(2)

2.

(4)

3.

(1)

9.

(2)

1 0 . (3)

1 1 . (4)

1 7 . (2)

1 8 . (4)

1 9 . (3)

4.

(1)

5 . (2)

6.

(2)

1 2 . (4)

13. (1)

1 4 . (1)

7.

(1)

1 5 . (3)

8.(2)
16.(3)

BANK PROBATIONARY OFFICER


QUANTITATIVE APTITUDE
BOAT AND STREAMS
Important Formulae:

Ans: Let man's rate in upstream = x km/hr.

1 . If the speed of a boat in still water be x


km/hr and that of stream be y km/hr
a)

Speed of the boat downstream

man's rate downstream = 2 x km/hr.


Man's rate in still water=

= ( x+y) km/hr
b)

Speed of the boat upstream


= ( x-y) km/hr

2 . If the speed of a boat downstream is


u km/hr and speed upstream is v km/hr
a)

Speed of the boat in still water


=

b)

1
(u v)km
km/hr
2

Speed of the current


=

1
(u v) km/hr
2

3 . If a man rows in still water at x km/hr


and the rate of current (or stream) is y km/hr,
a)

Man's rate with the current


= ( x+y)km/hr

b)

Man's rate against the current


= ( x-y)km/hr

ie,

3x
6x 4
2

Rate upstream = 4 km/hr.

& Rate downstream = 8 km/hr.


1
(8-4) km/hr.
2
= 2 km/hr.

Rate of current =

3 . A boatman can row 3 km against the


stream in 45 minutes and return in 30 minutes. Find the rate of his rowing in still water
and also the speed of the stream.
Ans: Let the speed of the boatman in still water be x km/hr and the speed of the stream be
y km/hr.
Time taken to row against the stream
45 3

hr.
60 4
Time taken to row with the stream

Solved Examples
1 . A man can row upstreams at 6 km/hr and
down streams at 11 km/hr. Find man's rate in
still water and the rate of the current.
1
. km/hr.
Ans: Rate in still water= (6 11) 85
2

Rate of current =

1
(11 6) 2.5 km/hr.
2

2 . A man can row 9 km/hr in still water. It


takes him twice as long to row up as to row
down the river. Find the rate of stream.

1
( x+2 x)km/hr.
2

30 1

hr.
60 2

Speed against current= x-y =

Dis t an ce
Time

3
= 3
= 4 km/hr.
4

Speed with current = x+y =

3
= 1 = 6 km / hr
2

Dis t an ce
Time

hours to row to a place and back. Then the


distance of the place is

x+y = 6 & x-y = 4


x=5 km/hr. & y = 1 km/hr

PRACTICE TEST
1 . A person can row down stream at
12 kmph. and upstream in 6 kmph. Then, the
persons' rate in still water is:
1)

9 kmph

2) 5 kmph

3)

12 kmph

4) 6 kmph

2 . If a person's rate down the current is


11
km/hr. and the rate of the current is 3 km/hr.,
then the person's rate against the current (in
km/hr.) is
1)

7 kmph

2) 8 kmph

3)

5 kmph

4) 6 kmph

1)

14 kmph

2) 20 kmph

3)

17 kmph

4) 3 kmph

4 . The downstream speed of a boat is


15 kmph. and the speed of the stream is 2
kmph. Then the upstream speed of the boat is
1)

6.5 kmph

2) 13 kmph

3)

11 kmph

4) 12 kmph

5 . The speed of a boat in still water is


16 kmph. and the rate of the current is 2 kmph.
The distance travelled down stream in 15 minutes is:
4 km

2) 4.5 km

3)

6 km

4) 8.5 km

2 km.

2) 1.5 km

3)

3 km

4) 4.5 km

7 . A boat takes 9 hrs to travel from A to B


upstream. If the river current is 3 km/hr, how
long will it take to travel downstream between
B and A?
1)

3 hrs

2) 6 hrs

3)

4.5 hrs

4) 5 hrs

8 . A boat goes 11 km in an hour with the


stream and 5 km in an hour against the stream.
The speed of the boat (in km/hr.) in still water
is
1)

3 . A person can row upstream 12 km/hr.


and down stream at 18 km/hr. Then, the rate
of the current is

1)

1)

6 . A man can row 6 kmph. in still water. If


the river is running at 2 kmph, it takes him 3

2) 6

3) 8

4) 9

9 . A man rows upstream 13 km and down


stream 28 km taking 5 hrs each time. The
velocity of the current in km/hr. is
1)

0.5

2) 1

3) 1 . 5

4) 2

1 0 . A boat moves down stream at the rate


of 1 km in 7.5 mts and upstream at the rate
of 5 km/hr. The speed of the boat (in km/hr.)
in still water is
1)

2) 6 . 5

3) 7

4) 7 . 5

1 1 . A boat can be rowed 9 km upstream or


18 km downstream in a period of 3 hrs. What
is the speed of the boat in still water in kmph?
1)

1.5

2) 3

3) 4 . 5

4) 6

1 2 . A boatman can row to a place 48 km


distant and back, in 14 hrs. If he can row
4 km with the stream in the same time as
3 km against it, the speed of the stream (in
km/hr) is
1) 1

2) 2.2

3) 3

4) 0 . 5

ANSWERS TO PRACTICE TEST


1.

(1)

2.

(3)

9.

(3)

1 0 . (2)

3.

(4)

1 1 . (3)

4.

(3)

1 2 . (1)

5 . (2)

6.

(1)

7.

(4)

8.(3)

Bank Probationary officer

T IM E AN D W OR K

Quantitative Aptitude

Points to remember
1.

If A can finish a piece of work in `n' days,

then A's 1 day's work is

1 .
n

ab
(a b)

Solved Examples:

2 . If the number of men engaged to do a


piece of work is changed in the ratio a:b, the
time required for the work will be changed in
the ratio b:a
3.

and the entire tank is filled in 1 1


2
b
hours.

If A is X times as good a workman as B,

then A will take 1 x of the time that B takes


to do a certain work.

1 . 8 boys can arrange all the books of school


library in 12 days. In how many days can 6
boys arrange them?
Ans: M1 D1 = M2 D2
D2

8 x12
= 16 days
6

4 . If M 1 persons can do `W 1' works in D 1


days for T 1 hours and M2 persons can do `W2'
works in D 2 days for T 2 hours then
M1 D1 T 1 W2 = M2 D2 T 2W1.

2 . A can do a piece of work in 12 days and


B alone can do it in 15 days. How much time
will both take to finish the work?

5 . If A can finish a work in ` x' days and B


can finish the same work in `y' days, then

Ans: A's 1 day's work = 112

time taken by both to finish the work is

xy
x y

days

(A+B)'s 1 day's work =

6 . If A and B together can do a piece of


work in x days and A alone can do it in y days
then B alone can do it in

xy
days
y z

7 . If A, B and C can do a work in x,y, and z


days respectively, then all of them working
xyz
together can finish the work in
xy yz xz

days
8 . If two taps A and B take a and b hours
resepectively to fill a tank, then the two taps
together fill

B's 1 day's work = 115

1 1

part of the tank in an hour


a b

1
1
3

12 15 20

Both together can finish the work in

20
2
or 6
days
3
3
Using formula :
Time taken to finish the work
=

xy
12 x15

x y 12 15

12 x15 20
2

or 6
days
27
3
3

3 . A and B together can do a piece of work


in 12 days. B alone can finish it in 30 days. In

how many days can A alone finish the work?


Ans: (A+B)'s 1 day's work =
B's 1 day's work =

how many days will 10 women finish it?


Ans: Considering one day's work,

1
12

1
30

1
1
1

A's 1 day's work =


12 30 20
A alone can finish the work in 20 days
Using formula :
xy
yx

12 x30 12 x 30

20 days
30 12
18

4 . 16 men can do a piece of work in 10


days. How many men are needed to complete
the work in 40 days?

3M + 7W =

1
...... (2)
10

(1) x 3 - (2)x 4 gives


1
1
or 10W =
10
40

10 Women can do the work in 40


days
7 . A certain number of men complete a
piece of work in 60 days. If there were 8 men
more, the work could be finished in 10 days
less. How many men were there originally?
Ans: Let the original number of men be x.
M1 = x, D1 = 60, M 2 = x +8, D2 = 50,

Ans: Using formula.


M1 D1 = M2 D2
M 1 = 16, D 1 = 10,

1
....... (1)
8

18W - 28W =

Time taken by A to finish the work =


=

4M + 6W =

M1 D1 = M2 D2
x x 60 = ( x+8) x 50

D 2 = 40

60 x - 50 x = 400

16x10 = M 2 x 40

10 x = 400 x = 40

16 x10
= 4 men
40
5 . A and B can do a piece of work in 18
days, B and C in 24 days, A and C in 36 days.
In what time can they do it all working together?

8 . A cistern can be filled separately by two


pipes in 12 and 16 minutes respectively. If both
pipes are opened together, when will the cistern be filled?

Ans: [(A+B)+(B+C)+(A+C)]'s 1 day's work

Ans: Work done by Ist pipe in 1 minute =

1
12

Work done by 2nd pipe in 1 minute =

1
16

M2 =

1
1
1 1

18 24 36 8

or 2 (A+B+C)'s 1 day's work =

or (A+B+C)'s 1 day's work =

1
8

1
16

So they all can finish the work in 16 days


6 . 4 men and 6 women finish a job in 8 days,
while 3 men and 7 women finish in 10 days. In

Work done by both in 1 minute=

1 1

12 16

4 3 7

48
48

Both the pipes together will fill the cistern


48
6
in
minutes ie, 6
minutes.
7
7

Using formula :

PRACTICE TEST

Time taken to fill the cistern by both the pipes

ab
12 x16
6

6
minutes
a b 12 16
7

9.
Two inlet pipes of filling rate 10 minutes
per cistern and 6 minutes per cistern and one
outlet pipe of emptying rate 15 minutes per
cistern are all fitted to a cistern and are opened
together. Find when the cistern will be full?
Part of the cistern filled by working the three
pipes in one minute.
=

1 1 1 1

10 6 15 5

Time needed to fill the full cistern = 5


minutes

1 0 . A cistern can be filled separately by two


pipes A and B in 36 minutes and 45 minutes
respectively. A tap C at the bottom can empty
the full cistern in 30 minutes. If the tap C is
opened 7 minutes after the pipes A and B are
opened, find when the cistern becomes full.
Ans: Part of the tank filled by A and B in
7 minutes

1 7
1

= 7 x
36 45 20

7 13
1

of the tank should be


20 20
filled.
Part filled by A, B and C in 1 minutes
=

1
1
1
1

36 45 30 60

Time needed to fill

13
x 60 39
20

13
part of the tank
20

minutes

Total time taken to fill the tank

= 39 + 7 = 46 minutes

1 . Ramesh alone does a piece of work in 4


days and Suresh does it in 12 days. In how
many days will the two do it together?
1)

3 days

3)

4 days

1
days
2
4) 8 days

2) 1

2 . Pranesh and Sumesh can finish a work in


16 days while Pranesh can do the same work
in 24 days. In how many days can Sumesh
alone finish the same work?
1)

40 days

2) 25 days

3)

48 days

4) 20 days

3 . Vinod can do a work in 15 days, Vijay in


25 days and Vinay in 30 days. How long will
they take to do the work if they work together?
1)

12 days

3)

70 days

1
days
7
4) 20 days
2) 7

4 . If A, B and C together can finish a piece


of work in 4 days, A alone in 12 days and B in
18 days, then C alone can do it in
1)

21 days

2) 15 days

3)

12 days

4) 9 days

5 . 3 men or 6 women can do a piece of work


in 20 days. In how many days will 12 men and
8 women do the same work?
1)
3)

7
2
5

15
4
4) 4

2)

6 . Some persons can do a piece of work in


12 days. Two times the number of those persons will do half of that work in
1) 3 days
2) 4 days
3) 6 days
4) 12 days
7 . 3 men can do a work in 6 days. After 2
days 3 more men joined them. How many days
will they take to complete the remaining work?
1) 5 days
2) 4 days
3) 3 days
4) 2 days

8 . A is twice as good a workman as B and


they took 7 days together to do the work. B
alone can do it in:
1)

12 days

2) 18 days

3)

21 days

4) 16 days

9 . A can do a piece of work in 25 days and


B can do the same work in 30 days. They work
together for 5 days and then A leaves. B will
finish the remaining work in
1)

21 days

2) 11 days

3)

20 days

4) 19 days

1 0 . An army of 2000 men had enough food


to last for 30 days. After 10 days 500 more
men joined them. How long did the food last
then?
1)

20 days

2) 15 days

3)

12 days

4) 16 days

1 1 . Amar can do a piece of work in 15 days.


When he had worked for 3 days, Sameer joined
him and the remaining work was finished in 8
days. In how many days can Sameer alone finish the whole work?
1)

30 days

2) 27 days

3)

20 days

4) 24 days

1 2 . A, B and C can do a piece of work in 18


days, 27 days and 36 days respectively. They
start working together. After working for 4
days, A goes away and B leaves 7 days before the work is finished. Only C remains at
work from beginning to end. In how many days
was the whole work done?
1)

17 days

2) 18 days

3)

16 days

4) 15 days

1 3 . A and B can do a piece of work in 6 days.


B and C in 4 days and A and C in 5 days. How
long will they take to complete the work if
they work together?
1)

9
3
days
37

2) 15 days

3)

23
1
days
37

9
4) 6
days
37

1 4 . A man, a woman or a boy can do a piece


of work in 3,4 and 12 days respectively. How
many boys must assist 1 man and 1 women to
do the work in 1 day?
1)

5 boys

2) 6 boys

3)

2 boys

4) 20 boys

1 5 . Two pipes can fill a tank in 9 hours and


12 hous respectively. In how much time will
they fill the tank when opened together?
1
hours
2

1)

3)

5 hours

2) 5

1
hours
7

4) 3

2
hours
3

1 6 . A tap can fill a tank in 8 hours and another can empty it in 16 hours. If both the
taps are opened simultaneously, the time (in
hours) to fill the tank is:
1)

2) 1 0

3) 1 6

4) 2 4

1 7 . A cistern can be filled by a pipe in 15


hours. But due to a leak in the bottom the cistern is just full in 20 hours. When the cistern
is full, the leak can empty it in:
1)

60 hours

2) 40 hours

3)

45 hours

4) 30 hours

1 8 . A cistern can be filled by pipes A and B


in 20 hours and 30 hours respectively. When
full, the tank can be emptied by pipe C in 60
hours. If all the taps be turned on at the same
time the cistern will be full in
1)

10 hours

2) 15 hours

3)

16 hours

4) 30 hours

1 9 . Two pipes A and B can separately fill a


tank in 12 minutes and 15 minutes respectively. Both the pipes are opend together but
4 minutes after the start, pipe A is turned off.
How much time will it take to fill the tank?
1)

11 min

2) 12 min

3)

6 min

4) 8 min

2 0 . Two pipes A and B can fill a cistern in 24


minutes and 32 minutes respectively. If both

the pipes are opened together, then after how


many minutes B should be closed so that the
tank is full in 18 minutes?
1)

2) 8

3) 1 0

4) 1 2

2 1 . Two pipes A and B can separately fill a


cistern in 18 minutes and 24 minutes respectively. If they are turned alternatively for one
minute each, how long will it take to fill the
cistern?
1)

2) 24 min
3) 25 min
4) 20 12 min
2 2 . A tank can be filled by one tap in 20 minutes and by another in 25 minutes. Both the
taps are kept open for 5 minutes and then the
second is turned off. In how many minutes
more is the tank completely filled?
1)

21 min

2) 1 1

4) 17 12
2 3 . If 12 men or 18 women can reap a field
in 14 days; then the number of days that 8
men and 16 women will take to reap the same
field is
3)

1)

12

2) 7

3) 8

4) 9

2 4 . 6 men and 6 women finish a job in 8 days,


while 3 men and 7 women finish it in 10 days.
How many days will 10 women take to finish
the job?
1)

24

2) 3 2

2
3) 10
3

together but A left 3 days before the completion of work. The total number of days to complete the work is
1) 6 3 5 days
2) 9 days
3) 9 15 days
4) 10 15 days
2 6 . 9 men have finished one-third of the work
in 10 days. The number of additonal men required for finishing the remaining work in 2
more days will be
1)

2 5 . A can do a piece of work in 12 days and


B can do it in 18 days. They started the work

2) 7 8

3) 5 5

4) 3 0

2 7 . Two pipes A and B can fill a tank in 20


and 24 minutes respectively and a third pipe
can empty 3 gallons per minute. If A, B and C
working together can fill the tank in 15 mts,
The capacity of the tank in gallons is
1)

60

2) 1 2 0

3) 1 5 0

4) 1 8 0

2 8 . A, B and C together can do a piece of


work in 20 days. After working with B and C
for 8 days, A leaves and then B and C complete the remaining work in 20 days more. In
how many days, A alone could do the work?
1)

40

2) 5 0

3) 6 0

4) 8 0

2 9 . A certain job was assigned to a group of


men to do in 20 days. But 12 men did not turn
up for the job and the remaining men did the
job in 32 days. The original number of men in
the group was
1)

4) 3 6

81

32

2) 3 4

3) 3 6

4) 4 0

3 0 . If x men can do a piece of work in 8 days


and ( x+4) men can do the same work in 6
days. Then x is equal to
1)

2) 1 0

3) 1 2

4) 2 4

ANSWERS TO PRACTICE TEST


1.

(1)

2.

(3)

9.

(4)

1 0 . (4)

1 7 . (1)
25. ( b )

3.

(2)

4.

(4)

5 . (2)

6.

(1)

7.

(4)

8.(3)

1 1 . (1)

1 2 . (3)

13. (1)

1 4 . (1)

1 5 . (2)

16.(3)

1 8 . (2)

1 9 . (3)

2 0 . (2)

21.(4)

2 2 . (2)

2 3 . (4)

2 4 . (3)

2 6 . (1)

2 7 . (2)

28.(2)

29.(1)

3 0 . (3)

BANK PROBATIONERY OFFICER


QUANTITATIVE APTITUDE
SIMPLE & COMPOUND INTEREST
Interest is the money paid by the borrower
to the lender for the use of money lent. Interest is of two kinds, simple and compound.
Money borrowed or deposited is called the
principal. The sum of principal and interest is
called the amount.
i)

The simple interest (I) for a principal (P)


for (N) years at (R) rate percent per annum is
PNR
I =
100
Ix100
Ix100
Ix100
; R
; N
P =
NxR
PxN
PxR

Money is said to be lent at Compound


Interest if the interest is not paid as soon as it
falls due, but is added to the principal after a
fixed period, so that the amount at the end of
the period, becomes the principal for the next
period.
a)

When interest is compounded annu-

ally:
R

Amount = P 1

100

b)
yearly:

c)

When interest is compounded half

Amount = P 1
200
terly

d)

R
P 1 100 1

(or)

1 . Calculate the amount on Rs. 4480 at 8%


per annum for 3 years.
Ans: S.I. =

PxNxR
100

Amount

4480 x 3 x 8
Rs.1075.20
100

= Rs. (4480 + 1075.20)


= Rs. 5555.20

2 . S.I. on Rs. 1500 at 7% per annum for a


certain time is Rs. 210. Find the time
Ans: Time, N =

210 x100
= 2 years
1500 x7

3 . A certain sum of money at simple interest amounts to Rs. 1260 in 2 years and to Rs.
1350 in 5 years. The rate percent per annum
is ------Ans: S.I. for 3 years = Rs. (1350 - 1260)=
Rs. 90

2N

When interest is compounded quar-

Solved Examples:

= Rs.

Compound Interest:

4N

C.I.=P 1
100

Simple Interest:

If the interest on a certain sum borrowed


for a certain period is reckoned uniformly, it is
called Simple Interest.

ii)

Amount = P 1

400

S.I. for 2 years = Rs.

90
x 2 = Rs. 60
3

Principal = Rs. (1260 - 60) = Rs. 1200

Rate, R =
4.

100 x 60
% 2.5%
1200 x2

A man invested 13 of his capital at 7%,

Then,

x x (R 1) x 2 x x Rx 2

24
100
100

2 xR 2 x 2 xR

24
100 100 100

4 at 8% and the remainder at 10%. If his


annual income is Rs. 561, the capital is -----Let the capital be Rs. x. Then,
x
7
x
8
5 x 10
x
x1 x
x 1
x
x 1 561
3 100
4 100
12 100

7x
8x
5 0x
x
x1
561
300 400
1200

102 x
561
1200

x =

5.

x=

24 x100
Rs.1200
,
2

7 . Find compound interest on Rs. 5,000 at


10% per annum for 3 years

Ans: Amount = P 1
100
10

= 5000 1
100

561x1200
Rs. 6,600
102

Find the sum of money which increases

1
10 of itself every year and amounts to
Rs. 450 in 5 years at S.I.
Ans: Let P = Rs. 100
S.I. = Rs. 100 x 110 = Rs. 10
S.I. for 5 years = Rs. 50
Amount after 5 years=100+50
=Rs. 150

100 x 450
Rs.300
150

6 . A sum was put at simple interest at a


certain rate for 2 years. Had it been put at
1% higher rate, it would have fetched Rs. 24
more. Find the sum.
Ans: Let the sum be Rs. x and rate be R% and
(R+ 1)%

= Rs. 6,655

Compound Interest
= Rs. (6,655 -5,000) = Rs. 1,655
8 . If the compound interest on a certain sum
for 3 years at 20% per annum is Rs. 728, find
the simple interest.

20

Ans: Given that P 1


100

- P = Rs. 728

or 1.728 P - P = Rs. 728

P Rs.1000
,
Now, S.I. = Rs.

If the amount is Rs. 150, P = Rs. 100

If the amount is Rs. 450,

1000 x3 x20
Rs. 600
100

9 . The difference between the compound


interest and the simple interest on a certain
sum at 10% per annum for two years is
Rs. 60. Find the sum.
Ans: Let the sum be Rs. x
So, S.I. =Rs.

x x 10 x2
x
Rs.
100
5
2

10
21x

x Rs.
C.I.=Rs. x 1

100
100

C. I. S. I.

21x x
x

Rs.60
100 5 100

x = Rs. 6,000

PRACTICE TEST
1 . At what rate percent per annum will a
sum of Rs. 3,600 become Rs. 4,500 in 10 years
at simple interest?
1)

5%

2) 2 . 5 %

3)

10%

4) 6 . 7 5 %

2 . A sum of Rs. 1600 lent at simple interest at 12.5% per annum will become double
in
1)

6 years

2) 7 12 years

3)

8 years

4) 9 14 years

3 . The difference in simple interest at 13%


and 12% p.a. of a sum in one year is Rs. 110.
Then the sum is
1)

Rs. 13,000

2) Rs. 15,000

3)

Rs. 10,000

4) Rs. 11,000

4 . The difference in the interests received


from two different banks on Rs. 1000 for 2
years is Rs. 20. Thus, the difference in their
rates is
1)

2%

2) 1 %

3)

1.5%

4) 0 . 5 %

5 . Find out the capital required to earn a


monthly interest of Rs. 600 at 6% simple interest.
1)

Rs. 1 lakhs

2) Rs. 1.2 lakhs

3)

Rs. 1.1 lakhs

4) Rs. 1.3 lakhs

6 . A man invested 1/3 rd of the sum at 7%,


1/4th at 8% and the remaining at 10% for one
year. If the annual interest is Rs. 408, then
the investment is
1)

Rs. 8,400

2) Rs. 4,800

3)

Rs. 5,000

4) Rs. 7,200

7 . The difference in simple interest on a


certain sum of money for 3 years and 5 years
at 18% per annum is Rs. 2,160. Then the sum
is
1)

Rs. 6,500

2) Rs. 4,500

3)

Rs. 6,000

4) Rs. 7,500

8 . At what rate percent per annum simple


interest will a sum of money triple itself in 25
year s?
1)

3)

2) 8
1
11

1
3

4) 1 0

9 . What sum of money lent out at compound


interest will amount to Rs. 968 in 2 years at
10% per annum, interest being charged annually?
1)

Rs. 900

2) Rs. 825

3)

Rs. 780

4) Rs. 800

1 0 . The difference between compound interest and simple interest on certain sum of money
in 2 years at 4% per annum is Rs. 50. Find the
sum
1)

Rs. 30,550

2) Rs. 31,250

3)

Rs. 25,670

4) Rs. 35,400

1 1 . A sum of money lent at compound interest amounts to Rs. 1210 in two years and to
Rs. 1464.10 in 4 years. Find the rate of interest .
1)

12%

2) 1 0 %

3)

8%

4) 1 5 %

1 2 . A man borrows Rs. 4,000 at 8% per annum on compound interest. At the end of every year he pays Rs. 1,500 as part payment
of loan and interest. How much does he still
owe to the bank after 3 such annual payments?
1)

Rs. 1,799

2) Rs. 169.25

3)

Rs. 2,000

4) Rs. 234

1 3 . The cost of a machine is estimated to be


decreasing at the rate of 15% every year. If
it costs Rs. 6,000 now, what will be the estimated value of the machine after 2 years?
1)

Rs. 3,750

2) Rs. 4,335

3)

Rs. 3,225

4) Rs. 5,000

1 4 . A tank contains 18,000 litres of water.


If it decreases at the rate of 5% a day, what
will be the quantity of water after 2 days

rate of interest per annum is ------1)

2%

2) 5 %

3)

4%

4) 3

1
%
3

1 9 . The simple interest on a certain sum of


money for 4 years at 4 percnet per annum
exceeds the compound interest on the same
sum for 3 years at 5 percent per annum by
Rs. 57. Find the sum.

1)

16,245 litres

2) 15,234 litres

1)

Rs. 24,000

2) Rs. 12,500

3)

17,225 litres

4) 18,200 litres

3)

Rs. 26,700

4) Rs. 23,050

1 5 . A sum of money doubles itself at compound interest in 15 years. It will become 8


times in
1)

40 years

2) 30 years

3)

60 years

4) 45 years

1 6 . Ram borrowed Rs. 5,000 from Sanjay


with simple interest. After 2 years, Sanjay got
Rs. 1,000 more than what he had given to Ram.
What was the percentage of interest per annum?
1)

10

2) 2 0

3) 1 5

4) 5

1 7 . A sum doubles in 20 years at simple interest. How much is the rate per annum?
1)

5%

2) 4 %

2
3) 1 2 % 4) 6 %
3

1 8 . The simple interest on a sum of money is


1 th of the principal and the number of years
9
is equal to the rate percent per annum. The

2 0 . A man borrows Rs. 4,000 from a bank at


1
% interest. At the end of every year, he
2
pays Rs. 1,500 as part repayment of loan and
interest. How much does he still owe to the
Bank after 3 such annual payments?
7

1)

Rs. 123.25

2) Rs. 25

3)

Rs. 124.25

4) 1 2 5

2 1 . A certain sum lent out at simple Interest


and the true discount on a certain sum for 1
year at 5% is Rs. 1 find the sum.
1)

Rs. 400

2) Rs. 420

3)

Rs. 450

4) Rs. 500

2 2 . If the amounts for a fixed principal after


3 and 2 years at a certain rate of compound
interest are in the ratio 21 : 20. The rate of
interst is
1)

7%

2) 6 %

3)

5%

4) 4 %

ANSWERS TO PRACTICE TEST


1.

(2)

2.

(3)

9.

(4)

1 0 . (2)

1 7 . (1)

1 8 . (4)

3.

(4)

4.

(2)

5 . (2)

6.

(2)

1 1 . (2)

1 2 . (2)

13. (2)

1 4 . (1)

1 9 . (1)

2 0 . (1)

2 1 . (2)

2 2 . (3)

7.

(3)

1 5 . (4)

8.(1)
16.(1)

Bank Probationary officer


Quantitative Aptitude

PROBLEMS ON AGE

(1) + (2) y = 30

Solved Examples
1 . A father was 4 times as old as his son 8
years ago. Eight years hence, father will be
twice as old as his son. Find their present ages.

Substituting y = 30 in equation (1) we


get x = 70

Ans: Let son's age 8 years ago be x years.

4 . Ratio of Ashok's age to Pradeep's age is


equal to 4:3. Ashok will be 26 years old after
6 years. How old is Pradeep now?

Thus, father's age at that time = 4 x years


After 8 years, son's age
= ( x+8) +8 = ( x+16) years

Ratio of their ages = 70 : 30 or 7:3

Ans: Ashok's present age = (26-6)

After 8 years, father's age

= 20 years

= (4 x+8)+8 = (4 x+16) years

2( x+16) = 4 x + 16 or x=8
The present age of the son = x+8 = 16
yea rs

Pradeep's present age = 20x

3
4

= 15 years

The present age of the father


= 4 x+8 = 32+8= 40 years

5 . The ratio of the ages of father and son at


present is 6:1. After 5 years the ratio will become 7:2. The present age of the son is:

2 . A is twice as old as B was two years ago.


If the difference in their ages be 2 years, find
A's age.

Ans: Let their present ages be 6x and x years


respectively.

Ans: Let B's age 2 years ago be x years

A's present age = 2 x years


Also 2 x - ( x+2) = 2 or x=4
A's age = 2x4 = 8 years

Then

6x 5 7

x5 2

= 2 ( 6x+5) = 7 ( x+5)

x=5

Present age of the son = 5 years.

3 . The age of a father 10 years ago was


thrice the age of his son. Ten years hence, the
father's age will be twice that of his son. The
ratio of their present ages is:

6 . Three years ago the average age of A


and B was 18 years. With C joining them now,
the average becomes 22 years. How old is C
n ow?

Ans: Let the present ages of father and son


be x and y years respectively.

Ans: (A+B)'s total present age

Then (x-10) = 3 (y-10) or


3y-x = 20 ------ (1)
and ( x+10) = 2 (y+10) or
x-2y = 10 ----- (2)

= (2x18+3+3) = 42 years
(A+B+C)'s total present age
= 22 x 3 = 66 years
C's age = 66-42 = 24 years

PRACTICE TEST
1 . A father is twice as old as his son. 20
years ago, the age of the father was 12 times
the age of the son. The present age of the son
is
1) 44 years
2) 22 years
3) 40 years
4) 20 years
2 . The respective ages of a father and his
son are 41 and 16 years. In how many years
will the father be twice as old as his son?
1) 19 years
2) 9 years
3) 15 years
4) 10 years
3 . The ratio of ages of Mohan and Sohan is
4:3. The sum of their ages is 42 years. The
age of Mohan is
1) 24 years
2) 18 years
3) 32 years
4) 30 years.
4 . The ratio of ages of Rani and Vinita is
3:5. The difference in their ages is 12 years.
Then the age of Vinita is
1) 20 years
2) 15 years
3) 18 years
4) 30 years
5 . Two years ago, Vinod was four times as
old as Indhu. 8 years hence, Vinod's age will
exceed Indhu's age by 12 years. The ratio of
the present ages of Vinod and Indhu
1) 5: 1
2) 4: 1
3) 3: 1
4) 2: 1
6 . The ages of A and B are in the ratio 3:5.
After 9 years the ratio of their ages will be
3:4. The present age of B is
1) 9 years
2) 15 years
3) 20 years
4) 16 years
7 . A's mother was four times as old as A
ten years ago. After 10 years she will be twice
as old as A. Then, A's present age is
1) 30 years
2) 25 years
3) 20 years
4) 15 years

8 . Afather's age is three times the sum of


the ages of his two children, but 20 years hence
his age will be equal to sum of their ages. Then
the fathers age is 1) 30 years
2) 40 years
3) 35 years
4) 45 years
9 . The ratio of the father's age to the son's
age is 4:1. The product of their ages is 196.
The ratio of their ages after 5 years will be:
1) 3: 1
2) 10 : 3
3) 11 : 4
4) 14 : 5
1 0 . In 10 years, A will be twice as old as B
was 10 years ago. If A is now 9 years older
than B, find the present age of B.
1) 3 9
2) 2 7
3) 4 5
4) 2 6
1 1 . A is as much younger than B as he is older
than C. If the sum of B's and C's ages is 40
years, find the age of A.
1) 40 years
2) 10 years
3) 25 years
4) 20 years
1 2 . The ages of Ram and Mohan differs by
16 years. Six years ago, Mohan's age was
thrice as that of Ram's. Then Ram's present
age is
1) 15 years
2) 20 years
3) 14 years
4) 30 years
1 3 . A father is 4 times as old as his son; in
20 years he will be only twice as old as his
son. Then the respective ages of father and
son are
1) 40, 10 years
2) 80, 20 years
3) 60, 15 years
4) 48, 12 years
1 4 . The difference between the ages of two
persons is 8 years. 15 years ago, the elder
one was twice as old as the younger one. Then
the present age of the elder person is
1) 23 years
2) 31 years
3) 34 years
4) 40 years

ANSWERS TO PRACTICE TEST


1.

(2)

2.

(2)

9.

(3)

1 0 . (1)

3.

(1)

1 1 . (4)

4.

(4)

5 . (3)

6.

(2)

1 2 . (3)

13. (1)

1 4 . (2)

7.

(3)

8.(1)

BANK PROBATIONARY OFFICER


QUANTITATIVE APTITUDE
MENSURATION

AREA OF PLANE FIGURES

Circumference of a circle = 2 r

1.

Length of an Arc =

Triangle

1
x base x height
2
3
(side)2
Area of an equilateral triangle =
4
If the length of the sides of a triangle are a, b

Area of triangle =

(a b c)
and c and if the semiperimeter s =
2
then,
Area of the triangle

=
2.
3.

4.

r2
360
The unit of area is square metre
Area of a sector =

Volume & area of Solid Structures.


1.

Volume of a cuboid = lbh


Longest diagonal of a cuboid

Parallelogram
Area = base x height
Rectangle
Area
= Length x Breadth
Perimeter = Sum of all sides
= 2 (Length + Breadth)
Square
Let `a' be the side of square

2a

Area of a room = (length x breadth)


Area of 4 walls of a room
= 2 (l+b) height
2.

6.

7.

1
Area =
(Product of the diagonals)
2
Circle

Volume of the cube = a 3


Longest diagonal =

3 x (a
Total surface area = 6a 2
Total length of all edges = 12a.

Area =

Let r be the radius of the circle


2
Area of a circle = r ,

22
or 3.14
7

Cube
Let `a' be the side of a cube

1
diagonal2
2
Perimeter = 4a
Trapezium
1
(sum of parallel sides) x
2
(distance between them)
Rhombus

l2 b2 h2

Total surface area = 2(lb+bh+lh)

Area = a2 =
5.

Cuboid

Let l, b, and h are respectively the length,


breadth and height of a cuboid

s (s a) (s b)(s c)

Diagonal =

2 r
360

3.

Cylinder
Volume = r2 h ; r - radius of the base
h - height
2

Base area = r

Curved surface area = 2r h


Total surface area = 2 rh 2 r2
4.

Pyramid
Volume =

5.

1
x (area of base) x height
3

Cone

1
x 3 x 4 6 sq. m.
2
2 . A lawn is in the form of a rectangle having its sides in the ratio 2:3. The area of the

1
hectare. Find the length and breadth
6
of the lawn.

l 2 = h2 + r 2

lawn is

1 2
r h
3
Area of curved surface =r l
Total surface area =r l + r 2
= r(l + r)

Ans: Let the length and bread be 3 x and 2 x


metres.

Volume =

Then

1
6 x2 10000
x 10000
6

50
3

Sphere
4 3
r ; r- radius of the sphere
Volume =
3
2
Curved surface area = 4 r

7.

1
x base x altit ude
2

If the radius of the base of a cone is r,


the height is h and the slant height is l then

6.

5 2 3 2 16 4m.
Area of the triangle
=

Hemisphere
Volume =

Length = 3 x

50
1
33 m
3
3
3 . A rectangular grassy plot is 112 metres
by 78 metres. It has a gravel path 2.5 metres
wide all round it on the inside. Find the area of
the path and the cost of constructing it at 72
paise per 1000 sq.cm.
Breadth = 2 x

2 3
r ;
3

Curved surface area = 2 r2


Total surface area = 3 r2
4
(R3 r 3 )
3
Volume of a metal in a hollow pipe

Volume of spherical shell =

Ans: Area of path = Area of grassy plot Area of plot excluding the path
= (112 x 78 - 107 x 73) = 925 m 2

= (R2 r2 )h ; R - external radius


Total surface area of an open pipe

50
50 m
3

2.5

78

2.5

2
2
= 2 Rh rh (R r )

112

The unit of volume is cubic metre.


Solved Examples
1 . Find the area of a right triangle whose
hypotenuse is 5 metres and base is 3 metres.
Ans: Altitude of the triangle

Cost on 1000 sq.cm. = 72 paise.


Total Cost
= Rs.

72
925 x100 x100
x
Rs.6,660
1000
100

4 . How many bricks will be required for a


wall 8m long, 6 m high and 22.5 cm thick if
each brick measures 25 cm by 11.25 cm by 6
cm ?
Ans: Volume of wall = (800x600x22.5) cu.cm
Volume of brick = (25x11.25x6) cu.cm.
Volumeof t hew all
Volume of abrick

Number of bricks =

Area of JKLM = 5x5 = 25 sq.m.


Area of the path of be gravelled
= (300+200-25) = 475 sq.m.
= cost of gravelling the path.
= Rs.

60 x 475
Rs. 285
100

Quicker Method
800 x 600 x22.5
6400
=
25 x1125
. x6
5 . Half cubic metre of gold sheet is extended
by hammering so as to cover an area of 1 hectare. Find the thickness of the gold sheet.
1
cu.metre
2

Ans: Volume of sheet =

1
x 100x100 x100 cu.cm
2
Area of sheet=1hectare
=10,000 sq. metre
= [10,000x100x100] sq.cm.

Thickness =
=

= 5(60 + 40 - 5) = 475 sq.m


Cost =

60 x 475
Rs.285
100

7.
A rectangular courtyard 3.78 metres
long and 5.25 metres broad is to be paved
exactly with square tiles, all of the same size.
What is the largest size of such a tile? Also
find the number of tiles?
Ans: Largest size of tile
=H.C.F. of 378 cm and 525 cm=21 cm.

Volume
Area

Area of courtyard = (378x525) sq.cm.

1x100 x100 x100


2 x10,000 x100 x100

1
cm = 0.005 cm.
200
6 . A rectangular lawn 60 metres by 40
metres has two roads each 5 metres wide,
running in the middle of it, one parallel to length
and the other parallel to the breadth. Find the
cost of gravelling them at 60 paise per square
metre.
Ans: Area of the road ABCD=60x5=300 sq.m
Area of the road EFGH = 40x5
= 200 sq.m.

Number of tiles =

Ans: Distance covered by wheel in 1 minute


=

66 x1000
,
x100
= 1,10,000 cms.
60

Circumference of wheel

K
G

22
x 70 440 cm.
7

Number of revolutions in 1 minute


D C

A B

378 x525
450
21x21

8 . The diameter of the driving wheel of a


bus is 140 cm. How many revolutions per
minute must be the wheel make in order to
keep a speed of 66 km. per hour?

= 2x

E H

Area of the road = (width of the road) x (length


+ breadth of the lawn - width of the road)

110
, ,000
250
440

9 . The length of a rectangle is increased by


60%. What per cent should the width be decreased to maintain the same area?
Ans: Let the length of rectangle be increased
by x%. Then the decrease percentage of
breadth to maintain the same area

100

= x
100 x

cylinder be r cm and h cm respectively.


Then V1 r2 h
V2 x(2 r)2 x

h
2 r h
2

New Volume
2 r2 h 2
2
Pr evious volume r h 1

In the above problem decrease percentage of

Required ratio = 2:1

1
100
37 %
breadth = 60

100 60
2

PRACTICE TEST

1 0 . A cube of sides 3 cm. is melted and


smaller cubes of sides 1 cm. each are formed.
How many such cubes are possible?

1 . The perimeter of a rectangle is 82 m. and


its length is 25m. The breadth of the rectangle
is:

Ans: Quicker method

1)

14 m.

2) 16 m

Required Number

3)

18 m.

4) 12 m.

Original lengt h of side

=
New length of side

3
= 27
1
1 1 . Three cubes of metals whose edges are
3, 4 and 5 cm. respectively are melted and
formed into a single cube. If there is no loss of
metal in the process, find the side of the new
cube.
Ans: Quicker method
When many cubes integrate into one cube, the
side of the new cube is given by
3

side = sum of cubes of sides of all thecubes


Here, side
=

3 3 43 5 3 3 27 64 125 6cm

1 2 . If the radius of a cylinder is doubled and


the height is halved, what is the ratio between
the new volume and previous volume?
Ans: Let the initial radius and height of the

2 . The length of a plot of land is 4 times its


breadth. A play ground measuring 1200m 2 occupies one third of the total area of the plot.
What is the length of the plot, in metres?
1)

90

2) 8 0

3)

60

4) 1 2 0

3 . If the width of a rectangle is 2m. less


than its length and its perimeter is 32m., the
area of the rectangle is:
1)

224 m 2

2) 108 m 2

3)

99 m 2

4) 63 m 2

4 . The sides of a rectangular park are in the


ratio 3:2 and its area is 3750m 2. The cost of
fencing it at 50 paise per metre is:
1)

Rs. 312.50

2) Rs. 375

3)

Rs. 187.50

4) Rs. 125

5 . The least number of equal square slabs


that can be fitted in a verandah 10.5m. long
and 3m wide is
1)

15

2) 1 4

3)

13

4) 1 2

6 . Three equal cubes are placed adjacently


in a row. Find the ratio of the total surface
area of the new cuboid to that of the sum of

the surface area of the 3 cubes.


1) 7: 9
2) 6: 5
3) 7: 8
4) 5: 6
7 . If the radii of the two sphere are in the
ratio 1:4, what will be the ratio of their volumes?
1) 1: 8
2) 1: 16
3) 1: 24
4) 1: 64
8.
are
the
rod

Six spherical copper balls of radius `r'


melted and cast into a cylinderical rod of
metal of same radius. The height of the
will be
1) 4 r
2) 6 r
3) 8 r
4) 12 r

9 . A man walked 20 metres to cross a rectangular field diagonally. If the length of the
field is 16 metres, what is the breadth of the
field?
1) 1 6
2) 4
3) 1 2
4) 8
1 0 . If the ratios of areas of two squares is
9:1, the ratio of their perimeter is
1) 9: 1
2) 3: 4
3) 3: 1
4) 1: 3
1 1 . If the area of a cirlce is equal to its circumference, then diameter of the circle is:
1) 2cms.
2) 4cms.
3) 1 cm.
4) cms
1 2 . If the side of a sqaure is increased by
20%, the area of the square is increased by:
1) 4 0 %
2) 3 5 %
3) 4 4 %
4) 2 0 %
1 3 . If the area of a circle is decreased by 36%
then radius of a circle decreases by
1) 2 0 %
2) 1 8 %
3) 3 6 %
4) 6 4 %
1 4 . If the area of a square is increased by

96%, then the side of the square is increased


by
1) 9 6 %
2) 4 8. 5 %
3) 4 0 %
4) 4 9 %
1 5 . A rectangular hall 10m. long and 8m.
broad, is surrounded by a verandah 2 metres
wide. Find the area of the verandah.
1)
3)

104m2
96 m

2) 88 m 2
4) 120 m 2

1 6 . The surface area of a cube is 600m2. The


length of its diagonal is:
1)
3)

10
3

cm

10 3 cm

2)

10
2

cm

4) 10 2 cm.

1 7 . Two circular cylinders of equal volume


have heights in the ratio 1:2. The ratio of their
radii is:
1)

1:

3)

1: 4

2)

2 :1
4) 2: 1

1 8 . A copper sphere of radius 3cm. is beaten


and drawn into a wire of diameter 0.2cm. The
length of the wire is
1) 9m .
2) 12m .
3) 18m .
4) 36m .
1 9 . The base of a prism is a triangle of sides
3m., 4m, and 5m. respectively. The height of
the prism is 10m. Then its volume is:
1) 60 m 2
2) 75 m 2
2
3) 42 m
4) 83 m 2
2 0 . One
extended
roof with
thickness

cubic metre of aluminium sheet is


by hammering, so as to cover the
an area 10,000 square metre. The
of the sheet is:

1)

0.1cm.

2) 0.01m.

3)

. 01cm

4) 0.5cm

ANSWERS TO PRACTICE TEST


1.

(2)

2.

(4)

9.

(3)

1 0 . (3)

1 7 . (1)

1 8 . (4)

3.

(4)

4.

(4)

5 . (2)

6.

(a )

1 1 . (2)

1 2 . (3)

13. (1)

1 4 . (3)

1 9 . (1)

2 0 . (3)

7.

(4)

1 5 . (2)

8.(3)
16.(3)

Bank probationary Officer


Quantitative Aptitude

SERIES

A series is a sequence of numbers


where the sequence of numbers is obtained by
some particular predefined rule and applying that
predefined rule it is possible to find out the next
term of the series.
A series can be created in many ways.
Some of these are as explained below:
1.
Arithmetic series:- An arithmetic series is one in which successive numbers are
obtained by adding or subtracting a fixed number
to the previous number
Eg: i. 5, 9, 13, 17, -------ii. 35, 31, 27, 23, 19, -----2.
Geometric Series: A geometrical series is one in which each successive number is
obtained by multiplying or dividing a fixed number
by the previous number.
Eg: i. 4, 8, 16, 32, 64, -----ii. 15, -30, 60, -120, ----iii. 3125, 625, 125, 25, 5, ----3.
Series of squares, cubes etc:These series can be formed by squaring or cubing every successive number.
Eg: i. 2, 4, 16, 256, ----ii. 2, 8, 512, -----4.
Mixed Series:
A mixed series is basically the one we
need to have sound practice of because it is
generally the mixed series which is asked in
the examinations. By a mixed series, we mean
a series which is created according to any nonconventional (but logic) rule. Because there is
no limitations to people imagination, there are
infinite ways in which a series can be created and
naturally it is not possible to club together all of
them. Still we are giving examples of some more
popular ways of creating these mixed series.

i.
Two - tier Arithmetic series - A two
tier Arithmetic series shall be one in which the
differences of successive numbers themselves
form an arithmetic series.
Eg:
i.
1, 2, 5, 10, 17, 26, 37, ---ii.
3, 5, 9, 15, 23, 33, ---Two - tier arithmetic series can be denoted as a
quadratic function, which can be denoted as
f ( x) x 2 1, where x 0, 1, 2,
ii.
Three - tier Arithmetic series:
This as the name suggest is a series in
which the differences of successive numbers
form a two - tier arithmetic series, whose successive terms differences, intern, form an arithmetic series.
Eg: (i) 336, 210, 120, 60, 24, 6, 0, --Here the differences in successive terms
are 126, 90, 60, 36, 18, 6, ---The differences of successive terms of
this new series are 36, 30, 24, 18, 12, ----- which
is an arithmetic series.
The three-tier arithmetic series can be
denoted as a cubic function, which can be denoted as f ( x) x 3 x, where x 1, 2,
(iii)
We know that,
(a) In an arithmetic series, we add or
deduct a fixed number to find the next number,
and
(b) In a geometric series we multiply or
divide a fixed number to find the next number.
We can combine these two ideas into
one to form.
1.
Arithmetico - Geometric Series: As
the name suggests in this series each successive term should be found by first adding a fixed
number to the previous term and then multiplying it by another fixed number.

Eg:

1, 6, 21, 66, 201, -------Here each successive term is obtained


by first adding 1 to the previous term and then
multiplying it by 3. The differences of successive numbers should be in Geometric progression.
2.
Geometrico-Arithmetic Series :- As
the name suggests, a geometrico - Arithmetic
series should be the one in which each successive term is found by first multiplying or dividing
the previous term by a fixed number and then
adding or deducting another fixed number.
Eg:
3, 4, 7, 16, 43, 124 -----Here each successive term is obtained
by first multiplying the previous number by 3 and
then subtracting 5 from it.
The differences of successive numbers
should be in geometric progression.
(iv)
Twin Series: As the name suggests,
a twin series are two series packed in one.
Eg: 1, 3, 5, 1, 9, -3, 13, -11, 17, ----Here the first, third, fifth, etc. terms are
1, 5, 9, 13, 17, which is an arithmetic series.
The second, fourth, sixth, etc. are 3, 1, -3, -11
which is a geometrico -arithmetic series in which
successive terms are obtained by multiplying
the previous term by 2 and then subtracting 5.
(v)
Other series: Besides, numerous other
series are possible and it is impossible to even
think of all of them. It is only through a lot of
practice and by keeping abreast with the latest
trends that one can expect to master the series.
Steps for Solving Series Questions:Despite the fact that it is extremely difficult to lay down all possible combinations of
series, still, if you follow the following step-bystep approach, you may solve a series questions easily and quickly.
Step 1 : Preliminary Screening:
First check the series by having a look
at it. It may be noted that the series is very simple and just a first look may be enough and you
may know the next term.

Step II. Check Trend : Increasing/Decreasing


Alternating
If you fail to see the rule of the series
by just preliminary screening you should see
the trend of the series. By this we mean that
you should check whether the series increases
continuously or decreases continuously or
whether it alternates ie., increases and decreases alternating.
Step III : Use the following rules:
1.
If the rise of series is slow or gradual,
the series is likely to have an addition - based
increase, successive numbers are obtained by
adding some numbers.
2.
If the rise of a series is very sharp initially but slows down later on, the sereis is likely
to be formed by adding squared or cubed numbers
3.
If the rise of a series is throughout
equally sharp, the series is likely to be multiplication-based; successive terms are obtained by
multiplying by some terms (and, maybe, some
addition or subtraction could be there, too)
4.
If the rise of a series is irregular and
haphazard there may be two possibilities. Either there may be a mix of two series or two
different kinds of operations may be going on
alternatively. The first is more likely when the
increase is very irregular, the second is more
likely when there is a pattern, even in the irregularity of the series.
Finding wrong Numbers is a series
In today's examinations, a series is likely to be
given in format of a complete series in which an
incorrect number is included. The candidate is
required to find out the wrong number.
Obviously, finding the wrong number in a series
is very easy once you have mastered the art of
understanding how the sereis is likely to be
formed. On studying a given series and applying the concepts employed so far you should
be able to understand and thus decode the formation of the series. This should not prove very
difficult because usually six terms are given and
it means that at least five correct terms are given.
This should be sufficient to follow the series.

PRACTICE TEST
Direction (Qs. 1-10) : Each of the following number series contains a wrong number. Find out that
number.
1.
527
318
237
188
163
154
150
(1) 318 (2) 237 (3) 188 (4) 163 (5) 154
2.
14
40
77
229
455
1367 2723
(1) 40
(2) 77
(3) 229 (4) 455 (5) 1367
3.
20250
3375
1350
225
75
15
6
(1) 3375
(2) 1350
(3) 225
(4) 75
(5) 15
4.
80
81
83
90
100
115
136
(1) 81
(2) 83
(3) 90
(4) 100
(5) 115
5.
8
15
50
250
1100
5475
27350
(1) 15
(2) 50
(3) 250
(4) 1100
(5) 5475
6.
70.21
71.49
71.81
70.85
72.13
71.17
(1) 71.49
(2) 71.81
(3) 70.85
(4) 72.13
(5) 70.56
7.
10
21
64
255
1286
7717
54020
(1) 21
(2) 255
(3) 64
(4) 1286
(5) 7717
8.
3
4.5
9
22.5
67.5
270
945
(1) 4.5
(2) 9
(3) 22.5
(4) 67.5
(5) 270
9.
376
188
88
40
16
4
-2
(1) 188
(2) 88
(3) 40
(4) 16
(5) 4
10. 5
6
10
37
56
178
(1) 6
(2) 56
(3) 37
(4) 10
(5) 178
Directions (Q. 11- 20) in each of the following questions, a number series is established if the positions of two out of the five marked numbers are interchanged. The position of the first unmarked
number remains the same and it is the beginning of the series. The earlier of the two marked numbers
whose positions are interchanged is the answer. For example, if an interchange of the number marked
'1' and the number marked '4' is required to establish the series, your answer is '1'. If it is not necessary to interchange the position of the numbers to establish the series, give 5 as your answer. Remember that when the series is established, the numbers change from left to right (ie from the unmarked number to the last marked number) in a specific order.
11.
2
11
59
227
697
1369
(1)
(2)
(3)
(4)
(5)
12. 379
500
591
556
267
331
(1)
(2)
(3)
(4)
(5)
13. 0
1
-2
68
21
465
(1)
(2)
(3)
(4)
(5)
14. 11880
1225.125 10890
1089
9801
990
(1)
(2)
(3)
(4)
(5)

15.

25

26
710
175
56
3563
(1)
(2)
(3)
(4)
(5)
16. 192
24
28
168
140
35
(1)
(2)
(3)
(4)
(5)
17. -1
0
-2
-15
-6095
-236
(1)
(2)
(3)
(4)
(5)
18. 739
547
635
106
10
186
(1)
(2)
(3)
(4)
(5)
19. 0
0.25
1.50
6.75
31
161.25
(1)
(2)
(3)
(4)
(5)
20. 714
125
2
9
0
20
(1)
(2)
(3)
(4)
(5)
Directions (Qs. 21-30): In each of the following questions a number series is given. After the series, a
number is given below it, followed by A,B,C,D and E. You have to complete the series starting with the
number given and following the same property as in the given number series. Then answer the questions below it.
21. 123
149
182
224
277
321
A
B
C
D
E
Find the value of E
(1) 532
(2) 558
(3) 528
(4) 545
(5) None of these
22. 1
5
14
39
88
4
A
B
C
D
E
What should replace B?
(1) 43
(2) 17
(3) 34
(4) 40
(5) None of these
23. 2520
280
2240
320
1920
384
504
A
B
C
D
E
What should come in the place of C.
(1) 448
(2) 384
(3) 74
(4) 120
(5) 64
24. 6
9
21
39
83
163
333
4
A
B
C
D
E
Find the value of D
(1) 61
(2) 51
(3) 57
(4) 49
(5) None of these
25.

659
130
491
266
1009
A
B
C
What should come in the place of (D)
(1) 616
(2) 737
(3)
6
3
1
-4
7
A
B
C
Find the value of C.

26.

387
D
762
-25
D

338
E
(4)
E

726

(5) None of these

(1) -6
(2) -3
(3) -2
(4) 4
(5) None of these
27. 7
42
48
336
343
3
A
B
C
D
E
What is the value of D?
(1) 181
(2) 175
(3) 167
(4) 168
(5) None of these
28. 198
166
144
130
122
263
A
B
C
D
E
What should replace E?
(1) 183
(2) 178
(3) 182
(4) 180
(5) None of these
29. 8
17
44
125
368
4
A
B
C
D
E
Find the value of B.
(1) 8
(2) 42
(3) 22
(4) 9
(5) 39
30. 4
10
17
49
95
6
A
B
C
D
E
What is the value of C?
(1) 81
(2) 83
(3) 87
(4) 85
(5) None of these
Directions (Qs. 31-40): One number is wrong in each of the number series given in each of the
following questions. You have to identify that number and assuming that a new series starts with that
number following the same logic as in the given series, which of the numbers given in (1), (2), (3), (4)
and (5) given below each series will be third number in the new series?
31. 3
5
12
38
154
914
4634
(1) 1636
(2) 1222
3) 1834
(4) 3312
(5) 1488
32. 3
4
10
34
136
685
4116
(1) 22
(2) 276
(3) 72
(4) 1374
(5) 12
33. 214
18
162
62
143
90
106
(1) -34
(2) 110
(3) 10
(4) 91
(5) 38
34. 160
80
120
180
1050
4725
25987.5
(1) 60
(2) 90
(3) 3564
(4) 787.5 (5) 135
35. 572
4600
576
4032
672
3352
(1) 3371
(2) 3375
(3) 26397
(4) 4399.5 (5) None of these
36. 7
14
42
165
840
5040
(1) 330
(2) 165
(3) 990
(4) 3960
(5) None of these
37. 72
143
287
570
1147
(1) 4557
(2) 2289
(3) 1139
(4) 573
(5) None of these
38. 1
5
21
57
120
221
(1) 140
(2) 120
(3) 124
(4) 176
(5) None of these
39. 5
17
27
60
115
(1) 247
(2) 501
(3) 127
(4) 60
(5) None of these

40.

10
6
15
120
1879
(1) 116
(2) 120
(3) 4079
(4) 455
(5) None of these
Directions (Qs. 41-50): In each of the following questions a number series is given. A number in the
series is suppressed by letter 'A'. You have to find out the number in the place of 'A' and use this
number to find out the value in the place of the question mark in the equation following the series.
41. 36
216
64.8
388.8
A
699.84
209.952
A 36 = ?
(1) 61.39
(2) 0.324
(3) 3.24
(4) 6.139 (5) 32.4
42. 42
62
92
132
A
242
312
A + 14 = ? x 14
(1)

11

6
7

(2) 14

43.

44.

A2 4 ?
(1) 1365
18
24
A

(1)

45.

(3)

12

12

19

(2) 1353
A

5
7

28

(3)

1505
72

(3)

12

51

(4)

12 1 2

(5) 12 1 6

A 52

(4) 1435
(5) 1517
98
129

3 4
?
7 5

(2) 11

12

3
8

3
4

23
35

9
16

9
8

12
35
27
32

(4)

14

2
5

27
16

(5) 10

2
7

A ?
(1)
46.

47.

3
2

(2)

3
6
2A + 5 = ?
(1) 35
5
8

6
8

7
(2) 30
13

(3)

6
4

(4)
11

(3)

37

29

3
4

13

(5)

9
8

18

(4) 25
40

(5) 15
53

A2 2 A ?
48.

(1)
1

380
A

(2) 400
4

(3)

440
15

(4) 360
60

(5) 200
64

(3)

12

(4) 20
83

(5) 8

12

A2 3 A 7 ?
49.

(1)
A

2
27

(2) 17
38

51

66

50.

A2 A3 ?
(1) 40
2
5
5A

4A
?
2

(1)

60

(2) 27
9

(3)

30

14

(2) 20

(3)

100

Answers:
1.

(1)

13 2 , 112 , 9 2 , 7 2

2.

(5)

3 2, 2 3, 3 2, 2 3,

3.

(4)

6, 2.5, 6, 2.5,

4.

(2)

A three tier sereis

5.

(3)

5 52 , 5 52 ,

6.

(1)

128
. , 0.96, 128
. , 0.96,

7.

(2)

2 1, 3 1, 4 1, 5 1,

8.

(5)

15
. , 2, 2.5, 3, 35
. ,

9.

(1)

192, 96, 48, 24, 12, 6,

10.

(2)

13 , 22 , 3 3 , 4 2 , 5 3 ,

11.

(5)

6 12 , 5 2 2 , 4 32 , 3 4 2

12.

(2)

112 , 13 2 , 15 2 , 17 2 ,

Replace (2) with (5)


13.

(3)

1 13 , 2 2 2 , 3 32 , 4 4 2 ,

Replace (3) with (4)


14.

(1)

12, 11, 10, 9

Replace (1) with (5)


15.

(2)

1 1, 2 4, 3 7, 4 10,

Replace (2) with (4)


16.

(2)

8, 7, 6, 5,

Replace (2) with (3)


17.

(4)

12 1, 2 2 21, 32 3,

(4) 18
27

(5) 15

(4)

(5) 50

70

Replace (4) with (5)


18.

(1)

27 2 , 25 2 , 23 2 , 212 ,

Replace (1) with (4)


19.

(5)

1 0.25 12 , 22 0.25 2 2 , 3 0.25 32

20.

(2)

6 ! 6, 5 ! 5, 4 ! 4, 3 ! 3

21.

Replace (2) with (5)


(5)

22.

(2)

4, 9, 25, 49
27,

23.

(5)

9, 8, 7, 6,

24.

(2)

2 3, 2 3, 2 3, 2 3,

25.

(2)

23 2 , 19 2 , 15 2 , 112

26.

(5)

1 3, 2 5, 3 7, 4 9,

27.

(2)

6 6, 7 7, 8 8,

28.

(1)

32, 22, 14, 8, 4,

29.

(1)

3 7, 3 7,

30.

(4)

3 2, 2 3, 3 2, 2 3,

31.

(3)

1 2, 2 2, 3 2, 4 2,

32.

(3)

1 1, 2 2, 3 3,

33.

(4)

14 2 , 12 2 , 10 2 , 8 2 , 6 2 ,

34.

(5)

0.5, 15
. , 2.5,3.5,

35.

(3)

36.

(3)

2, 3, 4, 5,

37.

(2)

2 1, 2 1, 2 1, 2 1

38.

(1)

22 , 4 2 , 6 2 , 8 2 ,

39.

(1)

2 7, 2 7, 2 7, 2 7,

40.
46.

(4)
(1)

41 (3)
47. (4)

42 (2)
48. (2)

43. (5)
49. (3)

44. (1)
50. (1)

45. (5)

BANK PROBATIONARY OFFICER


QUANTITATIVE APTITUDE
PROBABILITY
If an event can happen in 'a' ways and fail
in 'b' ways, and each of these ways is equally
likely, then probability or the chance, or its happening is

b
a
, and that of its failing is
a b .
a b

e.g., If in a lottery there are 7 prizes and 25


blanks, the chance that a person holding 1 ticket

7
will win a prize is
, and his chance of not
32
25
winning is
.
32
If p is the probability of the happening
of an event, the probability of its not happening
is 1 p .
Instead of saying that the chance of the
happening of an event is

a
a b , it is sometimes

stated that the odds are 'a' to 'b' in favour of the


event, or 'b' to 'a' against the event.
If c is the total no. of cases, each being
equally likely to occur, and of these 'a' are
favourable to the event, then the probability that

a
, and the probability
c
a
that it will not happen is 1 .
c
the event will happen is

ex.1. What is the chance of throwing a number


greater than 4 with an ordinary die whose
faces are numbered from 1 to 6.
Sol: There are 6 possible ways in which the
die can fall, and of these two are
favourable to the event required.

2 1
required chance .
6 3

ex.2. From a bag containing 4 white and 5 black


balls a man draws 3 at random, what are
the odds against these being all black?
Sol: The total no. of ways in which 3 balls can
be drawn is 9 C3 and the no. of ways of
drawing 3 black balls is 5 C3 ; therefore
the chance of drawing 3 black balls is
5
9

C3 5.4.3 5

C3 9.8.7 42

Thus the odds against the event are 37


to 5.
ex.3. Find the chance of throwing at least one
ace in a simple throw with two dice.
Sol: The possible no. of cases is 6 6 or 36 .
An ace on one die may be associated
with any of the 6 numbers on the other die, and
the remaining 5 numbers on the first die may be
associated with the ace on the second die, thus
the number of favourable cases is 11.

Required chance

11
36

ex.4. Find the chance of throwing more than


15 in one throw with 3 dice.
Sol: A throw amounting to 18 must be made
up of 6, 6, 6 and this can occur in 1 way,
17 can be made up of 6, 6, 5 which can
occur in 3 ways, 16 may be made up of
6, 6, 4 and 6, 5, 5 each of which arrangements can occur in 3 ways.
Therefore the no.of favourable cases is =
1+ 3 + 3 + 3 = 10 and the total number of
cases is 63 216 required chance

10
5

216 108

ex.5. What is the probability that a digit selected at random from the logarithmic
table is (i) 1, (ii) 3 or 7.

will win is

heat is impossible, find the chance


that one of them will win.

Sol: Various digits in the log table are 0, 1, 2,


3, 4, 5, 6, 7, 8, 9, i.e. a total of 10 digits
is used.
(i) The number of favourable cases for getting 1, out of 10 all equally likely cases is
one Prob. of getting 1 is

Sol:

1
.
10

will win the race is p1 p2 p3 i.e.

1 1 1 47


.
6 10 8 120

(i) repetitions are not allowed.

ex.8.

Two balls are to be drawn from a bag


containing 5 red and 7 white balls; find
the chance that they will both be white.

Sol:

Here any one pair of balls is as likely


to be drawn as any other pair. The

P3 9 8 7 504 . Of these,

total number of pairs is

favourable are two, i.e. 563 and 169.


So the required probability is

C2 , and the

is 7 C2 , .
The required chance is therefore

7.6 12.11 7

1.2 1. 2
22 .

(ii) When repetitions are allowed: Out of


given 9 digits, 3 digit numbers can be
formed in 9 9 9 729 ways.
Of these two are favourable cases,

ex.7.

12

number of pairs which are both white

2
1

504 252

Probability

1
and that C wins
10

sible, these are mutually exclusive


events, so the chance that one of them

ex. 6. Find out the probability of forming 563 of


169 with the digits 1, 2, 3, 4, 5, 6, 7, 8, 9
when only numbers of three digits are
formed and when

Sol: (i) when repetitions are not allowed:


The total ways of forming numbers of
three digits with 9 given digits is

1
, that
6

1
( p3 ) . As a dead heat is impos8

2
1
.
10 5

(ii) repetitions are allowed.

Probability that A wins ( p1 )


B wins ( p2 )

(ii) The no. of favourable cases for getting 3


or 7 is 2. Required Prob.

1
. Assuming that a dead
8

2
729

In a race where 12 horses are running,


the chance that horse A will win is

1
1
, that B will win is
and that C
6
10

ex.9.

Find the chance of drawing 2 white


balls in succession f rom a bag
conatining 5 red and 7 white balls, the
balls drawn not being replaced.

Sol:

The chance of drawing a white ball the


first time is

7
; and, having drawn a
12

white ball the first time, there will be


5 red and 6 white balls left, and

therefore the chance of drawing a white

contains only one red ball, the chance

6
ball the second time will be
.
11

being

Hence the chance of drawing two


white balls in succession will be

EXERCISE
1.

7
6 7

.
12 11 22
ex. 10.

Sol:

There are two bags, one of which contains 5 red and 7 white balls and the
other 3 red and 12 white balls, and a
ball is to be drawn from one or other
of the two bags; find the chance of
drawing a red ball.

1
6
and
respectively..
11
11

From a pack of 52 cards, two are drawn


at random. Find the chance that one is a
knave and the other a queen.
a) 6 663

b)

c)

d)

52

663

663
663

e) None
2.

A bag contains 5 white, 7 black, and 4


red balls. Find the chance that three balls
drawn at random are all white.

The chance of choosing the first bag

1
is , and if the first bag be chosen
2

a)

1
56

b)

c)

d)

the chance of drawing a red ball from

e) None of these

it is

5
; hence the chance of draw12

3.

ing a red ball from the first bag is

1
5
5

.
2 12 24

Similarly the

chance of drawing a red ball from the


second bag is

1
3
1

.
2 15 10

56

What is the chance of throwing an ace


in only the first of two successive throws
with an ordinary dice.
a)

c)

36

b)

36

d)

36
36

e) None of these
4.

Hence as these events are mutually


exclusive, the chance required is

ex. 11.

56

56

Three cards are drawn at random from


an ordinary pack; find the chance that
they will consist of a knave, a queen and
a king.

5
1
37

.
24 10 120

a)

In two bags there are to be put altogether 2 red and 10 white balls, neither bag being empty. How must the
balls be divided so as to give a person
who draws one ball from either bag.

c)

11
5525

15
5525
e) None of these

5.

b)

12
5525

d)

16
5525

If 8 coins are tossed, what is the chance


that one and only one will turn up head?

Sol: (i) the least chance and (ii) the greatest


chance of drawing a red ball.

a)

1
32

b)

(i) The least chance is when one bag contains only one white ball, and the
greatest chance is when one bag

c)

d)

32

e) None of these

32
32

6.

In a certain town, the ratio of males to


females is 1000 : 1987. If this tendency
is expected to continue, what is the
chance that a newly born baby is male?
a)

1000

c)

1987

1987

b)

1000

d)

1000

Answers:
1. (b)

2. (a)

3. (c)

4. (d)

5. (a)

6. (a)

7. (b)

8. (d)

9. (a)

10. (a)

2987

INEQUALITIES AND MAXIMA & MINIMA


2987

1000

e) None
7.

What is the chance that a leap year, selected at random, will contian 53
sundays?
a)

1
7

b)

c)

d)

Zero must be regarded as greater than any -ve


quantity and less than any +ve quantity.

RULE 1- An inequality will still hold after each


side has been increased, diminished, multiplied
or divided by the same +ve quantity.

Out of all the integers from 1 to 100, a


number is selected at random. what is
the probability that the selected number
is not divisible by 7 ?
a)

40

c)

42

50

b)

41
50

50

d)

43

50

e) None of these
9.

100 students appeared for two examinations. 60 passed the first, 50 passed the
second and 30 passed both. Find the
probability that a student selected at random has failed in both the examinations.
a)

1
5

b)

c)

d)

5
5

e) None
10.

A child is asked to pick up two balloons


from a box containing 10 blue and 15 red
balloons. What is the probability of the
child picking at random two balloons of
different colours ?
a)

1
2

b)

c)

1
4

d)

Also, b is said to be less than a when b-a is -ve.


Thus - 5 less than - 2, because - 5 - (-2)= -3,
which is -ve.

e) None
8.

Any quantity is said to be greater than another


quantity b when a - b is positive. Thus, 2 is
greater than -3 as 2 - (-3) = 5 is +ve.

i.e.,

if a > b,
a+c>b+c
a-c>b-c
ac > bc c is positive

a b

c c
RULE 2 - In an inequality any term may be
transposed from one side to the
other if its sign is changed. i.e. if a c> b, then a> b+c, or -c>b-a.
RULE 3 - If the sides of all the terms of an
inequality be changed, the sign of
the inequality must be reversed. i.e.
if a>b, then b < a.
RULE 4 - If the signs of all the terms of an
inequality be changed, the sign of
the inequality must be reversed. ie.
if a> b, then -a< -b or -ac < -bc,
where c is +ve.
RULE 5 -

If the sides of an inequality be


mutiplied by the same -ve quantity,
the sign of the inequality must be
rev ersed. i.e. if a > b, then
-ac < -bc; where c is +ve.

RULE 6-

If a>b, then

1
3
1
5

a n b n , and

RULE 7 -

1 a n 1 b n or a n b n ; if n is

then

a +ve quantity.

a m b m c m ..... k m a b c ..... k

n
n

The square of every real quantity is


+ve and therefore must be greater
than zero.
2

unless m is a positive proper fraction.


RULE 11-

i.e. ( a b) 0; a b zab; Similarly,,


xy
xy , x 0, y 0
2

Hence the arithmetic mean of two +ve quantities


is greater than their geometric mean.
RULE 8 - If the sum of two +ve quantities is
given, their product is greatest when
they are equal; and if the product of
two +ve quantities is given, their sum
is least when they are equal.

RULE 12-

It a, b, c are +ve and not all equal,


then (a + b + c) (ab+ bc + ca) >
9 abc and, (b + c) (c + a) (a + b) >
8 abc.

ax
a
OR
b x
b according as
a < OR > b is x be positive or
according as a> OR <b if x is
negative.

RULE 13-

RULE 9 - If a, b, c, .......k are n unequal


quantities, then,

a c e ......
b d f ...... is less than the
greatest and greater than the least,

a b c ...... k

a. b. c. d ....... k

i.e.

a b c ......... k
( a. b. c. ..... k )
n

Therefore , the arithmetic mean of any number


of +ve quantities is greater than their geometric
mean.
RULE 10- If a and b are positive and unequal,

of the fractions

RULE 14- If a>x , b>y, c>z then


a + b + c + .....> x + y + z +.... and
abc......> x y z........
RULE 15-

a 2 b 2 c 2 bc ca ab .

RULE 16 -

| n2

a m bm a b

2 , except
2

a c e
, , ,.......
b d f

nn .

RULE 17- For any positiv e integer n

1
2 (1 ) n 3
n

when 'm' is a position proper fraction.


If m is a positive integer or any
m m

negative quantity
If

is

positiv e

a m bm a b
,

2
2

and

a b
a b

2
2
less

than1,

If there are n positive quantities a, b, c, ......k,

RULE 18 - a 2 b b 2 c c2 a 3abc

RULE 19 -

a b c d
4
b c d a

RULE 20 - a 4 b 4 c 4 d 4 4 abcd
ex.1.

W hich of

(1000001
.
)

the two number


1000000

and 2 is greater?

(1. 000001)1000000 (1

Sol:

1
)1000000
1000000

Solve 2 x 4

Sol:

2x

ex. 6.

Solve (-2x + 3) 6
3
2 x 3 x
2
Solve ( x 3) ( x 4) 0

Which is greater than 2. (RULE 17).


ex. 2.

which of the two numbers 10001000


and 1001999 is greater ?
999

1001

Sol:

10001000

1001

1000

1000

1000

which is 3 .

1000

1001

ex. 7.

20
8
4
4 2 x x
3
3
3

( x 3) ( x ( 4) 0;

1
1001

20

ex. 5.

x does not lie between - 4 and


3.

1
1
1001

Solve x 2 8 x 7 0

ex. 8.

( x 1) ( x 7) 0 ;

10001000 1001999

[ x ( 1)] [ x ( 7)] 0
ex.

3.

Find that min value of x 4 x 7


f or
real
v alues
of
x.
2

x lies between - 7 and -1.


Slove x 2 8 x 7 0

ex. 9

x 4 x 7 ( x 2) 3
Sol:

A perfect square is always positive,


i.e., it cannot be less than zero.
the given expression is least when

( x 7) ( x 1) 0; x does not lie


between 1 and 7.
ex. 10.

( x 2) 2 0

ex . 4

Sol:

If w satisfies both the following in


equalities, and w is an integer, what
values can w have ?

min: value = 3

(i)

5 ( w 10) 4 w 0

Find the maximum v alue of


35
. 4 x 4 x 2 for real values of x .

(ii)

8 7 w 3 (2 w 1)

35
. 4 x 4 x 2 4.5 (1 4 x 4 x 2 )

Sol:

8 7 w 6w 3 ;w 5 w

4.5 (1 2 x ) 2

lies between -50 and -5.

the given expression is maximum


when (1 2 x ) 2 is least. ie, when

1 2x 0 .
maximum value = 4.5 and this

occurs when x

5w 50 4 w 0 ; w 50

1
.
2

ex. 11.

2
x
P 22 ;
3
3
Q 12 2 x 8 x
R 10 4 x 14 ;
S 0, 1, 2, 3, 4, 5

If x is an integer, list the members of the


following sets;
(a)

PS

(b)

QS

(c)

RS

(d)

PQS

(e)

( P R) S

Sol:

P:

5
5
but 7; x
+ve and
2
2
7 x is ve . y is positive.

When x

when x 7, x

x
2
x 2
2 2 i.e.
3
3
3 3
P 3, 4, 5.....

Q:

y is negative

12 2 x 8 x i. e. x 4; x 4,
Q [3, 2, 1, 0, 1,.....]

is -ve;

5
is +ve and 7-x
2

y is -ve.

The given expression is positive only as long


1
as x is between 2 and 7.
2
ex. 13. Find the greatest v alue of

(a x ) 3 (a x ) 4 for any real value

R 10 4 x 14 i. e. 4 x 4, x 1,

of x.

R [ 0, 1, 2, 3, .....]

The given expresssion is greatest


when:

Therefore, P S [ 3, 4 , 5]

ax ax

3
4

Q S [ 0, 1, 2, 3]

( P R ) S [ 0, 1, 2, 3, 4, 5]
(b) 0,1,2,3

(c)

0,1,2,3,4,5

(d) 3

(e)

0, 1, 2, 3, 4, 5.

ex. 12.

19 x 2 x 2 35

positive?
Sol:

[Note : a m b n c p ........... will be


greatest when the f actors

a b c
.... ]
m n p

Between what values of x, is the


expression

Let y denote the given expression,

y (2 x 2 19 x 35)

(2 x 5) ( x 7)

Remember : In order that a x 2 bx c may


be always +ve, b 2 4ac must be ve or 0, and a must be +ve. In order
that ax 2 bx c may be always
negative, b 2 4ac must be -ve or
0 and a must be -ve.
ex. 14

5
(2 x 5) ( 7 x ) 2( x ) ( 7 x )
2

when x

5
5
;x
is -v e and
2
2

(7 - x) is +ve,

a
.
7

63 . 84 7
a .
77

P Q S [ 3]
3, 4, 5

Thus the greatest v alue is

R S [ 0, 1, 2, 3, 4, 5]

(a)

OR

Find the greatest v alue of

x2
2 x 3x 6 for real values of x.
2

Sol:

Let

x2
2

2x 3 x 6

then

The Second f actor is always

2 yx 2 (3 y 1) x 6 y 2 0 .
If

is

positive, hence x 3 16 x is greate

real,

if x 1 .

(3 y 1) 2 8 y (6 y 2) must be
positive (1+13y) (1 3 y) must be
+ve. Hence y must lie between

1
3

1
and
and its greatest value is
13
1
.
3
ex. 15:

EXERCISE
1.

(5 x ) ( x 3)

2.

Find the least value of 3x 4 y if

Let A

a) 16

b) 16

c) 18

d) 20

e) None of these

If x be real, find the max. and min. values

x2 x 1
of 2
x x 1

x 2 y 3 6, x and y are positive.


Sol:

If x be positive, find the greatest values of

3
4
x ; B y;
2
3

a) 2, 12

b) 3, 13

c) 4, 14

d) 4, 15

d) 5, 15

e) None of these

x2 y 3

4 A 27 3
.
B 6
9
64

3.

A2 B 3 32
Now, 3x + 4y = 2A + 3 B. also,
A.A.B.B.B = 32. The least value of
A+A+B+B+B will be so when all the
quantities are equal. (RULE 8).

real values of x.

4.

A B 2; Least v alue of
3x 4 y 2 2 3 2 10 .
ex. 16.

If x may have any real value, find


which is greater, x 3 16 x OR

7 x 2 10 .
Sol:

x 3 16 x 7 x 2 10 has a factor
x-1.

a)

1
3

b)

c)

d)

e) None

1
Find the min. value of x .
x
a) 2

b) 3

c) 4

d) 5

e) None
5.

Find the greatest value of x 2 y 2 when

x 3 y 6c.
a)

49 4
c
16

36 4
c
16
Answers:

c)

x 3 7 x 2 16x 10

1
1 x x 2 for

Find the max. value of

( x 1) ( x 2 6 x 10)

1. (b)

( x 1)[( x 3) 2 1]

5. (d)

2. (b)

b)

25 4
c
16

d)

81 4
c e) None
16

3. (c)

4. (a)

Bank Probationery Officer


Quantitative Aptitude
DATA

INTERPRETATION

Data Interpretation is one of the easy


sections of one day competitive Examinations.
It is an extension of Mathematical skill and
accuracy. Data interpretation is nothing but
drawing conclusions and inferences from a
comprehensive data presented numerically in
tabular form by means of an illustration, viz.
Graphs, Pie Chart etc. Thus the act of
organising and interpreting data to get meaningful information is Data Interpretation.
A good grasp of basic geometric as well
as arithmetic formulae is must to score high
in this section. Familiarity with graphical representation of data like Venn diagrams, graphs,
pie charts, histogram, polygon etc. should be
thought. Once the data are grasped well, questions based on tables and graphs take little
time.
In some competitive examinations data
are presented in more than one table or graphs.
The aim is to test not only quantitative skill
but also relative, comparative and analytical
ability. The crux of the matter is to find a relationship between the two tables or graphs
before attempting the questions.
Some Useful tips:
1 . Data Interpretation questions are based
on information given in tables and graphs.
These questions test your ability to interpret the information presented and to
select the appropriate data for answering a question.
2 . Get a general picture of the information
before reading the question. Read the
given titles carefully and try to understand its nature.
3 . Avoid lengthy calculations generally, data
interpretation questions do not require to
do extensive calculations and computations. Most questions simply require reading the data correctly and carefully and

putting them to use directly with common sense.


4 . Breakdown lengthy questions into smaller
parts and eliminate impossible choices.
5 . Use only the information given and your
knowledge of everyday facts, such as the
number of hours in a day, to answer
the questions based on tables and
graphs.
6 . Answer the questions asked and not what
you think the questions should be.
7 . Be careful while dealing with units.
8 . To make reading easier and to avoid errors observe graphs keeping them
straight.
9 . Be prepared to apply basic mathematical
rules, principles and formulae.
1 0 . Since one of the major benefits of graphs
and tables is that they present data in a
form that enables you to readily make
comparisons, use this visual attribute of
graphs and tables to help you answer the
questions. Where possible, use your eyes
instead of your computational skills.
Tables
Tables are often used in reports, magazines and newspaper to present a set of
numerical facts. They enable the reader
to make comparisons and to draw quick
conclusions. It is one of the easiest and
most accurate ways of presenting data.
They require much closer reading than
graphs of charts and hence are difficult
and time consuming to interpret.
One of the main purposes of tables is to
make complicated information easier to
understand. The advantage of presenting
data in a table is that one can see the
information at a glance.
While answering questions based on

tables, carefully read the table title and


the column headings. The title of the table
gives you a general idea of the type and
often the purpose of the information presented. The column headings tell you the
specific kind of information given in that
column. Both the table title and the column headings are usually very straight
forward.
Graphs
There may be four types of graphs.
1)

Circle Graphs: Circle graphs are used to


show how various sectors are in the
whole. Circle graphs are sometimes
called Pie Charts. Circle graphs usually
give the percent that each sector receives
In such representation the total quantity
in question is distributed over a total
angle of 360.
While using circle graphs to find ratios
of various sectors, don't find the
amounts each sector received and then
the ratio of the amounts. Find the ratio
of the percents, which is much quicker.

2)

3)

4)

Line Graphs: Line graphs are used to


show how a quantity changes continuously. If the line goes up, the quantity is
increasing; if the line goes down, the quantity is decreasing; if the line is horizontal, the quantity is not changing.
Bar Graphs: Given quantities can be compared by the height or length of a bar
graph. A bar graph can have either vertical or horizontal bars. You can compare
different quantities or the same quantity
at different times. In bar graph the data
is discrete. Presentation of data in this
form makes evaluation of parameters
comparatively very easy.
Cumulative Graphs : You can compare
several catagories by a graph of the
cumulative type. These are usually bar
or line graphs where the height of the
bar or line is divided up proportionally
among different quantities.

SOLVED EXAMPLES
I.

Directions (Qs. 1-5) study the following


table and answer the questions given
below it.

Production of sugar by six major production


units of India in Million Tonnes
PRODUCTION UNITS

Month

April

310

180

169

137 140 120

May

318

179

177

162 140 122

Jun e

320

160

188

173 135 130

July

326

167

187

180 146 130

August

327

150

185

178 145 128

1 . In which month the unit B has a contribution of approximately 15% in the total sugar
production?
1)

August

2) Jun e

3)

July

4) April

2 . Which of the following units shows continuous increase in production of sugar over
months?
1)

2) B

3)

4) D

3 . In the case of Unit E, in which of the following pairs of months the production of sugar
was equal?
1)

April & June

2) June & July

3)

July & August

4) April & May

4 . In the month of June, how many units


have a share of more than 25% of the total
production of sugar?
1)

one

2) Three

3)

Two

4) Four

5 . What was approximate percentage decrease in sugar production of unit B in June as


compared to April?
1)

8%

2) 1 0 %

3)

15%

4) 1 8 %

Ans:
1.

20
180 160
x 100
100
=
180
180

(3) Total production in April


= 310+180+169+137+140+120=1056
15% of 1056 =

= 11.11% 10%
II. Directions (Qs. 6-10): Study the following
graph carefully and answer the questions given
below it:
A
1 2 0 0 __________________________________

15
x 1056 158.4
100

Total Production in June


320+160+188+173+135+130

1106

15% 1106 =

15
x 1106 165.9
100

Total production in July


=

326+167+187+146+130=1136

15% of 1136 =

15
x 1136 170.4
100

Total Production in August

B
1 1 5 0 __________________________________
1 1 0 0 __________________________________
PRICE IN RS

1 0 5 0 __________________________________
1 0 0 0 __________________________________
9 5 0 __________________________________
9 0 0 __________________________________
8 5 0 __________________________________
8 0 0 __________________________________
Jan

= 327+150+185+178+145+128= 1113

Feb Mar Apr May June July Aug

MONTHS
15
x 1113 166.95 167
15% of 1113 =
100

ie, in Month July, the Unit B has a contribution of approximately 15% in the total sugar
production.
2 . (1) Unit A shows continuous increase in
production of sugar over months.
3 . (4) In Unit E, the pair of months that shows
equal sugar production is April and May.
4 . (1) Total Production in June =
320+160+188+173+135+130
=
1106
25% of 1106=

25
x 1106 276.5 units
100

ie, Unit A shows more than 25% of the


total production of sugar in the month June.
5 . (2) In unit B, the production in June =
160 the production in April = 180 units
Decrease in percentage

6) What was the price difference between


commodity A and B in the month of April?
1)

250

2) 1 5 0

3) 1 0 0
4) 9 0
7 . What was the difference in average price
between commodity A and B from April to
August?
1)

86

2) 7 5

3)

95

4) 8 5

8 . In which of the following pairs of months


was the price of commodity A same?
1) January - March 2) May-June
3) April- August
4) July-August
9 . What was the approximate percentage
decrease in the price of commodity A from
March to April?
1) 1
2) 9
3) 1 4
4) 1 2
1 0 . What was the percentage increase in
price of commodity B from January to April?
1) 1 5
2) 2 0
3) 1 7
4) 1 0

Ans 6 (2).The price of commodity A in April


= 1000
The price of commodity B in April
= 1150

III Directions (11-15): Study the following


graph carefully and answer the questions given
below it.
PRODUCTION OF FOODGRAIN OVER THE
YEARS (1000 TONNES)

Difference = 1150 - 1000 = 150

130
120

7) (b). Average price of commodity A from

110

April to August
1000 1050 975 900 1000
=
5

4925
985
5

Average price of commodity B from


April to August
=

1150 1100 1100 1000 950


5

5300
1060
5

Difference = 1060 - 985 = 75


8)(3) Price of commodity A in April = 1000
Price of commodity A in August = 1000
Therefore, in April - August the price of
commodity A was same.
9)(4) Price of commodity A in March = 1125
Price of commodity A in April = 1000
Decrease
125
1125 1000
x 100
100
=

1125
1125

= 11.11% 12%
10)(1)Price of commodity B in January= 1000
Price of commodity B in April = 1150
1150 1000
100
Increase =

1000

150 x100
15%
1000

100
80
65

1993

1994 1995

1996 1997

1998

11) In the case of how many years was the


production below the average production of
the given years?
1)

one

2) t w o

3)

three

4) four

12) What was the percentage drop in production from 1996 to 1997?
1)

100

2) 5 0

3)

65

4) 4 0

13) In which year was the production 50%of


the total production in 1993 and 1998 together
1)

1994

2) 1 9 9 5

3)

1996

4) 1 9 9 7

1 4 . If the production in 1999 will be above


the average production of the given years,
which of the following could be the minimum
production for 1999?
1)

1 0 50 0 0

2) 1 2 00 0 0

3)

1 0 09 0 0

4) 1 3 00 0 0

1 5 . What was the approximate percentage


increase in production from 1993 to 1994
1)

60

2) 4 0

3)

110

4) 3 0

Ans:11.(3)

IV. Directions (Qs. 16-20):- Study the following graph carefully and answer the questions
given below it:

Average production

80 110 100 130 65 120


6

605
6

INCOME AND EXPENDITURE OF A


COMPANY OVER THE YEARS
(In lakhs Rupees)
INCOME
EXPENDITURE

= 100.833 thousand tonnes


= 100900 tonnes

60

60

ie. production in 1993, 1995 and 1997


was below the average.
Production in 1996 = 130 thousand

Production in 1997 = 65 thousand tonnes

130 65
decrease= 130 100
=

65
x100 = 50%
130

13.(2). Total production in 1993 and 1998


= 80+120 = 200

50
x 200 100
50% of 200 =
100
ie, the production in 1995 was 50% of
the total production in 1993 and 1998.

50
Rupees in Lakhs

12.(2)
tonnes.

60

40

40

40

35
30

30

30
20

20

1994

1995

1996 1997
Years

1 6 . What was the difference in profit between


1995 and 1996?
1)

Rs. 10 lakhs

2) Rs. 5 lakhs

3)

Rs. 15 lakhs

4) No profit

1 7 . In the case of how many years was the


income more than the average income of the
given years?

14.(3).Average production = 100900 tonnes.

1)

one

2) t w o

Therefore the minimum production in


1999 will be 100900 tonnes

3)

three

4) four

15.(2). Production in 1993 = 80 thousand tonnes.


Production in 1994 = 110 thousand tonnes

1 8 . What was the percentage increase in


expenditure from 1996 to 1997?
1)

10

3)

66

2) 33

Increase

30
110 80
x 100 40%
100
=

80
80

1998

2
3

1
3

4) 2 0

1 9 . The income in 1996 was equal to the


expenditure of which of the following years?
1)
3)

1994
1997

2) 1 9 9 5
4) 1 9 9 8

2 0 . In which of the following years was the


profit the maximum?
1) 1 9 9 4
2) 1 9 9 5
3) 1 9 9 6
4) 1 9 9 8
Ans:16.(1).Profit in 1995 = 50-30=Rs. 20 lakhs
Profit in 1996 = 40-30 = Rs. 10 lakhs
Difference in profit=20-10=Rs. 10 lakhs
17.(3)
Average income
30 50 40 60 60
=
5

240
48
=
5

Therefore in 1995, 1997 and 1998, the


income is more than the average income
18.(2) Expenditure in 1996 = Rs. 30 lakh
Expenditure in 1997 = Rs. 40 lakh
Increase

10
1
40 30
x100 33 %
100
=
30
30
3
19.(3)
The income in 1996 was equal to the
expenditure in 1997 ie Rs. 40 lakhs.
20.(4)
The profit is maximum in the year
19 98 .
V. Directions (Qs. 21-25) The pie-chart
drawn below shows the spendings of a country on various sports during a particular year.
Study the pie-chart carefully and answer the
questions given below it.
PERCENT OF MONEY SPENT ON VARIOUS
SPORTS FOR ONE YEAR
Te n n is
10%
Others
10%

Golf
12.5%
Bas ket
b all 12.5%
Fo ot Ball
15%

Cricket
25%
Ho ckey
15%

1)

Rs. 2500000

2) Rs. 3750000

3)

Rs. 5000000

4) Rs. 6000000

2 2 . Out of the following, the country spent


the same amount on
1)

Hockey and Tennis

2)

Golf and foot ball

3)

Cricket and Foot ball

4)

Football and Hockey

2 3 . Pie-chart shows that the most popular


game of the country is (on the basis of money
spent)
1)

Cricket

2) Foot ball

3)

Basket ball

4) Hockey

2 4 . The ratio of the total amount spent on


football to that spent on hockey is
1)

1: 15

2) 1: 1

3)

15 : 1

4) 3: 20

2 5 . If the total amount spent on sports during the year was Rs. 12000000, how much
was spent on basket ball?
1)

Rs. 950000

2) Rs. 1000000

3)

Rs. 1200000

4) Rs. 1500000

Ans:21.(4)
The amount spent on Cricket and
Hockey

15000000 x 40
Rs. 6000000
100

22.(4)
The country spent the same amount
on football and hockey
23.(1)

The most popular game is cricket

24.(2)

The required ratio = 15:15 = 1:1

25.(4)

The amount spent on basket ball

12000000 x12.5
100

= Rs. 1500000
21. If the total amount spent on sports during
the year was Rs. 15000000, the amount spent
on cricket and hockey together was

(1) 75 : 79
(3) 79:75

PRACTICE TEST
Directions (Q.1- 5): Study the following table carefully and answer the questions given below:
Number of Males and Females Staying in Various Societies
Societies

Males

Females

A
B

250
400

350
150

C
D

300
280

275
300

E
F

180
325

250
300

Percentage of Children (Males and Females)


in Societies
Male Children Female Children
40%
60%
75%
25%
25%
75%
80%
20%
50%
50%
46%
54%

(5) None of these


4. What is the total number of members staying
in all the societies together ?
(1) 3520
(3) 4100

(5) None of these


5. What is the difference between the number of
male children in Society B and the number of
male children in Society F ?
(1) 84
(2) 14
(3) 96
(5) None of these

(2) 112 : 71

(3) 82 : 243

(4) 71:112

(4) 26

Directions(Q.6 to 10) Study the following


Pie-chart carefully and answer the questions
given below:
Percentage of People in a city working in
Night Shifts from various Industries (Total
number of People-40250 )

Sa

1. What is the respective ratio of the number of


adult females to the total number of female
children staying in all the societies together ?
(1) 243 : 82

(2) 3360
(4) 3000

ing
nk
Ba 14%

Societies Children
A
25%
B
40%
C
16%
D
25%
E
40%
F
24%

(2) 14 :17
(4) 17:14

les

8%

Call Centre
32%

IT 12%
Gaming
18%

(5) None of these


2. What is the total number of female children
staying in all the societies together ?
(1) 314

(2) 433

Percentage of Females from Various


Industries Working in Night Shifts
Industries
Females

(3) 410

(4) 343

IT

20%

Gaming

20%

Call Centre

45%

Sales

60%

Banking

40%

Chemical Industries

15%

(5) None of these


3. What is the respective ratio of the total number
of adult males in Societies A and B together
to the total number of adult males in Societies
E and F together?

6. What is the respective ratio of men to the


women working in night shifts from the Call
Centre industry?
(1) 9 :11
(2) 7:5
(3) 8:13
(4) 11:7
(5) None of these
7. What is the approximate average number of
females working in night shifts from all the
industries together?
(1) 2227
(2) 4481
(3) 3326
(4) 2823
(5) 4107
8. What is the total number of men working in
night shifts from all the industries together ?
(1) 28297
(2) 25788
(3) 28678
(4) 26887
(5) None of these
9. The number of women from the gaming industry
are what percent of the total number of people
working in the night shifts from all the
industries together ?
(1) 5.6
(2) 3.6
(3) 3.2
(4) 4.4
(5) None of these
10. What is the difference between the total
number of men and the total number of women
working in night shifts from all the industries
together ?
(1) 13254
(2) 13542
(3) 13524
(4) 13363
(5) None of these

12.

13.

14.

15.

Directions-(Q. 11 to 15) Study the following


table carefully and answer the questions
given below:
Number of People Working in Various
Departments from Various Organisations
Departments
Organisations
A
B
C
D
E
HR
1050 1015 976
888 1004
Finance
1017 960
786 1025 963
Marketing 1382 1384 1275 1300 1290
Production 1542 1545 1550 1570 1580
Accounts
786 745
801
800
735
Legal
48
54
36
30
53
11. The total number of employees working in the
legal department are approximately what per

16.

cent of the total number of employees working


in HR department of all the organizations
together ?
(1) 4
(2) 8
(3) 12 (4) 6
(5) 10
What is the approximate difference between
the average number of people working in
Marketing and Production departments from
all the organization together?
(1) 578 (2) 231 (3) 330 (4) 1156 (5) 300
What is the respective ratio of the total number
of employees working in organization A to the
total number of employees working in
organization E ?
(1) 225:233
(2) 71: 75
(3) 75 : 71
(4) 233 : 215
(5) None of these
What is the total number of employees from
all the departments working in all the
organizations together?
(1) 26960
(2) 28910
(3) 28190
(4) 29660
(5) None of these
The number of people working in the Finance
department f rom organization B are
approximately what per cent of the total number
of employees working in organization B ?
(1) 12 (2) 15 (3) 20 (4) 17 (5) 25
Directions-(Q. 16 to 20) Study the following
data carefully and answer the questions
given below:
Out of a total number of commuters
commuting daily in a city, 17,171 commuters
commute only by trains daily. 7359
commuters commute only by bikes and
22,077 commuters only by bus. 14,718
commuters commute only by their private cars
and 4,906 commuters commute only by
autos. 7,359 commuters commute only by
taxis. 26,983 commuters commute by bus as
well as trains daily. 9,812 commuters
commute by autos as well as trains daily.
12,265 commuters commute by bus as well
as autos daily.
The total number of commuters commuting
by trains form what per cent of the total number
of commuters commuting daily ?

17.

18.

19.

20.

(1) 22
(2) 44
(3) 14
(4) 36
(5) None of these
The total number of commuters commuting
by autos form what per cent of the total number
of commuters commuting daily ?
(1) 10
(2) 18
(3) 22
(4) 4
(5) None of these
The total number of commuters commuting by
bikes and taxis together form what per cent of
the total number of commuters commuting daily?
(1) 12
(2) 6
(3) 8
(4) 16
(5) None of these
What is the total number of commuters in the
city commuting daily?
(1) 122650
(2) 126250
(3) 162250
(4) 152260
(5) None of these
The number of commuters commuting only
by bus form what per cent of the total number
of commuters commuting daily ?
(1) 40
(2) 22
(3) 32
(4) 18
(5) None of these
Directions-(Q. 21 to 25) Study the following
graph carefully and answer the questions
given below:
Units of Raw Material Manufactured and
Sold by a Company Over the Years (units
in crores)

8
7
6
5
4
3
2
1
0
2003

2004

2005

Manufactured

2006

sold in the year 2005 to the difference between


the number of units manufactured and sold in
the year 2006?
(1) 2 : 3
(2) 1: 2
(3) 1: 4
(4) 3 : 5
(5) None of these
23. What is the respective ratio of the number of
units manufactured in the year 2003 to the
number of units manufactured in the year
2007?
(1) 7:11
(2) 9 :14
(3) 7:9
(4) 9 :11
(5) None of these
24. What is the approximate per cent increase in
the number of units sold in the year 2007 from
the previous year ?
(1) 190
(2) 70
(3) 60
(4) 95
(5) 117
25. What is the difference between the number
of units manufactured and the number of units
sold over the years ?
(1) 50000000
(2) 5000000
(3) 500000000
(4) 500000
(5) None of these
Directions-(Q. 26 to 30) Study the following
table carefully and answer the questions
given below :
Dozens of Eggs produced by various
Poultry Farms over the years
Years
Farms
A
B
C
D
E
F
2000 420
360
396
528
492
444
2001 564
492
576
612
576
540
2002 588
612
624
648
576
564
2003 600
660
648
636
612
600
2004 648
708
684
672
660
672
2005 732
744
720
756
708
720

2007

Sold

21. What is the average number of units sold over


the years ?
(1) 440000000
(2) 4400000
(3) 440000
(4) 44000000
(5) None of these
22. What is the respective ratio of the difference
between the number of units manufactured and

26. What is the average number of eggs produced


by Farm D over the years ?
(1) 7704
(2) 7526
(3) 7732
(4) 7609
(5) None of these
27. The eggs produced by Farm B in the year 2003
are approximately what per cent of the eggs
produced by Farm B over the years ?
(1) 24
(2) 18
(3) 21
(4) 3
(5) 15

28. What is the respective ratio of the dozens of


eggs produced by Farm A to Farm E in the
year 2005?
(1) 60 : 61
(2) 61: 63
(3) 60 : 63
(4) 63 : 61
(5) None of these
29. What is the respective ratio of the dozens of
eggs produced by Farms A, B and C together
in the year 2000 to the dozens of eggs
produced by Farms D, E and F together in
the same year ?
(1) 49 : 64
(2) 51: 61
(3 49 : 61
(4) 51: 55
(5) None of these
30. What is the difference between the total
number of eggs produced by all the farms
together in the year 2001 and the year 2004 ?
(1) 8200
(2) 8280
(3) 8028
(4) 8208
(5) None of these

No. of Companies

Directions-(Q. 31 to 35) Study the following


graph carefully and answer the questions
given below :
Number of Computers Manufactured and
Sold by Various companies Over the Years
(Number in Lakhs)
4.5
4
3.5
3
2.5
2
1.5
1
0.5
0
P

Companies
Manufactured

32.

33.

34.

35.

(4) The difference between the number of


computers manufactured and the number
of computers sold by Company S is 50000
(5) The total number of computers sold by all
the companies together is 1375000
What is the respective ratio of the number of
computers manufactured by Companies R and
S together to the number of computers sold
by Companies R and S together ?
(1) 29 : 26
(2) 9 : 10
(3) 31: 28
(4) 11 : 13
(5) None of these
The number of computers manufactured by
Company T are approximately what per cent
of the number of computers manufactured by
Company R?
(1) 58 (2) 52 (3) 60 (4) 56
(5) 64
The number of computers sold by Company
Q are what per cent of the number of
computers manufactured by Company Q ?
(1) 65
(2) 60
(3) 75
(4) 50
(5) None of these
What is the difference between the average
number of computers manufactured by all the
companies together and the average number
of computers sold by all the companies
together ?
(1) 45000
(2) 40000
(3) 50000
(4) 35000
(5) None of these
Directions(Q. 36 to 40) Study the following
graph carefully and answer the questions given
below.
Number of Students from various Age
Groups studying in various Schools

Sold

31. Which of the following statements is true ?


(1) The number of computers sold by
Company R are 50 per cent of the number
of computers manufactured by it.
(2) The average number of computers
manufactured by Companies R, S and T
together is 475000.
(3) The respective ratio of the number of
computers sold by Company P to
Company Q is 3:7.

4000
3500
3000
2500
2000
1500
1000
500
0
H

I
4-7 Years

Scho o ls
8-11 Years

J
12-15 Years

41. What is the respective ratio of the number of


students from the discipline of Arts and
Commerce together from college A to the
number of students from the same disciplines
from college B ?
(1) 6 :11
(2) 7 : 9
(3) 2:3
(4) 7:10
(5) None of these
42. What is the difference between the total
number of students from the discipline of
Arts from all the colleges together and the total
number of students from the discipline of
Science from all the colleges together ?
(1) 22874
(2) 23863
(3) 22963
(4) 23874
(5) None of these
43. What is the average number of students from
the discipline of Commerce from all the
colleges together?
(1) 9745
(2) 9735
(3) 9720
(4) 9750
(5) None of these
44. Which college has the least number of
students from the discipline of Science ?
(1) A
(2) C
(3) E
(4) D
(5) None of these
45. The number of students from the discipline of
Arts from college D are approximately what per
cent of the number of students from the
discipline of Commerce from the same college?
(1) 61
(2) 58
(3) 53
(4) 63
(5) 49
Directions (Q. 46-50): Study the following piechart carefully and answer the questions given
below:

Bread
20%

Vegetables
30%

Salad
15%
Sprouts
13%

Fis
h

%
t6
ea
M

36. What is the approximate difference between


the average number of students in the age
group of 4-7 years from all the schools and
the average number of students in the age
group of 12-15 years from all the schools ?
(1) 60 (2) 58 (3) 66 (4) 68 (5) 63
37. The total number of students studying in school
H is what percent of the total number of
students studying in school I ? (Rounded off
to two digits after decimal)
(1) 85.16
(2) 84.32
(3) 88.88
(4) 86.11
(5) None of these
38. The number of students studying in school J
in the age group of 12-15 years are
approximately what per cent of the total
number of students in other age groups
together from the same school ?
(1) 32 (2) 41 (3) 28 (4) 37 (5) 47
39. What is the approximate average number of
students studying in all the schools together?
(1) 8329 (2) 8313 (3) 8317 (4) 8309 (5)8333
40. What is the respective ratio of the number of
students in the age-group of 8-11 years
studying in schools H and I together to the
number of students from the same agegroup studying in schools J and K together?
(1) 27 : 23
(2) 27:25
(3) 25 : 23
(4) 23 : 21
(5) None of these
Directions(Q. 31 to 35) Study the following table
carefully and answer the questions given below :
Percentage distribution of students in various
Disciplines from five different colleges
Colleges
Disciplines
Arts Commerce
Science Total
number of
students
A
25
35
40
17500
B
15
45
40
25000
C
15
30
55
35300
D
28
48
24
23000
E
29
30
41
32400

Fruits
11%

5%

46. What is the total number of people preferring


vegetables and those preferring sprouts?
1) 11,900
2) 12,300
3) 12,500
4) 11,500
5) None of these
47. What is the difference between the total
number of people prefferring meat to the total
number of people preferring fish?
1) 150
2) 200
3) 300
4) 350
5) None of these
48. People preferring fruits are approximately what
per cent of the people preferririg vegetables?
1) 48
2) 35
3) 46
4) 37
5) 30
49. Out of the total sample population, how many
people have given preference for fish?
1) 1600
2)1800
3) 1500
4) 1400
5) None of these
50. What is the ratio of the number of people
preferring meat to the number of people
preferring salad?
1) 2 : 5
2) 3 : 5
3) 2 : 3
4) 4 : 3
5) None of these
Directions (Q. 51-55): Study the following
graph carefully to answer these questions.
Production in Lakh Tonnes

300
250
200
150
100
50
0
1997

1998

1999

2000
Years

2001

2002

2003

51. For which of the following pairs of years the


total production of the two commodities
together is equal?
1) 1997 & 2000
2) 2002 & 2003
3) 200l & 2003
4) 1998 & 1999
5) None of these
52. What is the ratio between the total production
of commodities X and Y for all the seven years
together?
1) 26 : 27
2) 13 : 14
3) 27 : 26
4) 14 : 13
5) None of these

53. During which year the percentage increase/


decrease in production of commodity X from
the previous year was the maximum?
1) 1998
2) 2000
3) 2003
4) 2002
5) None of these
54. Approximately, what was the av erage
production (in lakh tonnes) of commodity Y?
1) 225
2) 216
3) 195
4) 185
5) 205
55. What is the ratio between total production of
the two commodities together for years 1997,
1998 & 1999 and the total production of the
two commodities together for years 2001, 2002
& 2003?
1) 9 : 13
2) 13 : 18 3) 18 : 13
Directions (Q. 56-60): Study the following table
carefully to answer these questions.
Subject
Marks obtained out of
History Geogra Science Maths English Hindi

Student (75) phy (75) (120) (150) (80)


(60)
P
45
53
100
117
50
45
Q
56
58
96
132
46
50
T
48
60
112
120
52
42
D
62
67
88
108
48
34
G
66
55
92
140
60
32
F
54
64
108
116
53
40
56. What is the average percentage of marks
obtained by all the students in Science?
(rounded off to two digits after decimal)
1) 99
2) 82.78
3) 88.72
4) 78.88
5) None of these
57. What is the overall percentage of marks
obtained by D in all subjects? (rounded off to
two digits after decimal)
1) 72.88
2) 76.28
3) 73.17
4) 72.68
5) None of these
58. What are the average marks obtained by all
students in English ?
1) 52.5
2) 64.38
3) 56.38
4) 60.5
5) None of these
59. What is the ratio between total marks obtained
in all subjects by F and G respectively ?
1) 87 : 89
2) 89 : 87
3) 67 : 69
4) 69 : 67
5) None of these

60. Which student has scored in Maths closest


to the average marks in Maths?
1) P
2) Q
3) T
4) F
5) None of these
Directions (Q. 61-65): Study the table carefully
to answer the questions that follow.
Standard
I
42
50
40
45
48
52

II
54
60
48
55
55
52

III
48
58
58
46
44
54

IV
58
45
46
40
55
42

V
50
45
42
52
52
60

VI
38
46
54
50
48
54

61. Which standard has the lowest total number


of students from all the given schools together?
1) V
2) VI
3) 1
4) IV
5) None of these
62. Which school has the highest total number of
students from all the given standards together?
1) E
2) F
3) B
4) A
5) None of these
63. What is the ratio of students studying in
Standard III of Schools A and B together to
those studying in standard VI of schools C
and D together?
1) 53 : 52
2) 43 : 47
3) 25 : 27
4) 39 : 38
5) None of these
64. What is the approximate average number of
students studying in Standard I from all
schools together ?
1) 38
2) 50
3) 40
4) 43
5) 46
65. The number of students studying in Standard
IV in School E is what per cent of that in
Standard IV in School D?
1) 128
2) 132.5
3) 124
4) 137.5
5) None of these

g
hi n %
itc 11
St ses
as
Cl

School
A
B
C
D
E
F

Directions (Q. 66-70): Study the pie-chart


carefully to answer the questions that follow.
Percentage of students enrolled in different
Hobby classes in a School.
Total number of students = 3600

Singing
Classes 18%

Painting
Classes 15%
Cooking
a % Classes 22%
m
3
a
Dr es 1
s
s
a
Cl
Dancing
Classes 21%

66. The number of students enrolled in Cooking


Classes is what per cent of that in Dancing
Classes? (rounded off to two digits after
decimal
1) 101.45
2) 104.76
3) 113.84
4) 110.28
5) None of these
67. What is the total number of students enrolled
in Stitching and Drama Classes together ?
1) 684
2) 846
3) 648
4) 864
5) None of these
68. How many students are enrolled in Painting
Classes?
1) 550
2) 480
3) 450
4) 520
5) None of these
69. The number of students enrolled in Painting
Classes is approximately what per cent of that
in Singing classes?
1) 78
2) 92
3) 83
4) 66
5) 72
70. What is the ratio of the number of students
enrolled in Singing and Dancing classes
together to that in Drama classes?
1) 3 : 1
2) 4 : 7
3) 7 : 5
4) 3 : 5
5) None of these

Directions (Q. 76-80) : Study the graph


carefully to answer the questions that follow.
Profit (in lakhs) made by three companies
over the years Profit = Income - Expenditure
Co mpany A

Profit (in lakhs)

Directions (Q. 71-75): Study the table carefully


to answer the questions that follow.
Percentage of men employed in five different
departments of different organisations
Organi Total
Department
sation Number HR Mark Produ Acc IT
of Empleting ction ounts
oyees
A
2600
53 66
83
67
80
B
1750
28 72
94
36
82
C
3000
62 61
91
43
81
D
2400
64 79
79
66
47
E
1900
42 83
87
74
57

Co mpany B

Company C

9
8
7
6
5
4
3
2
1
0
2001

2002

2003

2004

2005

2006

Years

71. What is the average number of men working


in the Accounts department of all
organisations together ?

72.

73.

74.

75.

1) 1624

1
5

2) 1450

2
5

3) 1748

1
5

4) 1330

2
5

5) None of these
What is the total number of men employed in
the Production department of all organisations
together?
1) 11245
2) 10260
3) 10082
4) 9825
5) None of these
Which organisation has the lowest number of
men working in the Marketing department?
1) A
2) B
3) C
4) D
5) E
The number of men in the IT department of
Company D forms approximately what per cent
of that in the same department of Company
B?
1) 68
2) 85
3) 90
4) 62
5) 79
What is the total number of women working in
the HR department and IT department together
from Company C ?
1) 1710
2) 1840
3) 2730
4) 2050
5) None of these

76. Profit made by Company A in the year 2002


was what per cent of the total profit made by
all the three companies in that year?
1) 31.25
2) 28.24
3) 21.43
4) 36.25
5) None of these
77. If the income of Company A in the year 2005
was Rs 13,54,300/-, what was its expenditure
in that year?
l) Rs 9,21,600/2) Rs 8,33,500/3) Rs 6,48,200/4) Rs 7,54,300/5) None of these
78. What is the approximate average profit made
by Company A in all the years together ?
l) Rs 3,98,000/2) Rs 3, 82,000/3) Rs 4,83,000/4) Rs 5,12,000/5) Rs 4,05,000/79. What is the per cent increase in profit of
Company C in the year 2002 from the previous
year ?
1) 7
2) 14
3) 21
4) 28
5) None of these
80. If the expenditure of Company B in the year
2006 was Rs 22,11,430/- what was its income
in that year ?
l) Rs 29,12,260/2) Rs 28, 14,680/3) Rs 32,09,670/4) Rs 27, 11, 430/5) None of these

Directions(Q. 81 to 85) Study the following


graph carefully and answer the questions given
below it.
Production of Six Types of Passenger Cars
Produced by a Manufacturer for Two Years
(in Thousands)

Directions(Q. 86 to 90) Study the following table


carefully and answer ic questions given below
Marks Obtained by Six Students in Five Subjects
StudSubjects
ents Phy- Mathe- Chem- Biol- Engl- Total
sics
matics istry
ogy
ish
(700)
(Max. (Max. (Max. (Max. (Max.
Marks Marks Marks Marks Marks
100) 200) 150) 100) 150)
A
60
130
75
45
60
370
B
75
124
90
62
90
441
C
55
108
85
59
132 439
D
65
165
108
48
116 502
E
80
114
124
64
98
480
F
64
144
98
54
106 466
86.

81. For which type of car was there a highest per


cent decrease in production from 2005 to 2006?
(1) B
(2) C
(3) E
(4) F
(5) None of these
82. What was the difference in the number of B
type cars produced by the manufacturers
between 2005 and 2006 ?
(1) 5,000
(2) 20,000
(3) 10,000
(4) 2,500
(5) None of these
83. The number of D type cars produced in 2005
Was approximately what per cent of the
number of E type cars produced in 2006 ?
(1) 94
(2) 90
(3) 85
(4) 105
(5) 100
84. For which type of car was there a highest
per cent increase in production from 2005 to
2006 ?
(1) A
(2) B
(3) E
(4) F
(5) None of these
85. The total production of which of the following
types of cars in 2006 was exactly equal to
the production of E type cars in 2005 ?
(1) B and A
(2) B and E
(3) E and D
(4) F and C
(5) None of these

87.

88.

89.

90.

Approximately what percentage of marks


did D get in English ?
(1) 77
(2) 70
(3) 68
(4) 90
(5) 85
Marks obtained by B in Physics and Biology
together were approximately what per cent of
the total marks obtained in all the five subjects
together ?
(1) 47
(2) 58
(3) 35
(4) 60
(5) 65
In which of the following subjects did C get
more than sixty per cent marks ?
(1) Biology and English only
(2) English only
(3) Biology only
(4) Physics and English only
(5) None of these
In which of the following subjects did F get
less than sixty per cent marks ?
(1) Biology and English only
(2) Physics and English only
(3) Physics and Biology only
(4) Biology only
(5) None of these
Which of the following students did get more
than sixty five per cent marks in total of all
the subjects together ?
(1) D and E only
(2) E and F only
(3) D and F only
(4) C and F only
(5) None of these

Directions(Q. 91 to 95) These questions are


based on the following information. Study it
carefully and answer the questions
In a group of 60 boys, all the boys play at least
one game out of Hockey, Football and
Volleyball. One-third of the boys play only
Hockey, 25% play only Football and 20% play
only Volleyball. 5 boys play both Hockey and
Volleyball, 4 boys play both Hockey and
Football, 2 boys play both Football and
Volleyball, 2 boys play all the three games.
91. Total how many boys play Volleyball?
(1) 17 (2) 16 (3) 21 (4) 23(5) None of these
92. How many boys play either Football or
Hockey or both ?
(1) 48 (2) 42 (3) 27 (4) 31 (5) None of these
93. How many boys play Hockey but not Football?
(1) 31 (2) 20 (3) 25 (4) 33 (5) None of these
94. What is the total number of boys in the group
who do not play Volleyball ?
(1) 20 (2) 39 (3) 35 (4) 43 (5) None of these
95. How many boys play only one game ?
(1) 35 (2) 48 (3) 32 (4) 47 (5) None of these
Directions-(Q. 96 to100) These questions are
based on the data provided separately for each
question.
96. Approximately what is the percentage rise in
total production of the three companies
together from 2005 to 2006 ?
Production (in lakh tonnes) of 3
companies in 2005 and 2006
Year
Company
I
II
III
2005
169
129
87
2006
185
132
110
(1) 18 (2) 22 (3) 8
(4) 15 (5) 12
97. During which year was the percentage
increase in income from the previous
year the highest?
Inco me o f a co mpany o ver the year
300
250
200
150
100
50
0
2001

2002

2003
Years

2004

2005

(1) 2004
(2) 2003 (3)2005
(4) 2003 and 2005
(5) 2002 and 2005
98. What is the ratio between the number of Girls
in Class X and Class VIII respectively ?
Details of No. of Students
Ratio
Total no.
Boys :
Girls
of Students
3
:
4
105
5
:
4
108
6
:
7
104
8
:
7
105

Class
X
IX
VIII
VII

(1) 15 :14
(2) 8 : 7
(3) 9:7
(4) 16:15
(5) None of these
99. Total population of these five states in 2002
is how much more/less than the population
in 2001?
Population (in lakhs) of 5 states
in 2001 and 2002
2001

2002

100
75
50
25
0
A

States

(1) 12.5 lakhs less


(2) 10.5 lakhs less
(3) 12.5 lakhs more (4) 10.5 lakhs more
(5) None of these
100. Which institute has the highest percentage
of candidates qualified over appeared ?
Number of candidates Appeared and
Qualified from five Institutes
Appeared
Qualified
A
1500
875
B
800
520
C
1200
750
D
1600
900
E
1100
650
(1) A
(4) D

(2) B
(5) E

(3) C

ANSWERS

A
B
C
D
E
F
Total

Male

Female Total

250
400
300
280
180
325
1735

350
150
275
300
250
300
1625

600
550
575
580
430
625
3360

Children
150
220
92
145
172
150
929

MaleChildren
60
165
23
116
86
69
519

FemaleChildren
90
55
69
29
86
81
410

1. (5) Reqd. ratio = 1625 : 410 = 325 : 82


2. (3)
3. (4) Reqd. ratio
= (190 + 235) : (94 + 256)
= 425 : 350 = 17 : 14
4. (2) Reqd. number = 3360
5. (3) Reqd. difference
= 165 - 69 = 96
6. (5) Reqd. ratio = 55 : 45 = 11 : 9
7. (1) Reqd. average
40250 12
20
18
20
32
45

6
100 100
100 100
100 100
8
60
14
40
16
15

100 100
100 100
100 100

40250
240 360 1440 480 560 240
6 10000

40250 3320
= 2227
60000
8. (4) Reqd. number
18 80
32 55
12 80

= 40250
+
100 100 100 100 100 100
8 40
14 60
16 85

100 100 100 100 100 100


40250
960 1440 1760 320 840 1360
=
10000
40250
6680 26887
=
10000
9. (2) No. of people working in night shifts = 40250
No. of women working in gaming
20
18

40250
100 100
20 18 40250
100

= 3.6
Reqd.% =
100 100
40250
=

10. (3) No. of women = 40250 -26887 = 13363


Reqd. difference
= 26887 - 13363 = 13524
11. (1) No. of employees working in legal deptt.
= 48 + 54 + 36 + 30 + 53 = 221 and no. of
employees working in H.R.
= 1050 + 1015 + 976 + 888 + 1004 = 4933

Reqd. % =

221 100
= 4 (App.)
4933

12. (2) Average number of people working in marketing


deptt. = 1382 + 1384 + 1275 + 1300 + 1290
5
= 1326.2
Average number of people working in production
deptt. = 1542 + 1545 + 1550 + 1570 + 1580
5
= 1557.4
Reqd. difference = 1557.4 -1326.2 = 231
(App.)
13. (5) No. of employees working in organisation A
= 1050 +1017+1382 + 1542 + 786 + 48 = 5825
No. of employees working in organization E
= 1004 + 963 +1290 + 1580 + 735 + 53 = 5625
Reqd. ratio = 5825 : 5625 = 233 : 225
14. (3) Total no. of employees from all the departments
= 5825 + 5703 + 5424 + 5613 + 5625 = 28190
15. (4) Reqd. % =

960 100
= 17(app.)
5703

16. (2) No. of commuters commuting by train


= 17171 + 26983 + 9812 = 53966
and no. of total commuters commuting by train 122650.

Reqd. % =

53966 100
= 44%
122650

17. (3) No. of commuter commuting by autos


= 4906 + 9812 + 12265 = 26983
Reqd. % =
18. (1) Reqd. % =
19. (1)

26983 100
= 22%
122650

( 7359 7359 ) 100


= 12
122650

20. (4) Reqd.% =

22077 100
= 18%
122650

34. (2) Reqd. % =

35. (5) Reqd. difference

21. (4) Average


(3.5 + 4 + 5 + 3 + 6.5) 1,000,0000
5

3.5 2.5 4 3.75 2.25 3.25 15


. 3.5 35
. 2
=
~
5
5

= 44,000,000
22. (C) Reqd. ratio =

55
. 5 0.5
=
=1:4
5 3
2

23. (1) Reqd. ratio = 4.5 : 7 = 9 : 14


24. (5) % increase =

( 6 .5 3.0 ) 100
3. 0

3.5 100
= 117 (App.)
3

= (3.2 - 2.75) lakhs = 0.45 lakhs = 45000


36. (1) Reqd. difference

2000 3000 2500 3000 2250 3250 1750 3500


~
4
4
= 2625 ~ 2687.5 = 63 (App.)
37. (4) Reqd. %
=

( 2000 3500 2250) 100


3000 2750 3250

775000
= 86.11
1
9000

25. (1) Reqd. diff.


= (4.5 + 5 + 5.5 + 5 + 7) - (3.5 + 4 + 5 + 3 + 6.5)
= Rs. (27-22) crore = Rs. 50000000
26. (1) Reqd. average number
= 528 + 612 + 648 + 636 + 672 + 756
6

3852
=
= 642 dozen
6
= 7704
27. (2)

660 100
Reqd. % =
= 18 (App.)
3576

28. (5) Reqd. ratio = 732 : 708 = 61 : 59


29. (3) Reqd. ratio =

420 360 396


528 492 444

1176
= 49 : 61
1464

30.

(4) Reqd. difference


= (564 + 492 + 576 + 612 + 576 + 540) ~ (648
+ 708 + 684 + 672 + 660 + 672)
= 3360 ~ 4044 = 684 dozen = 8208

31. (5) Total no. of computers sold by all companies


together = (3.25 + 1-5 + 3-5 + 3.5 + 2) Lakhs

38. (1) Reqd. % =

175000
= 32 (App.)
5500

2000 + 3500 + 2250 + 3000 + 2750 + 3250 +


2500 + 3000 + 1750 + 3000 + 2750 + 3500
4
=

33250
= 8313 (Apps.)
4

40.(3) Reqd. ratio


= (3500 + 2750) : (3000 + 2750)
= 6250 : 5750 = 25 : 23
41.(4) Reqd. ratio

(25 35)
17500 100
100
(15 45)
= 7 : 10
25000 100
100
42. (5) Reqd. difference

( 40 25) 17500 ( 40 15)


(55 15)

25000 + 100
100
100

32. (3) Reqd. ratio


= (4 + 3.75) : (3.5 + 3.5) = 7.75 : 7.00 = 31 : 28

2.25 100
= 56.25 = 56 (App.)
4

1750 100
2500 3000

39. (2) Reqd. average number

= 1375000

33. (4) Reqd. % =

15
. 100
60
2.5

35300

(28 24) 23000 ( 41 29)

32400
100
100

= 2625 + 6250+ 14120 + 920 + 3888 = 27803

43. (1) Reqd. average

No. of people preferring vegetables = 9000.

45
30
48
35
100 17500 100 25000 100 35300 100

23000

30
1
32400
100
5

= 6125 + 11250 + 10590 + 11040 + 9720


5

Required %=

3300
100 = 36.66 = 37(approx).
9000

49. 3; No. of people preferring fish = 30,000 5%


= 1500
50. 1; No. of people preferring meat = 30,000 6%
= 1800
No. of people preferring salad

48725
=
= 9745
5

= 30,000 15 % = 4500

44. (4) No. of students in Science of A


40 17500
=
=7000
100
No. of student in Science of B
40 25000
=
=10000
100
No. of students in Scince of C
55 35300
=
=19415
100
No. of students in Scince of
24 23000
D=
=5520
100
No. of students in Scince of E
41 32400
=
=13284
100
45. (2) Reqd. %

Required ratio =

51. 4; Total production of two commodities together


in 1998 = 175 + 175 = 350.
Total production of two commodities together
in 1999 = 150 + 200 = 350.
52. 1; Total production of X for all the seven years
together
= 125 +175 +150 +175+225 + 200+250 = 1300
Total production of Y for all the seven years
together
= 150+175 + 200+150 + 250+225+200 = 1350
Required ratio =

= 58 (App.)
46. 5; No. of people preferring vegetables
= 30,000 30% = 9000
No. of people preferring sprouts
= 30,000 13% = 3900
Required total no. = 9000 + 3900 = 12,900
47. 3; Difference between the total no. of people preferring meat, to the total no. of people preferring fish = 30,000 (6% - 5%) = 300
48. 4; No. of people preferring fruits
= 30,000 11% = 3300.

1300 26

26:27
1350 27

53. 1; In 1998, % increase in production of commodity X =

100
28

23000

48

= 100
23000
100

1800
= 2: 5
4500

(175 125)
50 100
100 =
40%
125
125

which is maximum.
54. 3; Avg production (in lakh tonnes) of commodity
Y =

1350
192.85 = 195 (Approx.)
7

55.2; Total production of the two commodities


together for years 1997, 1998 & 1999 = 125 +
150 + 350 + 350 = 975
Total production of the two commodities together for years 2001, 2002 and 2003
= 225 + 250 + 200 + 225 + 250 + 200
= 450 + 500 + 400 = 1350
Required ratio =

975
= 13: 18
1350

56. 2; Total marks obtained by all the students in


Science

100
(100 + 96 + 112 + 88 + 92 + 108)
120
100
596 = 82.77 = 82.78 (approx.)
120
57. 5; Total marks obtained by D in all subjects

62 4 67 4 88 10 108 2 48 5 34 5

3
3
12
3
4
3

No. of students enrolled in drama classes


= 13% of 3600 = 468
Required total number = 396 + 468 = 864
68. 5; No. of students enrolled in painting classes
= 15% of 3600 = 5400.
69. 5; No. of students enrolled in singing classes
= 18% of 3600 = 648
No. of students enrolled in painting classes = 540

= 82.66 + 89.11 + 73.30 + 72 + 60 + 56.66

433.73
72.28%
=
6
58. 2; Avg. marks obtained by all students in English

100
(50 46 52 48 60 53)
= 80
6
=

386.25
= 64.375 = 64.38 (approx.)
6

59. 1; Total marks obtained in all subjects by


F =54 + 64 + 108 + 116 + 53 + 40 = 435
total marks obtained in all subjects by G
= 66 + 55 + 92 + 140 + 60 + 32 = 445
Required ratio =

435
= 87 : 89
445

60. 3; Avg. marks in Maths =

733
= 122 (approx.)
6

61. 3
62. 2
63. 5; Required ratio = 106 : 86 = 53 : 43
64. 5; Average no. of students in standard 1
=

277
= 46.16 = 46 (approx).
6

65. 4; Required% =

55 100
137.5%
40

66. 2; No. of students enrolled in cooking classes


= 3600 22% = 792
No. of students enrolled in dancing classes
= 3600 21% = 756
Required % =

792 100
= 104.76%
756

67. 4; No. of students enrolled in stitching classes


= 11% of 3600 = 396

Required % =

540 100
= 83.33 = 83(approx)
648

70. 1; Total no. of students enrolled in singing +


dancing classes = 648 + 756 = 1404
No.of students enrolled in drama classes= 468
Required ratio = 1404 : 468 = 3 : 1
71. 5; Total no. of men working in accounts department of (A + B + C + D + E) organization
= 1742 + 666 + 1290 + 1584 + 1406 = 6688
Required average = 1337

3
5

72. 3; Total no. of men employed in the production


department of all organisations together =
2158 + 1645 + 2730 + 1896 + 1653 = 10082
73. 2; Number of men working in the marketing department in Organisation B
= 1750 72% = 1260.
74. 5; Required %=

1128 100
= 78.60%
1435

= 79% (approx)
75. 1; Total no. of women in (HR + IT) department of
company C = 1 140 + 570 = 1710.
76. 1;
Profit made by company A in the year 2002
= 5 lakhs
Total profit made by all the three companies
in the year 2002 = 16 lakhs
Required % =
77. 4

78. 3

5
100 = 3 1.25
16

79. 2

80. 5; Income = Rs 26,11,430


81. (3)

82. (4)

83. (5)

84. (4)

85. (2)

86. (1)

87. (3)

88. (2)

89. (4)

90. (5)

91. (3)

92. (5)

93. (3)

94. (2)

95. (4)

96. (5)

97. (3)

98. (1)

99. (3)

100. (2)

Bank Probationery Officer


QUANTITATIVE

APTITUDE
MISCELLANIOUS

BINARY NUMBER SYSTEM


This system has a base 2, using only 0
and 1. The electric switch has an analogy with
binary system. It is either close. i.e. ON representing 1 or it is OFF representing 0. It is this
analogy which is responsible for the construction of powerful digital computers.
Conversion of Decimal Number to Binary
Numbers
A popular method to convert decimal
numbers to binary numbers is to progressively
divide the decimal number is to progressively
divide the decimal number by 2 ; writing down
the remainders when taken in the reverse order
will form the binary number.
Example : method to convert decimal
numbers to binary numbers by 2 ; writing down
the remainders when taken in the reverse order,
will form the binary number.
11 + 2 = 5 with the remainder 1
5 + 2 = 2 with a remainder 1
2 + 2 = 1 with a remainder 0
1 + 2 = 0 with a remainder 1
Now we place the remainder in the reverse order which give 10112 binary number
equivalent to the decimal number 1110

IV.

Add the remaining weights to obtain the


decimal equivalent.

Example (1) : Conversion of binary


number 1012 to its decimal equivalent.
Step

Step

II

Step

III

Step

IV

1=5

i.e. 510 is decimal equivalent of binary


number 1012
Example (2) : Conversion of binary
number 101012 to its decimal equivalent.
Step I

Step II :

16 8

Step III :

16 /

Step IV :

16 +

1 = 21

10

Binary Addition
Rule

Example
1) Add 10102 and 1012

Conversion of Binary Number to Decimal


Number

1010 +

Following four steps can be used to convert a Binary Number to Decimal Number.

11112

101
(2) Add 101102 and 1012

I.

Write the binary number.

II.

Just under the binary write 1, 2, 4, 8,


16...... starting from right to left.

10110 +

If a Zero appears in a digit position cross


out the decimal weight for that position.

110112

III.

101

PRACTICE TEST
Directions (Qs. 1-10) : In a certain code, the symbol for 0 (zero) is * and for 1 is . There are
no other symbols for other numbers and all numbers greater than 1 are written using these two
symbols only : The value of symbol 1 doubles itself every time it shifts one place to the left. Thus
0 is written as *
1 is written as
2 is written as *
3 is written as
4 is written as ** and so on.
1.

If ** is added to *, what will be the result ?


(1) *

2.

(5) None of these

(2)

(3) *

(4)

(5) None of these

(2) **

(4) **

(5) None of these

(3) **

(2) *

(3) **

(4) **

(5) None of these

(2)

(3)

(5) None of these

(4) *

(5) None of these

(4) **

(5) None of these

(4) *

(5) None of these

(4)

(2) **

(3) **

Find the value of the expression, + 10 + * 4


(1)

9.

(4) *

(1) **
8.

(3) **

+ - = ?

(1)
7.

(2) ***

Which of the following will represent the value of the expression 4 9 + 3 ?


(1) *

6.

(5) None of these

Which of the following will represent 19 ?


(1) **

5.

(4) ***

If is divided by **, find the result,


(1) **

4.

(3) *

Which of the following will represent 25% of 36 ?


(1) **

3.

(2) **

(2) **

(3) *

Which of the following represent 625 ?


(1) **

(2) ***

(3) **

10. Find the value of the expression.



(1) ***
(2) ****
(3) **
(4) **
Direction (Qs . 11-20) : As by the above rule, here
0 is written as @
1 is written as #
2 is written as # @
3 is written as # # and so on. Then answer the following questions.
11. Which of the following is an odd number ?
(1) #@@
(2) #@9@
(3)#@#@
(4) #@@##
12. # @ # @ @ + # @ @ @ @ - # # @ @ = ?
(1) # # @ @
(2) # # @ # @
(3) # # @@@
(4) # @ # @@
13. What percent of # # # # is # # ?
(1) 15
(2) 20
(3) 10
(4) 16
#@@@
14. Find the value of [# @
]
(1) # # @
(2) # @ #
(3) @@@
(4) ###
15. Which of the following is an even number ?
(1) # @ @ #
(2) # @ # #
(3) # # # #
(4) # @ #
16. What is the representation of # # @ # ?
(1) 10
(2) 14
(3) 13
(4) 9
17. What is the ratio # # # and # # # @ ?
(1) 1 : 2
(2) 2 : 1
(3) 3 : 7
(4) 7 : 3
18. Find the value of

(5) None of these

(5) None of these


(5) None of these
(5) None of these
(5) None of these
(5) None of these
(5) None of these
(5) None of these

1
32 10 6
8

(1) @

(2) #

19. Find the value of

(3) @ @

(4) # @

(5) None of these

796 # # @@

(1) # # @ @ @
(2) # # # @ @
(3) # # @ # @
(4) # # # # @
20. if divide # @@@@@ by # # @, what will be the approximate result ?
(1) # # @
(2) # # #
(3) # @@
(4) # @ #
Direction (Qs . 21 - 25) : by the above questions, here
0 is written as $

(5) None of these


(5) None of these

1 is written as
2 is written as $
3 is written as and so on
3 64

21. Find the value of


(1)

3 $$$

(2) $

(3)

(4) $$

(5) None of these

22. If

K 3 48

, find K.

(1) $

(2) $$

(3) $

(4)

(5) None of these

(3) $ $

(4) $

(5) None of these

(3) ( ) $

(4) ( $ ) $

(5) None of these

23. What is represent for 22 ?


(1) $

(2) $$ $

24. Find the value of 13 24 6 - 10 + 7


(1)

(2) $$

25. Four of the following five are alike in a certain way and so form a group. Which is the one does not
belong to that group ?
(1) $$$

(2) $$

(3) $

(4)

(5) $

Directions (Qs. 26-35) : - The answering the questions below, use the following information.
A "B means 'add B to A'

A 'B means 'subtracte B from A'

A @ B means 'divide A by B'

A*B means 'multiply A into B'

26. The average age df 30 boys in a class in equal to 14 years. When the age of class teacher is
included the average becomes 15 years. To find the age of class teacher, which of following is
suitable.
(1) 15' (30 @ 1)

(2) (30 * 14)" (3115)

(3) 15 "(30 * 1)

(4) (30 * 14)' (31 * 14)

(5) None of these

27. A shopkeeper maifcs the price at 15% more than the original price. Due to increase indemand
he further increased the price by B%. How much % profit will he get ?
(1) 15' B" (15 * B)* 100

(2) 15' B" (15 @ B) 100

(3) 15" B" (15 * B)@ 100

(4) 15" B" @ (15 * B)@ 100

(5) None of these

28. Find the compound intrest on Rs.P at R % compounded annually for N years.
(1) P * ( 1 @ R " 100)N * P

(2) P * ( 1 @ R @ 100)N * P

(3) P * ( 1 ' R @ 100)N * "P

(4) P @ ( 1" R* 100)N " P

(5) None of these

29. Express the following equation into the above give rule : 8 4 (3 - 2) 5 + 6 - 9
(1) 8 @ 4 (3 " 2) * 5 " 6 " 9

(2) 8 @ 4 (3 ' 2) * 5 ' 6 " 9

(3) 8 * 4 (3 " 2) * 5 " 6 ' 9

(4) 8 @ 4 (3 * 2) * 5 " 6 " 9

(5) None of these

30. One-fifth of one - fourth of one - ninth of a number R is given by


(1) (1 @ 5)* (1 @ 4)*(1 @ 9) @ R

(2) (1 @ 5)* (1 @ 4)*(1 @ 9)* R

(3) (1 @ 5)* (1 @ 4) @ (1 @ 9) * R

(4) Can't be determined

(5) None of these

31. The LCM of (a + b) and (a - b ) is


(1) (a " b) (a ' b) (a ' b)

(2) (a " b) (a "ab " b) (a " b)

(3) (a " b) (a " b) (a ' b)

(4) (a " b) (a " b) (a ' b)

(5) None of these

32. If a

1
2a 1
1 x
,
find the value a 1 2
2x
a 1

(1) (1" x) (2x) @ 2x'1

(2) (1" x) (x1'2) @ 1'2x

(3) (1" x) (2' x) @ 2x'1

(4) (1" x) (x'2) @ 2x'1

(5) None of these

33. A man walks at the rate of 5km/hour for 6 hours and at the rate of 4km/hour for 12 hours. The
average speed of the man (in/hour) is
(1) (5 * 6" 4* 12) @ (6' 12)

(2) (5*6'4*12) @ (12 "6)

(3) (5* 6"4* 12 ) @ (6 "12)

(4) (5 4" 6* 12) @ (12 "6)

(5) None of these

34. The value of 1 x 1 1 x 2 1 x 3 1 x 4 is


(1) 1"1 @ (x"1)
35. How will the formula
(1) (P @ P)* 100

(2) (x"5) @ (x"1)

(3) 1 @ (x"5)

(4) (x"1) @ (x"5)

(5) None of these

P2
be written in the coded formula ?
100

(2) (P'P)*100

(3) (P*P) @ 100 (4) (P"P)@100

(5) None of these

Directions (Qs.36-45) : To answer the following questions use the information below :
M *N means 'add N to M'
M # N means 'subtract N from M'
M'N means 'multiply M into N'
M N means 'divided M by N'
36. 10% of 20% of 30% of Q can be written as
(1) Q' (10 100)' (20 100)' (30 100)

(2) Q(10 100) (20 100) (30 100)

(3) Q' (10 100)' (20 100) (30 100)

(4) Q' (10 100)' (20 100) (30 100)

(5) None of these


37. Anil buys a toy at 20% discount. He sells it at a profit of 5% on his cost price. What price does
he sell it at if the labelled price of the article be L ?
(1) L # (80 100) # (105 100)

(2) L'(80 100)' (105 100)

(3) L (80'100) (105' 100)

(4) L* (80'100) * (105'100)

(5) None of these


38. One fifth of one-sixth of one -tenth of D is give by
(1) D # (1 5) (1 6) (1 10)

(2) D'(1 5)' (1 6)' (1 10)

(3) D' (1# 5) ' (1 6)' (1 10)

(4) D' (1 5)' (1 6)' (1 10)

(5) None of these

39. Find the average runs scored by Sachin if he scored12, 20 and 75 in three successive innings.

(1) 12* 20 * 75) # 3

(2) (12 * 20 ' 75) 3

(3) (12* 20 * 75) ' 3

(4) 12 # 20 # 75) 3

(5) None of these


40. How will 103% of 92 + 81 % of 103 be written in the coded formula ?
(1) (103 100) 92* (81 100) 103

(2) (103 100) 92* (100 81) 103

(3) (100 103) 92* (81 100) 103

(4) (103 100)* 92* (81 100)103

(5) None of these


41. The area of a circle is 27.4 sqm. Find the circumstance (in m).
(1) 2N * 27.4

(2) 2 27.4

(3) 2 ' ' 27.4

(4) 2 ' ' 27.4 #

(5) None of these

42. A can do a piece of work in 9 days and B in 7 days. How many days will both take to finish the
work ?
(1) 1 (1 9 # 1 7)

(2) 1 (8 * 5)

(3) 1 (1 9*1 7)

(4) 1 (1 11 # 1 8)

(5) None of these


43. If you add 20 to 70, you get A. If you add to 40 to 80 and subtract 32, you get B. How will you
write A B ?
(1) (20 *70 ) (80* 40* 32)

(2) (20*70) (80* 4 # 32)

(3) (20 * 70) (40* 80 # 82)

(4) (20 70) # (40* 80 # 32)

(5) None of these


44. What sum of money (in Rs.) will yield Rs. 350 as interest at the rate of 5% in 3 years ?
(1) 100* 100 80 3

(2) (200 100)(8 3)

(3) (200*100) (8' 3)

(4) (200'100) B (8'3)

(5) None of these


45. Mohan bought a bag with 8% discount on the original price. He got a profit of Rs. 25 by selling it
at a price of 112% of that which he bought it. If the original price is x, how will you write the
equation ?
(1) (x # (8 x) * 100) (12* 100) = 25

(2) (x # (8x) 100) (12 100) =25

(3) (x # (8 x) 100) (12 100) = 25

(4) (x* (8 x) 100) (12 # 100) = 25

(5) None of these

Direction (Qs . 46 -50) : For the following questions the symbols used are , , , with the
following meanings.
A B means A B
A B means A - B
A B means A + B
A B means A B
46. The perimeter of a circle is 100m, find the area of the circle.
(1) 50 50

(2) 50 50

(3) 50 50

(4) 50 50

(5) None of these


47. The perimeter of a rectangle cardboard is A cm. The difference between the lengths of the side is
Mcm. Then the length (in cm) of the cardboard is
(1) (AM) 2

(2) (A M 2)2

(3) (A 2M ) 2

(4) (A 2M ) 2

(5) None of these


48. In a class the average height of a group of 12 persons is 165 cm. If Vimal leaves the group, the
average height reduces to 164cm. Then find the height of Vimal (in cm)
(1) 165 12 11 164

(2) 165 12 11 164

(3) 164 1112 165

(4) 165 12 11 164

(5) None of these


49. The sum of the two numbers is 24. Their difference is 10. Then their product may be expressed
as
(1) (24 24 10 10) 4

(2) (24 2410 10) 4

(3) (24 24 10 10) 4

(4) Can't say

(5) None of these

50. A man purchases a horse of Rs. M and sells it to B at a profit of x % who in turn sells the horse
at a loss of y%. Find the price at which the horse was last sold.
(1) M[(1 100)(1 y 100)]

(2) M[(1 100)(1y 100)]

(3) M[(1 100)(1y100)]

(4) M[(1 100)(1y 100)]


ANSWERS

1.

(3)

1001 + 101 = 1110 = *

2.

(1)

25% of 36 = 9
In the binary form, 9 is written as 1001 = **

3.

(2)

= 15 and * = 5

result is a 3 = 11two

(5) None of these

4.

(3)

5.

(5)

4 9 3 = 4 3 = 12 =11002 = **

6.

(2)

1 3 2
4

3 5 3 15

7.

(4)

8.

(1)

9.

(1)

9 4 6

10. (2)

35 16 21 16 16 14
=
= 16 = 100002 =
14
14

11. (4)

# @ @ # # = 19

12. (3)

The result is 24

****

13. (2)
14. (5)

[(# @)# @ @ @]=

1
4

= 22 = 4 = 1002 = # @ @

15. (5)
16. (3)
17. (1)
18. (1)
19. (2)

1
32 - 10 + 6 = 4 - 10 + 6 = 0
8

796 - 12 =

784 = 28 = 111002

20. (4)

32 + 6 = 5 = 1012

21. (5)

2-2=0=$

22. (2)

48 = 4 3

23. (3)

22 = (10110)2

24. (3)

72

25. (5)

$ = 2 which is a prime no. All others are composite numbers.

26. (5)

27. (3)

28. (2)

29. (5)

30. (2)

31. (3)

32. (3)

33. (1)

34. (2)

35. (3)

36. (1)

37. (2)

38. (4)

39. (5)

40. (1)

41. (3)

42. (3)

43. (2)

44. (5)

45. (2)

46. (2)

47. (5)

48. (1)

49. (2)

50. (4)

Você também pode gostar